Kaplan Questions – Flashcards

Unlock all answers in this set

Unlock answers
question
A 45-year-old woman comes to the physician because of headaches and palpitations. Her blood pressure in the office is 120/70 mm Hg, but when she is symptomatic her pulse is 105/min and blood pressure is 190/140 mm Hg. A CT scan of the abdomen shows an adrenal mass. The hormones secreted by the adrenal mass are derived from which of the following amino acids? A. Arginine B. Glutamate C. Glycine D. Tryptophan E. Tyrosine
answer
E - tyrosine The patient has an adrenal pheochromocytoma that is causing episodic hypertension (leading to headache) and arrhythmia (leading to palpitations). The tumor arises from the sympathetic chromaffin cells in the adrenal gland. Catecholamines secreted by the tumor include epinephrine, norepinephrine, and dopamine. All three of these are derived from tyrosine which is, in turn, derived from phenylalanine.
question
A strain of knockout mice in which the gene encoding phenylethanolamine-N-methyltransferase (PNMT) has been deleted is used to study catecholamine synthesis. The mice are stressed and adrenal glands are dissected and analyzed for catecholamine content. This analysis will most likely show the absence of which of the following in the adrenal glands? A. Dopa B. Dopamine C. Epinephrine D. Norepinephrine E. Tyrosine
answer
C - Epinephrine The enzymatic reactions responsible for catecholamine synthesis (see figure) begin with the conversion of tyrosine to L-dihydroxyphenylalanine (dopa) by tyrosine hydroxylase. This is the rate-limiting step in catecholamine synthesis. Dopa decarboxylase then converts dopa into dopamine. Dopamine beta-hydroxylase is an enzyme located within the vesicle membrane that acts on dopamine as it enters the vesicle to produce norepinephrine. Norepinephrine is stored in the vesicles and released via exocytosis upon demand. However, norepinephrine can diffuse into the cytoplasm, where it may be converted into epinephrine by the enzyme PNMT. Epinephrine may then return to the vesicle, diffuse from the cell, or undergo catabolism.
question
A 32-year-old primigravid woman at 32 weeks' gestation comes to the physician because of mild fever, headache, and a few painful genital lesions. She mentions that she has had these lesions a few times before pregnancy, but they went away. Physical examination shows two vesicular lesions on the labia and one similar cervical lesion. The physician explains that she should consider cesarean section prior to rupture of her membranes. Which of the following best describes the form of axonal transport that contributes to the latency of this organism? A. Transport is anterograde B. Transport is mediated by dynein C. Transport is mediated by kinesin D. Transport moves proteins to the axon terminal E. Transport moves vesicles to the axon terminal
answer
B The correct answer is B. This woman has evidence of a reactivated herpes simplex infection with active lesions, just prior to labor and delivery. This is an indication for treatment with acyclovir and for elective cesarean section. This therapy affords the best chance of minimizing the intrapartum infection of the baby. The soma (cell body) of the neuron, with its many dendrites, resides in a sensory ganglion. Each neuron has a primary axon, beginning at the cell body and extending peripherally. The axon is covered with a myelin sheath. Axons use anterograde and retrograde axonal transport to move subcellular elements toward or away from the axon terminal. Retrograde transport uses microtubules, is mediated by dynein, and transports lysosomes and recycled membrane. Herpesvirus, poliovirus, rabies virus, and tetanus toxin are examples of exogenous substances that affect neuron cell bodies as a result of retrograde axonal transport; this is how they get to the cell bodies to establish latency.
question
A study is conducted in which neurons are labelled with a fluorescent dye that allows for quantitation of the DNA content of each cell. The graphs represent the amount of DNA plotted as a multiple of the normal content of a haploid cell (N). Which of the following graphs best represents the DNA content of these cells? A. Peak at 2N B. Peak at 4N C. Peaks at both 2N and 4N with 2N larger D. Peaks at both 2N and 4N with a small peak inbetween E. Peak between 2N and 4N
answer
The correct answer is A. In the graphs shown, normal post-mitotic cells in G1 or G0 would have a diploid amount of DNA (2N). Cells either in G2 or undergoing mitosis would have double the normal amount of DNA (4N) (choice B). Cells undergoing DNA synthesis (S-phase) have intermediate amounts of DNA between 2N and 4N. Since neurons are primarily postmitotic and do not undergo DNA synthesis or mitosis, almost all the cells should be 2N. Choice C shows cells in both G1/G0 and G2/mitosis. Choice D most likely represents non-neuronal cells such as astrocytes or neural precursors where some cells are actively dividing. The 2N peak represents postmitotic cells, the 4N peak shows cells in G2, and the cells scattered in between the two represent cells actively synthesizing DNA. Choice E demonstrates a majority of cells undergoing DNA synthesis, and is likely to occur only if actively dividing cells are isolated.
question
A 7-year-old girl is brought to the physician because of a small mass on her anterior neck. Physical examination shows a mobile mass on the midline of the neck above the larynx that elevates upon protrusion of the tongue. This mass has most likely developed from a remnant of which of the following embryonic structures? A. First pharyngeal cleft B. First pharyngeal pouch C. Second pharyngeal cleft D. Second pharyngeal pouch E. Thyroglossal duct
answer
The correct answer is E. T he thyroglossal duct does not develop from a pharyngeal pouch in the lateral wall of the pharynx but rather develops as an evagination of the floor of the pharynx in the region where the tongue develops. The adult foramen cecum of the tongue marks the site of this evagination. The distal end of this duct normally forms the thyroid gland; the proximal part of the duct normally degenerates. Failure of a part of the duct to degenerate may lead to a thyroglossal duct cyst or median cervical cyst, as seen in this patient.
question
A 32-year-old primigravid woman at 20 weeks' gestation comes to the physician for a routine prenatal visit. Maternal serum alpha-fetoprotein is elevated. A high-resolution fetal ultrasound shows a 3 cm by 5 cm sac-like structure that contains neural elements overlying the lumbosacral area. Which of the following best explains these findings? A. Abnormal closure of the caudal neuropore B. Abnormal closure of the rostral neuropore C. Abnormal fusion of the lateral body folds D. Failure of absorption of cerebrospinal fluid by the arachnoid villi E. Failure of the intestinal loop to retract from the umbilical cord F. Progressive cavitation of the central canal of the spinal cord
answer
The correct answer is A. The lesion is most likely a meningomyelocele, a herniation of the meninges and spinal cord through a defect in the posterior arch of the vertebrae. Meningomyelocele is associated with increased alpha-fetoprotein (AFP), which may be detected in maternal serum or in amniotic fluid.
question
A male newborn born at 37 weeks' gestation has a large mass over the lumbar region. Examination of the mass shows cerebrospinal fluid-filled meningeal tissue that has herniated through a vertebral defect. The newborn is able to move all extremities and responds normally to external stimuli. Which of the following is the most likely diagnosis? A. Arnold-Chiari malformation type I B. Encephalocele C. Meningocele D. Meningomyelocele E. Spina bifida occulta
answer
The correct answer is C. Normally, the closure of the neural tube occurs during the fourth week of development with the rostral neuropore closing at around the 25th day and the caudal neuropore closing at around the 27th day (see figure). If the tube fails to close properly, a neural tube defect will occur. Failure of rostral neuropore closure results in anencephaly and failure of caudal neuropore closure results in spina bifida. Risk factors for neural tube defects include inadequate folate (inadequate intake or folic acid antagonists such as methotrexate), antiepileptic drugs (e.g., valproate, carbamazepine), and maternal diabetes.
question
A 14-year-old girl is brought to the physician because of headaches and neck pain. She says that the headaches become worse after coughing or sneezing. Physical examination shows an unsteady gait. A CT scan shows there is herniation of cerebellar tonsils through the foramen magnum. No other structural abnormalities are seen. Which of the following best describes this finding? A. Arnold-Chiari type I malformation B. Arnold-Chiari type II malformation C. Dandy-Walker malformation D. Encephalocele E. Holoprosencephaly
answer
A Arnold-Chiari malformations are congenital herniations of hindbrain structures into the spinal canal. In type I Arnold-Chiari malformation, the cerebellar tonsils herniate into the foramen magnum. Type I is more common than type II, but mostly asymptomatic. However, sometimes this becomes symptomatic during adolescence or young adulthood. In symptomatic patients, headache and neck pain may be worsened with Valsalva, sneezing, or coughing. Mild cerebellar symptoms, such as ataxic gait, may be seen. In Arnold-Chiari type II (choice B), parts of the hindbrain, cerebellar vermis, and fourth ventricle herniate into the foramen. This second type is commonly associated with meningomyeloceles. Other neonatal presentations associated with Arnold-Chiari malformation include hydrocephalus (because of cerebrospinal fluid obstruction) and brainstem dysfunction (causing stridor and poor swallowing). Milder cases may present later in life with spinal cord or cerebellar symptoms.
question
A 58-year-old man comes to the physician after experiencing numbness and tingling in all four of his limbs. He says that his symptoms have progressed to the point where he is having difficulty driving, walking, and using his hands. Bladder function is intact. MRI examination of the cervical spine shows demyelination in the area labeled by the arrow in the figure. It's the posterior column. Which of the following cells is responsible for normal neuronal myelination in the indicated region? A. Astrocytes B. Dorsal root ganglion cells C. Microglia D. Oligodendrocytes E. Schwann cells
answer
D The arrow is pointing to the fasciculus cuneatus, which is part of the white matter of the spinal cord. Therefore, it is within the central nervous system (CNS). Myelin in the central nervous system is formed by oligodendrocytes. Each oligodendrocyte myelinates several axons. Diseases and conditions that cause central demyelination include subacute combined degeneration (as in this patient), multiple sclerosis, leukodystrophies, and trauma. Because oligodendrocytes provide insulation to the neuronal axon, disruption of myelin slows action potential conduction, thus affecting neuronal signaling.
question
A 32-year-old man is brought to the emergency department after sustaining an injury to the back of the left side of his neck during a physical altercation. A CT scan of the spine shows penetration by a bony fragment into the lateral portion of the dorsal columns. Which of the following functions is most likely to be affected by a lesion at this site? A. Fine motor control of the left fingers B. Motor control of the right foot C. Pain and temperature perception from the right leg D. Proprioception from the left leg E. Sweating of the left face F. Vibratory sense from the left arm
answer
F The correct answer is F. At the level of the neck, the lateral portion of the dorsal columns (funiculus) is comprised of the fasciculus cuneatus. The fasciculus cuneatus, which carries tactile, proprioceptive, and vibratory information from the arms, is found in cervical and upper thoracic levels. The fasciculus gracilis, which carries similar information from the legs, is present at all levels.
question
A 35-year-old previously healthy woman presents with gradually progressive weakness in her extremities. She states that she initially had difficulty keeping her eyes open later in the day. However, the symptoms have progressed to the point where she now has difficulty walking up stairs. She reports no sensory loss in any extremities. Her family history is significant for Hashimoto thyroiditis. EMG shows decreased muscle firing with repetitive stimulation. Laboratory tests are negative for elevated creatine kinase. An electron micrograph of vesicles from an axonal synapse at the site of this patient's pathology is shown. Which of the following neurotransmitters is most likely to be found in these synaptic vesicles? A. Acetylcholine B. Dopamine C. Epinephrine D. Norepinephrine E. Serotonin
answer
A The patient in the vignette provides a characteristic picture of myasthenia gravis (MG). In myasthenia gravis, autoantibodies are directed against the acetylcholine receptor at the neuromuscular junction (nicotinic, Nm receptor) thus producing muscle weakness without sensory findings. Typically, these muscles have decreased firing with increased activity (as opposed to Lambert-Eaton paraneoplastic syndrome in which muscles firing increases with increased activity). The patient also has a family history of Hashimoto's thyroiditis, which is characteristic in patients with MG (they often have a family history of other autoimmune disease). The negative creatine kinase helps differentiate MG from myopathies such as polymyositis or dermatomyositis. Additionally, more frequently used muscles tire most easily which explains the patient's initial symptoms of tired eyelids. Patient's may also get diplopia from fatigue of extraocular muscles. This pathology may then progress to larger muscles.
question
An 18-year-old man who is a football player comes to the physician because of a right elbow injury after a sideways impact from an opposing team's player. He states that since the injury, he has had tingling and numbness of his right hand. Physical examination reveals that the fourth and fifth digits of the patient's right hand are flexed at the interphalangeal joints and extended at the metacarpophalangeal joints. There is also visible bruising of the medial elbow. Further examination of motor function reveals that wrist flexion is slightly weak and that finger abduction is moderately impaired. Damage to which of the following parts of the brachial plexus shown in the diagram is most likely responsible for the symptoms seen in this patient?
answer
The correct answer is D. The history of the elbow injury combined with a presentation of wrist flexion weakness and clawing of digits IV and V are highly suggestive of an ulnar nerve lesion. Typically, damage will occur at the level of the cubital tunnel, where the ulnar nerve passes medial to the elbow.
question
A 74-year-old man is brought to the physician by his wife because of headaches and difficulty walking. He says that he has had fatigue and left-sided headaches for the past 3 months since he was involved in a motor vehicle collision. His wife states that he has been stumbling frequently. To view the physical examination, click on the "Play Media" button. The axons of neurons that are most likely damaged in this patient decussate in which of the following locations? Media: shows Babinski sign on the right. A. Corpus callosum B. Medulla C. Midbrain D. Pons E. Spinal cord F. Thalamus
answer
The patient is exhibiting a Babinski sign, an abnormal reflex in adults. A normal plantar response to stroking the sole of the foot is downward movement of the toes. An upgoing big toe and fanning outward of the other toes is characteristic of a Babinski sign, which indicates an upper motor neuron (UMN) lesion anywhere along the corticospinal tract. UMNs are thought to have inhibitory control over lower motor neurons (LMN), thus removal of UMN input causes overstimulation of LMNs and leads to unmasking of primitive reflexes, such as the Babinski reflex. The corticospinal tract originates in the cerebral cortex, descends through the posterior limb of the internal capsule, then continues down through the midbrain cerebral peduncles, the base of the pons, and the medullary pyramids. In the caudal (closed) medulla, 90% of the fibers decussate in the pyramidal decussation and descend in the spinal cord as the lateral corticospinal tracts, terminating in the ventral horns of the spinal cord.
question
A newborn born at 26 weeks' gestation undergoes a transcranial ultrasound that shows a grade III intraventricular hemorrhage. The newborn's head circumference enlarges more than expected over the following month. A follow-up ultrasound of the head shows increased ventricular dilatation. The lateral ventricles are symmetrically enlarged, but the third and fourth ventricles are normal-sized. At which of the following sites is the obstruction most likely located? A. Arachnoid granulations B. Cerebral aqueduct of Sylvius C. Foramen of Magendie D. Interventricular foramina of Monro E. Subarachnoid space
answer
D
question
A 56-year-old man with a history of chronic hypertension comes to the physician because of recent onset of palpitations. He says that he particularly notices the palpitations when he lacks sleep or drinks too much caffeine. Physical examination shows a heart rhythm that is irregularly irregular but there is no evidence of murmurs or gallops. ECG reveals absent P waves. The patient is started on a beta blocker. Three months later, the patient arrives in the emergency department after acute onset weakness and lethargy. CT scan reveals no hemorrhage but diffusion weighted MRI reveals evidence of ischemic stroke. Absent blood flow to which of the following arteries is most likely to produce the greatest amount of ischemic damage? A. Anterior cerebral artery at its origin from the internal carotid artery B. Anterior communicating artery C. Middle cerebral artery at its origin from the internal carotid artery D. Posterior cerebral artery at its origin from the basilar artery E. Posterior communicating artery
answer
C The circle of Willis is a series of arterial anastomoses that circle around the pituitary stalk at the ventral surface of the brain. If any of the vessels of the circle of Willis become blocked, blood can be routed through the remaining vessels. The object of this question is to determine which vessel does not belong to the circle of Willis. Of the options listed, only the middle cerebral artery (MCA) does not belong to the circle, and instead courses laterally to supply the entire lateral convexity of the brain. Therefore, ligation of this vessel would produce the most ischemic damage.
question
A 52-year-old woman is admitted to the hospital for surgical removal of a brain tumor. She remains conscious during the surgery and the medial aspect of the postcentral gyrus is stimulated during the procedure. Which of the following will the patient most likely do as a result of this stimulation? A. Describe flashes of light B. Describe tactile sensation from her hand C. Describe tactile sensation from her foot D. Move her hand E. Move her foot
answer
C Two pieces of information are necessary to answer this question. First, you must know that the postcentral gyrus of the parietal lobe contains the primary sensory cortex. (Recall that that precentral gyrus of the frontal lobe contains the primary motor cortex.) Second, you must be familiar with the homunculus of the primary sensory cortex, which is very similar to that of the primary motor cortex. The lower limb area is located on the medial aspect of the sensory homunculus in the interhemispheric fissure. Therefore, stimulation of this area could cause her to feel her foot.
question
A 9-year-old girl is brought to the physician because of progressively worsening gait instability over the past 6 months. Her mother states that her daughter has had a history of clumsiness since she was a toddler with frequent falls throughout her development. Physical examination shows wasting of the muscles of the lower legs and examination of her gait reveals bilateral foot drop. Sensory examination shows decreased sensation to light touch on both feet and impaired proprioception. Involvement of which of the following nerves is most likely responsible for this patient's motor examination findings? A. Deep peroneal (fibular) nerve B. Obturator nerve C. Superficial peroneal (fibular) nerve D. Sural nerve E. Tibial nerve
answer
A This girl suffers from hereditary sensory motor neuropathy type I, or Charcot-Marie-Tooth disease. It is the most common inherited neuropathy and results from a mutation in genes expressed by Schwann cells, resulting in demyelination of peripheral nerves. The disease is slowly progressive and most commonly affects nerves of the lower leg, resulting in muscle wasting. These children fall frequently, sustaining injuries to their knees and ankles, and are generally described as being clumsy. Other features include foot drop, pes cavus (high-arched feet), and a stork-like appearance of the legs. Sensory loss may also occur primarily with light touch and proprioception (since these nerves are more likely to be myelinated than the C fibers associated with pain and temperature).
question
A 23-year-old man is brought to the emergency department 20 minutes after being involved in a motor vehicle collision during which he sustained a whiplash injury. The patient says he has a shock-like pain starting in his neck and radiating down his right upper limb. He also reports some weakness in his right shoulder. An MRI of the spine shows a right posterolateral herniation of the nucleus pulposus of the intervertebral disk between vertebrae C4 and C5. Which of the following neural structures is most likely to be injured? A. Anterior ramus C6 B. Posterior ramus C4 C. Spinal cord D. Spinal nerve C4 E. Spinal nerve C5
answer
E The spinal nerve in the intervertebral foramen between vertebrae C4 and C5 is the C5 spinal nerve.
question
A 35-year-old man is brought to the emergency department after receiving a knife wound to the neck. An MRI reveals destruction of the left C6 dorsal root ganglion. Two months later, the axons, dendrites, and nerve cell bodies of this damaged structure have completely degenerated, but there is no other damage. Which of the following changes in the spinal cord will most likely be produced as a result of this injury? A. Axonal loss in the contralateral spinothalamic tract B. Axonal loss in the ipsilateral dorsal columns C. Axonal loss in the ipsilateral lateral corticospinal tract D. Nerve cell body loss in the contralateral ventral horn E. Nerve cell body loss in the ipsilateral dorsal horn
answer
The dorsal root ganglion (DRG) contains cell bodies of primary sensory afferents (the neurons that actually detect the sensory input). The cells are pseudounipolar neurons in which the cell body is in the DRG (see figure below), and the axon is split into two branches. One branch carries sensory information from the periphery and the other branch carries the sensory information into the spinal cord. This question is essentially asking if the DRG and its axonal branches degenerate, in which tract would you see degeneration? The answer is the ipsilateral dorsal columns. The primary afferents that sense conscious proprioception, discriminative touch, and vibration have cell bodies in DRGs, send axons into the spinal cord via the dorsal roots, ascend in the ipsilateral dorsal columns (in this case in the fasciculus cuneatus), and synapse in the medulla (in this case in the nucleus cuneatus). Therefore, if the DRG were destroyed, axonal loss would be predicted in the ipsilateral dorsal columns.
question
A 35-year-old woman with a recent history of bloody diarrhea comes to the physician because of weakness in her legs. She describes her weakness as ascending and slowly progressive over the last 2 weeks. She also recently started noticing some tingling of her hands and feet as well as some weakness in her upper extremities. CSF examination shows elevated protein but normal cell counts. Tissue biopsy of the sural nerve is shown above with an arrow pointing to the likely source of pathology. Which of the following best describes the most likely cells involved in this patient's disease process? A. Astrocytes B. Microglia C. Neurons D. Oligodendroglia E. Schwann cells
answer
E This patient has Guillain-Barré syndrome (GBS), which once was described as a single disorder, but is now thought to be a collection of clinical syndromes that are characterized by an acute inflammatory polyradiculoneuropathy leading to weakness and diminished reflexes. The classic form of GBS is a demyelinating neuropathy with ascending, symmetric weakness with absent or depressed tendon reflexes. Up to two-thirds of patients often report an infectious illness (usually gastrointestinal or upper respiratory) 1-4 weeks prior to the onset of GBS, and Campylobacter jejuni infection is the most commonly identified precipitant. Approximately one-third of patients experience ventilatory failure that requires respiratory support during the course of their illness. Additional symptoms are paresthesias in the hands and feet (80% of patients), dysautonomia (70% of patients), and cranial nerve involvement (50% of patients). Patients often have elevated cerebrospinal fluid (CSF) protein with a normal CSF white blood count (albuminocytologic dissociation). A proposed mechanism for GBS is that an antecedent infection produces an immune response, which cross-reacts with epitopes in Schwann cell surface membrane or myelin. The image in the question shows peripheral nerve axons wrapped in myelin sheaths. The nuclei (marked by arrows) are a part of the cells that produce myelin, the Schwann cells.
question
A 50-year-old man with a past medical history of uncontrolled hypertension and hyperlipidemia is brought to the emergency department with sudden onset acute chest pain and nausea. He is treated for angina pectoris with sublingual nitroglycerin, 100% oxygen by face mask, aspirin, and a beta blocker. His symptoms continue unabated and he begins to vomit while in the emergency department. The patient is started on metoclopramide and his nausea and vomiting improve. Which of the following is the most likely site of action of metoclopramide? A. Floor of the fourth ventricle B. Floor of the third ventricle C. Frontal lobe D. Locus coeruleus E. Pons
answer
A Vomiting can accompany severe visceral pain involving the chest or abdomen. The vomiting appears to be typically mediated by vagal afferents that relay in the chemoreceptor trigger zone (CTZ) located in the heavily vascularized area postrema in the floor of the fourth ventricle. The signals from the chemoreceptor trigger zone are received by the vomiting center located in the lateral reticular formation of the medulla, meaning that the CTZ responds to circulating toxins because it is not behind the blood-brain barrier. Metoclopramide is a dopamine (DA) antagonist that acts at the area postrema to prevent nausea and vomiting.
question
A newborn is born at 38 weeks' gestation with Apgar scores of 9 and 9 at 1 and 5 minutes. Physical examination shows no abnormalities and the newborn is feeding well. Two months later, he is brought to the physician by his mother for a routine well-child examination. The mother states that he no longer feeds well and that he slows down through the feeding, and halfway through he cannot finish. Physical examination shows a weak sucking response and generalized hypotonia and areflexia. An EMG is performed on the limbs and shows fibrillation potentials. A defect in which of the following is most likely responsible for the symptoms seen in this newborn? A. Caudate nucleus B. Cerebellum C. Dorsal horn cells D. Internal capsule E. Lateral corticospinal tracts F. Medial lemniscus G. Pyramidal decussation H. Ventral horn cells
answer
H This baby has evidence of spinal muscle atrophy (SMA) type I, or Werdnig-Hoffman disease. This is due to abnormality of the gene that normally "turns off" perinatal programmed cell death. As a result, neuronal cell death continues postnatally and results in denervation of muscle with resultant atrophy. As such, this represents a disease of the anterior (ventral) horn cells and is an example of a lower motor neuron disease. It has all of the attributes of lower motor neuron disease: flaccid muscle weakness, hypotonia, loss of voluntary movement, loss of deep tendon reflexes, fasciculations, and fibrillation potentials seen on the EMG. Bulbar dysfunction causes poor sucking, reduced swallowing, and respiratory failure. There is no cure, and death usually occurs by age 2 due to respiratory failure.
question
A 30-year-old woman comes to the physician for a routine examination. She says that she jogs 2 to 3 miles daily but recently has had vague lower back pain that radiates down her legs to her feet. Physical examination shows decreased sensation on the skin of the lateral foot. The patellar reflex is normal but the Achilles tendon reflex is decreased. Muscle strength testing shows mild calf muscle weakness but there are no abnormalities of gait. Which of the following nerve roots is most likely affected? A. L1 B. L2 C. L4 D. S1 E. S3
answer
D The segmental motor innervation for the S1 and S2 segments is primarily to the plantar flexors of the ankle. These are primarily the gastrocnemius and soleus muscles, which are innervated by S1 and S2 fibers in the tibial nerve.
question
A 33-year-old man comes to the physician because of severe lower back pain, muscle weakness that began in his legs but now involves the arms as well, and tingling in his extremities. He had a diarrheal illness two weeks ago, but has otherwise been healthy. Physical examination shows generalized weakness, but there is no appreciable sensory loss. Over the next 2 months, the patient recovers, with minimal residual sequelae. The underlying pathogenic mechanism that contributes to this patient's neurological symptoms is most similar to the mechanism associated with which of the following conditions? A. Amyotrophic lateral sclerosis B. Botulism C. Diabetes mellitus D. Multiple sclerosis E. Syringomyelia
answer
D In this case, given a neurologic presentation of limb weakness, one would need to consider the patient's condition be due either to a degenerative (ALS), infectious/toxic (botulism), hyperosmolar/ischemic (diabetes), immunologic/inflammatory (MS), or congenital/structural anomaly (syringomyelia) pathogenic mechanism. The question is trying to elicit an immunologic or inflammatory pathogenic mechanism that can be found with Guillain-Barre syndrome and multiple sclerosis. The patient has Guillain-Barré syndrome (GBS), also known as acute idiopathic inflammatory polyneuropathy. Characteristically, this produces an ascending paralysis, which can eventually involve the diaphragm, leading to respiratory compromise and necessitating intubation. Most cases of Guillain-Barré syndrome resolve spontaneously. A few patients have recurrences, and rarely, patients can die because of respiratory muscle failure (artificial ventilation may be required, so vital capacity should be monitored). This condition, which typically follows an upper respiratory or gastrointestinal infection (e.g., by Campylobacter jejuni, as seen in our patient) by several days to a month, is due to an autoimmune attack on the myelin of peripheral nerves (Schwann cells). In this respect, it is most similar to multiple sclerosis, which is an autoimmune attack on the myelin in the brain and spinal cord (oligodendrocytes).
question
A 67-year-old man comes to the physician because of "shooting pains" in his legs and difficulty ambulating. Recently, he has also experienced urinary incontinence. Physical examination shows small irregular pupils that react to accommodation but not to light. VDRL test is positive. An MRI of the spinal cord will most likely show atrophy of which of the following spinal structures? A. Dorsal column B. Dorsal horn C. Lateral column D. Lateral horn E. Ventral column F. Ventral horn
answer
A The patient has tabes dorsalis, which is a form of neurosyphilis seen 10 to 25 years or longer after primary disease. The pupils described are Argyll Robertson pupils, and are strongly associated with neurosyphilis. These pupils are bilaterally small and do not constrict further with light, but do constrict to accommodation (light-near dissociation). The lesion is likely in the pretectal area of the midbrain. (A mnemonic to remember this is a "prostitute's pupil, which will accommodate but will not react.") With the advent of antibiotics, this is not commonly seen in developed countries. Bilateral degeneration of the dorsal roots and secondary degeneration of the dorsal columns occur.
question
A 42-year-old man comes to the physician because of tingling and weakness in the arms. He denies any recent syncope, fall, or trauma, but describes increased episodes of burning his fingers and hands while cooking lately. There is no significant past medical history. His temperature is 37°C (98.6°F), pulse is 74/min, respirations are 14/min, and blood pressure is 128/72 mm Hg. Physical examination shows some atrophy of the small muscles of the hands, but no motor deficits in the lower extremities. There is absence of pain and temperature sensation in both arms, shoulders, and upper chest, but intact touch, position, and vibratory sensations. Deep tendon reflex is 1+ in biceps brachii, brachioradialis, and triceps brachii. Which of the following findings would most likely be present on the MRI of the head and spine? A. Demyelination of the posterior and lateral columns B. Hyperintense demyelinating plaques in the white matter C. Lacunar infarcts in the thalamus and basal ganglia D. Syrinx within the cervical spinal cord E. Tumor metastasis to the spinal cord
answer
D The patient has syringomyelia, which is the development of a fluid- filled cavity(syrinx) within spinal cord that usually involves the cervical region. Interruption of spinothalamic decussation (in the anterior white commissure) can result in decreased pain and temperature sensation in a cape-like distribution across shoulders, arms and upper torso (cape distribution). Light touch, vibration, and position sensory are preserved (dissociated sensory loss). As the cavitation expands, lower motor neurons in the ventral horn may be affected and can cause atrophy of hand muscles, which can progress to the forearms and shoulders. Growing cavitation can also involve the corticospinal tracts, leading to upper motor neuron signs.
question
A 2-year-old boy with a history of meningomyelocele complicated by meningitis is brought to the physician for follow-up examination. Physical examination shows increased head circumference and impaired bilateral upward gaze. CT scan of the head demonstrates enlargement of both lateral ventricles and the third ventricle. The fourth ventricle is normal-sized. There also appears to be a mass arising from the pineal body. An obstruction at which of the following sites best explains these findings? A. Arachnoid villi B. Cerebral aqueduct C. Central canal D. Foramen of Luschka E. Foramen of Magendie F. Foramen of Monroe
answer
This child most likely has a noncommunicating hydrocephalus that has developed as a result of a pinealoma. A pinealoma is a neuroendocrine tumor, which can arise from the pineal body, a diencephalon structure responsible for melatonin production and maintenance of circadian rhythm. The pineal body overlies the superior colliculus of the midbrain, and a tumor there can compress the midbrain. Other than hydrocephalus, presenting symptoms include precocious puberty, nystagmus, and impaired conjugate upward gaze (Parinaud syndrome) due to superior colliculus impingement.
question
A 38-year-old woman comes to the physician because of blurry vision in her left eye. She has also had right arm weakness over the last year and a half. An MRI scan of the brain shows multiple periventricular lesions in the subcortical white matter. Which of the following histopathologic features would most likely be prominent in one of the older plaques in this patient? A. Acute degeneration of myelin B. Complete loss of axons C. Gliosis D. Histiocytic infiltration E. Lymphocytic infiltration
answer
C Demyelinating plaques in multiple sclerosis (MS) mature through an orderly sequence of events. Acute plaques show dense lymphohistiocytic infiltration and active digestion of myelin byproducts. As inflammatory infiltration and myelin breakdown abate, older plaques show hyperplasia and hypertrophy of astrocytes transform the plaque into a gliotic area, in which axons are relatively preserved but oligodendroglial cells are greatly diminished. The term reactive gliosis indicates proliferation of glial cells and is usually used as a synonym of astrocytosis. Astrocytosis occurs as a nonspecific reaction to any damage to the CNS (e.g., loss of neurons following hypoxic injury, inflammation caused by infectious agents, and neoplastic invasion). The hypertrophic astrocytes acquire abundant eosinophilic cytoplasm.
question
A 40-year-old man develops progressive weakness culminating in paralysis. Physical examination shows muscle atrophy, fasciculations, and hyperreflexia. Over the course of the next few years, the disease progresses to complete paralysis of all voluntary muscles, and he dies in respiratory failure. Microscopic examination of tissue obtained at autopsy shows inclusion bodies within neuronal cell bodies and axons. Which of the following CNS sites would likely show the most marked neuronal loss? A. Caudate nucleus B. Cerebellum C. Globus pallidus D. Spinal cord E. Substantia nigra
answer
D Our patient presents with amyotrophic lateral sclerosis (ALS), a devastating disease characterized by degeneration of both upper (UMN) and lower motor neurons (LMN). Depending on the stage of this disease, patients may experience UMN symptoms (hyperreflexia, spasticity, Babinski reflex) and/or LMN symptoms (weakness, muscular atrophy, fasciculations). In this case, the patient is displaying fasciculations and muscle atrophy (signs of LMN lesion) and hyperreflexia (sign of UMN lesion). On the USMLE, if you see both UMN and LMN disease in a patient, think ALS. Anatomically, cell bodies of UMNs are located in the motor area of the cerebral cortex (precentral gyrus) and axons descend the spinal cord in the corticospinal tract; LMNs are located in the ventral (anterior) horn of the spinal cord (choice D). Patients present without sensory, cognitive, or oculomotor deficits.
question
A stillborn male at 30 weeks gestation age has multiple physical deformities. He has microcephaly, a cleft lip, cleft palate, and eyes that are very small, with fissures of the iris, shallow supraorbital ridges, and slanted palpebral fissures. His ears are low-set and malformed. Each hand has six fingers and a simian crease. Which of the following is most likely associated with this congenital disorder? A. Anencephaly B. Encephalocele C. Holoprosencephaly D. Lissencephaly E. Polymicrogyri
answer
C This is the typical presentation of trisomy 13, also known as Patau syndrome. Affected infants usually die before 1 year of age, usually secondary to multiple congenital anomalies, particularly severe congenital renal and heart defects (VSD most common). Other features include holoprosencephaly, microphthalmia, microcephaly, cleft lip/palate, polydactyly, congenital heart disease, and severe mental retardation. A useful mnemonic for Patau syndrome is M1CRoCePHaly: Mental retardation, 13 chromosome, Cardiac defects, Renal defects, Cleft lip & palate, Polydactyly, Holoprosencephaly. Note: simian crease, as seen in our patient, can also be seen in Down syndrome.
question
A 12-year-old girl is brought to the physician because of stumbling and falls. She has a broad-based, slow, clumsy gait with a tendency to lurch from side to side. Several years later, she develops clumsiness of fine movements and a coarse intention tremor of the upper limbs. Her illness progresses slowly to kyphoscoliosis and foot deformities. She eventually becomes bedridden and dies in the fourth decade of life from congestive heart failure. Which of the following neurologic findings is most likely on autopsy? A. Atrophy of the caudate nucleus B. Atrophy of the spinal cord C. Depigmentation of the substantia nigra D. Diffuse cortical atrophy E. Selective frontal and temporal lobe atrophy
answer
B The disease is Friedreich ataxia, the most common form of hereditary ataxia. The condition is an autosomal recessive, progressive, and irreversible neurodegenerative disorder caused by a mutation in the frataxin (FXN) gene on chromosome 9, which codes for the mitochondrial protein, frataxin. The majority of these patients have an expanded trinucleotide (GAA) repeat in this gene, which leads to decreased expression of frataxin, an iron-binding protein. One theory is that decreased frataxin levels produce mitochondrial iron overload, leading to oxidative stress. The large myelinated axons in the peripheral nerves, posterior (dorsal) columns, spinocerebellar tracts, and corticospinal tracts are affected, causing the spinal cord and dorsal root ganglia to atrophy. The cerebellum also degenerates to a lesser extent.
question
A 9-year-old boy develops progressive difficulty walking and coordinating his limbs. The dysfunction spreads from the lower extremities to the arms and trunk. Physical examination shows ataxic gait, nystagmus, and decreased deep tendon reflexes. Genetic testing shows a trinucleotide repeat expansion in the frataxin gene. Degeneration of cells in the dorsal root ganglia will most likely produce transsynaptic degeneration in which of the following nuclei in this patient? A. Cerebellar dentate nucleus B. Gracile and cuneate nuclei C. Pontine nuclei D. Subthalamic nuclei E. Vestibular nuclei
answer
B This patient has Friedreich ataxia (FA), an autosomal recessive neurodegenerative disease associated with an unstable trinucleotide repeat (GAA) in the FXN gene on chromosome 9, resulting in decreased expression of the FXN gene product frataxin. The characteristic symptoms of FA include progressive limb and gait ataxia, dysarthria, loss of proprioception and vibration sense, absent deep tendon reflexes in the legs, and extensor plantar responses. Onset is generally between the ages of 8 and 15. The greater the trinucleotide expansion, the earlier the age of onset and the more severe the disease. Most patients are wheelchair bound by age 30 years. Symptoms in FA are caused in part by degeneration of large, myelinated sensory neurons in the dorsal root ganglia (DRG). Small, unmyelinated pain/temperature fibers are spared. The central axons of DRG neurons ascend in the posterior (dorsal) column within the gracile or cuneate fasciculus and project to their respective nuclei in the ipsilateral medulla. Transsynaptic degeneration occurs when a nucleus in the CNS degenerates in response to degeneration of its afferent pathway. Thus, as a result of loss of the DRG neurons, such as that which occurs in Friedreich ataxia, neurons in the gracile and cuneate nuclei will degenerate in response to loss of afferent input from the DRG neurons.
question
A 22-year-old man comes to the physician because of numbness in his hands. He has burned his hands on two occasions while using the stove at home. Neurologic examination shows bilateral loss of pain and temperature sensation in the upper extremities with normal touch sensation. No motor abnormalities are observed. Which of the following disorders is most frequently associated with these findings? A. Broca aphasia B. Chiari malformation C. Horner syndrome D. Spina bifida occulta E. Tabes dorsalis
answer
B The patient most likely has syringomyelia, in which softening and cavitation around the central canal of the spinal cord damages crossing fibers of the spinothalamic tract. This results in bilateral loss of pain and temperature (but not touch) sensation in the upper extremities (because the cavitation usually occurs in the cervical spinal cord). Many of the patients with syringomyelia have Chiari malformation, in which there is a congenital protrusion of the cerebellum and medulla through the foramen magnum. It occurs most commonly with Chiari I (up to 70% of patients), but is also common with Chiari II. Syringomyelia may result from the obstruction of CSF circulation from the posterior fossa. Trauma (e.g. whiplash) and prior meningitis can also cause syringomyelia.
question
A 28-year-old man comes to the physician because of ptosis, muscle weakness, and facial muscle atrophy. The physician notices that the patient had a stiff and delayed relaxation after shaking hands upon greeting. The patient states that his father and brother have similar problems, but his symptoms appear more severe. His birth and childhood were normal. His temperature is 37°C (98.6°F), pulse is 72/min, respirations are 14/min, and blood pressure is 126/78 mm Hg. Physical examination reveals bilateral ptosis and distal limb muscle weakness. Gait, cranial nerves, and funduscopic examination are normal. Which of the following would most likely be associated with this patient's condition? A. Antibodies to acetylcholine receptors B. Antibodies to Hu antigen C. Decreased dystrophin production D. Demyelination in the central nervous system E. Ring fibers seen on muscle biopsy
answer
E The patient has adult onset myotonic muscular dystrophy (DM1), an autosomal dominant disorder with CTG trinucleotide repeat expansion (>50 CTG repeats). Alleles >80 repeats tend to expand in maternal transmissions. Symptoms start in the second decade. Clinical features include frontal balding, eyelid and facial muscle atrophy, cataracts, distal muscle weakness, infertility, and insulin resistance. Proximal muscle weakness occurs later. Cardiac involvement with delayed conduction can also occur. Myotonia (delayed muscle relaxation) involves the tongue, forearm, and hand (delayed relaxation after hand shake). Histologically, ring fibers and central nuclei are present. Necrosis, regeneration, and increased collagen are not as severe as in Duchenne muscular dystrophy.
question
A 7-year-old boy is brought to the physician because of progressively worsening "uncoordinated" arm and leg movement and slurred speech. Physical examination shows preserved language ability but gross dysarthria. The boy has a staggering gait with a limited ability to walk unassisted. Motor examination shows normal bulk and tone, but strength is 3/5 in distal muscle groups and 4/5 in proximal groups in the upper and lower extremities. Patellar and ankle reflexes are absent; a plantar extensor response is present bilaterally. Vibration and position senses are impaired, but sensation of pinprick is intact. Which of the following is most likely a characteristic of this patient's illness? A. Autosomal dominant inheritance B. Delayed muscle relaxation after strong contraction C. Occurs after viral infection D. Results from a trinucleotide repeat expansion E. Sexually transmitted disease
answer
D Friedreich ataxia is characterized by progressive ataxia and severe dysarthria. Onset is usually in childhood, and classic findings also include loss of reflexes, spasticity, extensor plantar responses, and impaired vibration and position sense. This results from a loss of large myelinated axons in peripheral nerves, sparing unmyelinated fibers. Demyelination is evident in the dorsal (posterior) columns, corticospinal, and spinocerebellar tracts. Neuronal loss in the cerebellum also occurs. Loss of cells in the nuclei of CN VII, X, and XII causes dysarthria, swallowing difficulties, and facial weakness. Foot deformities, scoliosis, diabetes, and cardiac complications may be present in some individuals. This disease is unique because it is the first known recessive genetic disease that is caused by a trinucleotide repeat expansion; most are autosomal dominant. Approximately 98% of all carriers have a GAA expansion. Friedreich ataxia is an autosomal recessive condition resulting from a genetic defect in the frataxin gene. Common among trinucleotide repeat diseases is the phenomenon of anticipation, which is when recent generations of a pedigree develop the disease at an earlier age or with greater severity than do earlier generations. It is thought to occur because of the gradual expansion of the trinucleotide repeat polymorphisms within or near a coding gene.
question
An 18-year-old man comes to the emergency department because of shortness of breath and difficulty walking. Two days ago he noticed weakness in his legs, which has worsened. He is now unable to walk because of weakness. He had food poisoning and diarrhea 2 weeks ago that resolved on its own and did not require treatment with antibiotics. At that time, stool cultures revealed a Gram-negative curved rods grown on MacConkcy agar, which were catalase and oxidase positive. His temperature is 37.1°C (98.7°F), pulse is 74/min, respirations are 10/min, and blood pressure is 128/78 mm Hg. Oxygen saturation on room air is 80%. Physical examination shows use of neck accessory muscles, lungs clear to auscultation, and symmetric hyporeflexia of the lower extremities. Sensory examination is normal. Which of the following is the most likely location of the disease process? A. Lateral corticospinal tracts B. Neuromuscular junction C. Peripheral nerves D. Precentral gyrus E. Skeletal muscles
answer
C This patient has developed Guillain-Barré syndrome (GBS), also known as inflammatory polyneuropathy. His presentation is classic: absence of fever, rapidly evolving ascending weakness/paralysis with symmetric hyporeflexia but normal sensation. Sensory symptoms (e.g., paresthesias, numbness, pain) can occur, but the motor symptoms predominate. The syndrome frequently follows a Campylobacter jejuni gastrointestinal infection (as seen with our patient) or a respiratory tract infection (CMV, HSV, EBV, Mycoplasma pneumoniae) and may evolve into complete paralysis with respiratory failure (affects the diaphragm). Our patient has decreased oxygen saturation, use of neck accessory muscles, and decreased respiration rate due to diaphragm weakness. GBS is thought to be an autoimmune disease in which antibodies, produced from the previous infection, cross react with human nerve gangliosides (molecular mimicry). The clinical course is correlated with a chronic inflammatory infiltrate and demyelination of peripheral nerves.
question
A 45-year-old man with a history of a stroke comes to the physician for a follow-up examination. Physical examination shows hyperreflexia of the patellar tendon tap. The spasticity reflects loss of inhibitory influences that normally limit responses to afferent signals that convey information about muscle stretch. Which of the following structures within the muscle actively initiates the abnormally brisk patellar reflex arc in this patient? A. Alpha motor neurons B. Extrafusal fibers C. Gamma motor neurons D. Golgi tendon organs E. Spindle afferents
answer
The correct answer is E. Several mechanisms contribute to the regulation of muscle stretch. Spindle afferents and other sensory fibers innervate the intrafusal fibers and detect changes in stretch of the muscle by detecting tension of the intrafusal fibers. When the patellar tendon is tapped, the quadriceps muscle is stretched. The increased stretch is detected by the spindle afferents, which enter the spinal cord through the dorsal root and pass into the ventral horn where they synapse with and activate the alpha motor neurons that innervate the muscle. The stretch of the muscle thereby initiates contraction of the muscle to shorten it to its initial length.
question
A 3-year-old girl is brought to the physician because of delayed motor and mental development. Her blood pressure is 160/100 mm Hg. She appears cachectic. Physical examination shows a palpable abdominal mass on the right with no accompanying tenderness or guarding. CT scan of the abdomen shows a unilateral mass in the right kidney that does not cross the midline. Treatment is begun with a chemotherapeutic regimen that includes vincristine. The family has questions regarding this new medication. Which of the following is a potential adverse effect of this drug? A. Arrhythmia B. Cushing syndrome C. Hemorrhagic cystitis D. Hyperpigmentation E. Peripheral neuropathy F. Pulmonary fibrosis
answer
E The correct answer is E. This patient presents with Wilms tumor, the most common renal malignancy in children and the fourth most common childhood cancer. A classic presentation of this disease is as a painless, palpable abdominal mass, which does not cross the midline. Treatment often involves the use of vinca alkaloids such as vincristine and vinblastine, which act on microtubules to inhibit spindle formation during mitosis, arresting dividing cells in M phase. Vincristine is indicated for the treatment of Wilms tumor, Hodgkin disease and non-Hodgkin lymphoma, and various types of leukemia and other malignancies. Because neuronal axons rely heavily on axonal transport using microtubules, the drug can potentially lead to peripheral neuropathy as well.
question
A 50-year-old man is brought to the physician by his wife because of progressive weakness of his hands and arms and difficulty walking. She says he has also had progressive difficulty swallowing and talking over the past year. Vital signs are unremarkable. Neurologic examination shows moderate immobility of the soft palate bilaterally in response to phonation or the gag reflex. The tongue also appears mildly atrophic with bilateral fasciculations. There is spasticity with flexion and extension at the elbows, and moderate spasticity with flexion and extension at the knees. Deep tendon reflexes are moderately increased and Babinski responses are present bilaterally. Vibration testing at the fingers and toes is normal, as is testing for proprioception and light touch. Which of the following drugs will most likely prolong this patient's life? A. Baclofen B. Clonazepam C. Dantrolene D. Riluzole E. Tizanidine
answer
The correct answer is D. This patient has amyotrophic lateral sclerosis (ALS), a disorder of the lower motor neurons (located in the ventral horn, producing muscular atrophy and fasciculations), and upper motor neurons (corticospinal neurons, producing hyperreflexia, hypertonia, clasp knife spasticity, and the Babinksi sign). Bulbar involvement may lead to dysarthria and dysphagia. Advanced ALS may lead to respiratory compromise due to muscle weakness and aspiration. Sensation is commonly normal and intact, which distinguishes this disease from peripheral neuropathies. Currently there is no cure for ALS and the primary goal of medical care is to provide support and palliative intervention. Riluzole is the only agent that has been shown to extend the survival of patients and/or the time until tracheotomy. ALS may involve excitatory toxicity to neurons caused by glutamate. Riluzole is a glutamate release inhibitor and glutamate receptor blocker, as well as a sodium channel blocker.
question
A 10-day-old newborn is brought to the physician by his mother for a routine well-child examination. The patient's mother states that he jumps, extends his arms with fingers spread, and then jerks his arms to his chest as if he were afraid every time she drops something or when the dog barks. The physician explains that the behavior is normal and will most likely be naturally extinguished around which of the following ages? A. 1-2 months B. 4-6 months C. 8-10 months D. 12-18 months E. Persists through life
answer
B The Moro or startle reflex can be elicited in the infant by any startling event, such as making a loud noise or changing the baby's position (e.g., holding the infant supine in one's arms and dropping the baby's head slightly but suddenly). This reflex consists of extension and abduction of the arms with fingers spread, followed by flexion and adduction of the arms. This is a normal reflex that appears during gestation and will normally disappear between 4-6 months.
question
A 6-year-old boy is brought to the physician because of enuresis that seems to occur 4 to 5 times a week and sometimes twice in the same night. The mother states that the problem began over the past few months, after she gave birth to a baby girl. She says that the boy is very embarrassed about his enuresis and is afraid that his friends will find out. Which of the following would be the most appropriate next step? A. Arrange to interview the boy about his condition B. Direct the mother to have a serious discussion with her son about whether anything is bothering him C. Prescribe a course of imipramine for the boy D. Refer the mother to a seminar on enuresis and behavioral techniques to treat this condition E. Send the boy to a child psychologist for counseling F. Suggest that the mother block out some special time each day and give exclusive attention to her son G. Tell the mother that the boy's problem is normal and temporary, and will soon pass if left alone
answer
F This type of enuresis, the result of the defense mechanism of regression, is very common when new siblings enter the household. Regression is characterized by returning to an earlier stage of development. This defense mechanism is also common when people are tired, ill, or uncomfortable. In this case, the boy feels neglected and returns to response patterns more typical of a younger child, either out of anxiety or as a direct attempt to gain more parental attention. Spending more individual time with the boy will help to reassure him of the mother's continued love and affection. With the motive removed, the symptom is likely to dissipate as well.
question
A 23-year-old man with an IQ of 73 lives in a group home. He does not read or write, but he does communicate by one- or two-word utterances. He does not interact with other group-home residents and becomes agitated when others get physically close to him. Everything has a given place in his room and if anything is moved, he becomes disturbed. He repetitively opens and closes doors. Which of the following is the most appropriate therapy for this patient's condition? A. Behavioral therapy B. Electroconvulsive therapy C. Fluoxetine D. Haloperidol E. Methylphenidate
answer
A This man is displaying the classic signs of autism spectrum disorder (ASD), which include withdrawal from interaction with others, failure to use speech for communication, and the obsessive need for sameness. The best treatment for ASD involves behavioral therapy, including techniques such as shaping. Medication (often an as-needed atypical antipsychotic) is generally reserved for ASD patients with disruptive or harmful behavior that are not amenable to more conservative measures. The treatment of ASD is generally most successful when instituted shortly after a diagnosis is made. Shaping is a process in which an instructor teaches a new behavior by systematically and differentially reinforcing "successive approximations" of a target behavior, while no longer reinforcing previous approximations of the behavior.
question
A 3-year-old girl is brought to the physician for a well-child examination. She was born at term after an uncomplicated pregnancy. Physical examination shows no abnormalities. On history, the mother explains that her daughter can identify body parts by pointing to them, play in parallel with other children her age, ride a tricycle, can build a tower of 7-9 blocks, and speak in full sentences that both her parents and strangers can understand. Which of the following is an accurate assessment of this patient's development? A. Delayed gross motor, delayed fine motor, and normal language development B. Delayed gross motor, normal fine motor, and normal language development C. Normal gross motor, delayed fine motor, and normal language development D. Normal gross motor, normal fine motor, and delayed language development E. Normal gross motor, normal fine motor, and normal language development
answer
The correct answer is E. This patient has normal developmental milestones. A 3-year-old child should be able to ride a tricycle, balance on each foot for 1 second, build a tower of 7-9 blocks, copy and draw a vertical line, and speak with almost understandable sentences by both parents and strangers.
question
A 27-year-old man is pleasant, emotionally warm, and happy. He lives with his parents, and interacts well with coworkers at a sheltered workshop where he packages items for shipping. While he was growing up, he never showed any signs of physical disabilities. However, he did not complete high school and he speaks in three-word sentences. Which of the following is most likely associated with this patient's diagnosis? A. Defect in chromosome 15 B. Deficits in early parental attachment C. Difficulty sleeping D. Maternal alcohol use E. Treatment with methylphenidate
answer
D The correct answer is D. This young man is displaying mild-moderate intellectual disability (intellectual developmental disorder) (DSM-5), formerly known as mental retardation (DMS-IV). He is affable, interacts well with others to the point that he can work in a sheltered workshop, could not complete high school, and talks in three-word sentences. The most common known cause of intellectual disability is fetal alcohol syndrome. Fetal alcohol syndrome (FAS) or fetal alcohol spectrum disorder (FASD) is a broad diagnostic term that incorporates a range of physical, mental, behavioral, and cognitive effects that can occur in patients with prenatal alcohol exposure. Functional disabilities seen with FAS include developmental delays, global cognitive or intellectual defects and other CNS-related features.
question
A 65-year-old man with a history of well-controlled hypertension, atrial fibrillation, and type 2 diabetes is brought to the emergency department by his daughter because of decreased responsiveness. The patient's daughter says that he appeared normal 2 hours before arriving in the emergency department. His pulse is 102/min and irregular, and blood pressure is 132/80 mm Hg. The patient is oriented to self but not to time or place. Neurologic examination shows rightward deviation of his gaze and head posture and severe dysarthria with prominent left facial droop. He is able to follow only simple commands such as "squeeze my fingers." Strength is 2/5 in the left upper and lower extremities. Reflexes are preserved throughout, as are flexor plantar responses. CT scan of the head shows no hemorrhage, but MRI of the brain shows ischemic injury to the right cerebral cortex. Laboratory studies reveal his INR is 1.6. Which of the following is the most appropriate pharmacotherapy? A. Alteplase B. Aspirin C. Corticosteroids D. Metoprolol E. Warfarin
answer
The correct answer is A. Ischemic stroke is one of the leading causes of morbidity and mortality in the United States. Approximately 85% of all strokes are ischemic in nature and caused by a vascular occlusion. It is considered a clinical emergency, and timely diagnosis and treatment can improve the patient's outcome. When a stroke is suspected, emergent brain imaging is essential to confirm the diagnosis of ischemic stroke before thrombolytics can be administered. Noncontrast computed tomography (CT) scanning is the most commonly used form of neuroimaging to evaluate a patient suspected of having an acute stroke. In this patient, the ischemic injury most likely occurred as a result of a right middle cerebral artery (MCA) stroke. The affected area must be quite large, judging by the clinical presentation of severe right brain deficits. When a head CT scan reveals no hemorrhage, an acute stroke can be treated by thrombolysis within the first 4.5 hours after the onset of symptoms. Based on the results of the European Cooperative Acute Stroke Study III (ECASS III), the AHA/ASA revised the guidelines in 2010 for the administration of rt-PA after acute stroke, expanding the window of treatment from 3 hours to 4.5 hours. Among the choices given, the only short-acting thrombolytic agent is tissue plasminogen activator (tPA), also known as alteplase. Alteplase is a serine protease that binds to fibrin protein threads of a thrombus and converts the enmeshed plasminogen to plasmin, causing local fibrinolysis.
question
A 58-year-old man is brought to the emergency department because of weakness of the right arm and face and an inability to speak. He has a history of alcohol use, cigarette smoking, and hypertension. Neurologic examination shows an alert mental status. He is only able to say a few words with great effort, is unable to repeat, but can follow simple commands. He has a decreased right nasolabial fold and decreased movements of his right face, sparing the forehead. He has no right arm movement, but is able to raise his right leg. Occlusion of which of the following blood vessels best explains the findings in this patient? A. Left anterior cerebral artery B. Left middle cerebral artery C. Left posterior cerebral artery D. Left posterior inferior cerebellar artery E. Right anterior cerebral artery F. Right middle cerebral artery
answer
The correct answer is B. Because this patient has right-sided motor symptoms, choices E and F can be immediately eliminated because contralateral motor symptoms should result. There is no discussion of visual field defects that would occur with a lesion of the visual cortex, eliminating choice C. Lateral medullary (Wallenberg) syndrome (see below), which occurs with PICA strokes, is not evident, ruling out choice D. This leaves us with choices A and B. His arm is affected and his leg is spared (ruling out choice A); he also has Broca's aphasia. This indicates a stroke of the lateral language-dominant cortex, which is supplied by the left middle cerebral artery (choice B). We have quickly arrived at the correct answer, but let's review the neuroanatomy in more detail. This patient most likely had a stroke of the left middle cerebral artery. This artery supplies the lateral surface of the frontal, parietal, and upper temporal lobes; the internal capsule; and most of the basal ganglia. The MCA is the vessel that is most commonly affected by cerebrovascular accidents.
question
A 41-year-old man is brought to the emergency department because of a sudden, painful headache. He describes the pain as a 10/10 and denies history of cluster or migraine headaches. He tried ibuprofen, but it did not help. The patient had significant nausea and vomiting with the event, and he has since developed some neck stiffness. Physical examination reveals scattered bruises and scarring. The patient reports a history of repeated joint dislocation and recently progressing arthritis. Which of the following conditions has a potential association with this patient's illness? A. Carotid artery disease B. Ehlers-Danlos syndrome C. Polymyalgia rheumatica D. Trauma E. Visual auras
answer
The correct answer is B. The patient's presentation of sudden onset, severe headache ("worst headache of my life") with nausea, vomiting, and meningeal signs (neck stiffness) is consistent with subarachnoid hemorrhage. Meningeal signs result from irritation of meninges when blood enters the subarachnoid space. Several days after a subarachnoid hemorrhage, focal neurologic signs may develop as a result of cerebral vessel vasospasm, induced by blood break-down products in the subarachnoid space. A major cause of subarachnoid hemorrhage is rupture of cerebral berry (saccular) aneurysms. These aneurysms form at sites of cerebral vessel weakness (typically in the anterior cerebral artery at the junction of the anterior communicating artery), where there can be a thin tunica media and absence of an external elastic lamina. Berry aneurysms can potentially be associated with a number of connective tissue disorders include Marfan syndrome and Ehler-Danlos syndrome, due to weakened connective tissue in arterial walls. Hypertension and adult polycystic kidney disease can also produce berry aneurysms. Our patient's physical exam findings are consistent with Ehler-Danlos syndrome, a hereditary collagen disorder characterized by hyperflexible joints, skin hyperextensibility, bruising, early onset arthritis, and ascending aorta aneurysms.
question
A 45-year-old man is brought to the emergency department because of the sudden onset of severe headache associated with nausea and vomiting. He has a 10-year history of hypertension, but has not taken antihypertensive medications or seen a physician in many years. His mother and uncle both had kidney problems and required dialysis. Physical examination shows mild nuchal rigidity, normal heart and lung sounds, and a normal neurologic examination. Abdominal examination reveals bilateral abdominal masses. Laboratory studies show: Leukocyte count 8,000/mm3 Erythrocyte count 3.8 million/mm3 Mean corpuscular volume 90 μm3 Serum creatinine 3.9 mg/dL Serum urea nitrogen 38 mg/dL Urinalysis shows 2+ blood, negative for protein or casts. A CT scan of the head shows an intracranial hemorrhage. Lumbar puncture reveals cells with hemosiderin. Which of the following is the most likely source of the intracranial bleeding? A. AV malformation B. Bridging veins C. Charcot-Bouchard aneurysm D. Circle of Willis E. Middle meningeal artery
answer
The correct answer is D. Adult polycystic kidney disease is an autosomal dominant genetic disorder characterized by the presence of bilaterally enlarged kidneys with multiple cysts and presentation of hypertension, renal failure, and hematuria during middle-age. Two relationships are useful in solving this problem: (1) Adult polycystic kidney disease has a specific association with berry aneurysms involving the circle of Willis and its branches, and (2) spontaneous subarachnoid hemorrhages, which are most often the result of bleeding from berry aneurysms. The berry aneurysms develop at sites of congenital weakness (near branch points) of the relatively unsupported vessels of the circle of Willis. Subarachnoid hemorrhage can also be associated with Marfan syndrome, Ehlers-Danlos syndrome, hypertension, and smoking. In this case, our patient has persistent hypertension, bilateral palpable abdominal masses, and a family history of renal disorder requiring dialysis, which should point one towards polycystic kidney disease. The lab results show the presence of anemia and hematuria, indicating a chronic bleed from the kidneys. The elevated serum creatinine and urea nitrogen indicates renal failure, where the BUN/creatinine ratio is less than 10:1, indicating an intrarenal etiology.
question
A 45-year-old man comes to the emergency department because of sudden onset, severe headache. He has also developed significant nausea, vomiting, and photophobia since the initial onset of symptoms. His past medical history is significant for hypertension and flank mass. His father's side of the family has a history of hematuria and renal failure. A magnetic resonance angiogram (MRA) of the cerebral vessels is shown with abnormalities in the vessel identified by the yellow arrows. Which of the following arteries is most likely responsible for this patient's pathology? A. Anterior cerebral artery B. Basilar artery C. Middle cerebral artery D. Ophthalmic artery E. Posterior cerebral artery F. Vertebral artery
answer
The correct answer is A. This patient presents with the classic symptoms of subarachnoid hemorrhage, including sudden, "thunderclap" headache ("worst headache of my life"), nausea, vomiting, photophobia, and meningeal signs (due to irritation of meninges by blood in the CSF). The hemorrhage likely resulted from rupture of a berry aneurysm of the intracranial cavity. It develops as a saccular expansion of the arterial wall and most commonly affects the anterior part of the circle of Willis, including the anterior cerebral artery (ACA) close to where the anterior communicating artery branches off. This particular region is vulnerable to saccular expansion due to a thinner tunica media and absence of external elastic lamina. The patient's past medical and family history also points towards ruptured berry aneurysm as the source of his pathology. The patient most likely has autosomal dominant polycystic kidney disease (ADPKD) due to his history of hypertension and flank mass (kidney cysts). In late stage disease, patients can develop renal failure. These patients have a significantly elevated risk of developing of berry aneurysms (due to a combination of chronic hypertension and cerebral arterial wall defects).
question
A 55-year-old man with a history of uncontrolled hypertension is brought to the emergency department because of right-sided hemiplegia. His wife states that he has smoked 1 pack of cigarettes daily for the past 35 years and he consumes 3-4 beers daily. He appears to be in some distress and has difficulty speaking. Physical examination shows paralysis of the right side of the mouth, and the right upper and lower extremities. There is no loss of sensation noted on any of his extremities and he is able to wrinkle his forehead on both sides. The lesion responsible for this patient's symptoms is most likely located in which of the following? A. Corticospinal tract in the upper cervical spinal cord B. Left internal capsule C. Medial portion of the left frontal lobe D. Medial portion of the right frontal lobe E. Right internal capsule
answer
The correct answer is B. This patient has sustained a hemorrhage of the left internal capsule secondary to uncontrolled hypertension. The internal capsule is an area of white matter in the brain that separates the caudate nucleus and the thalamus from the lenticular nucleus (see figure). The internal capsule receives its blood supply from the lenticulostriate arteries which, in uncontrolled hypertension, are prone to develop a lipo-hyaline arteriosclerosis (Charcot-Bouchard aneurysms) resulting in ischemic changes of the surrounding tissue (lacunar infarcts) and corresponding neurologic deficits of the affected area. Weakening of the affected blood vessel wall can occur and cause hemorrhage, resulting in a hemorrhagic stroke and increased mortality risk. Lacunar infarcts account for approximately 20-25% of ischemic strokes, while 35-45% of intracerebral hemorrhages involve the basal ganglia. Moreover, hypertension is associated with a majority of all intracerebral hemorrhages and lacunar infarcts. There are 3 parts of the internal capsule: The anterior limb of the internal capsule conveys frontopontine fibers. The genu conducts corticobulbar fibers Posterior limb of the internal capsule conveys corticospinal fibers to the contralateral arm and leg.
question
A 65-year-old man with a history of hypertension and diabetes is brought to the emergency department because of dysarthria and the inability to move his left arm and leg. His wife states that he has smoked 1 pack of cigarettes daily for the past 35 years and he consumes 3-4 beers daily. He appears to be in some distress and has difficulty speaking. Physical examination shows an inability to move his left arm and leg, hyperreflexia on the left, and a upgoing plantar reflex on the left. He also has left facial weakness, sparing his forehead. Sensation in his face and extremities is intact. The lesion responsible for this patient's symptoms is most likely located in which of the following areas? A. Corticospinal tract in left upper cervical cord B. Corticospinal tract in right upper cervical cord C. Left internal capsule D. Left lateral frontal lobe E. Left pons F. Left thalamus G. Right internal capsule H. Right lateral frontal lobe I. Right pons J. Right thalamus
answer
The correct answer is G. Don't be taken aback by the number of options. The correct answer can be arrived at quickly with a combination of eliminating incorrect answers and narrowing down potentially correct answers. First, let's note his symptoms: Left-sided hemiplegia Left Babinski sign Left lower facial paralysis So, it's pure motor and all on the left. We need to think of something that can only cause left-sided motor symptoms. We also know that because he has a facial paralysis in addition to paralysis of the body, the lesion must be in the brain (eliminating choices A and B, which are in the spinal cord). The tract that must be affected is the corticospinal tract because he is showing upper motor neuron signs (paralysis, hyperreflexia, Babinski sign). Because we know the lesion is in the brain, the lesion must be contralateral to his signs and symptoms. In the figure below, we can see that the lesion must be above the decussation (crossing) of the corticospinal tract—so it must be on the right side (because he has left-sided symptoms). We can now eliminate every option on the left (choices A, C, D, E, F). Because our patient has only motor deficits, we can also eliminate the thalamus (choice F and J). The thalamus is the sensory relay center of the brain. A thalamic lesion would not cause motor deficits. Instead, a lesion of the thalamus could lead to a contralateral sensory loss. The lateral frontal lobes (choices D and H) can also be eliminated. The primary motor cortex is in the frontal lobes, however, the lateral portion controls the arm and face, where as the medial portion controls the legs. This patient has paralysis of both arms and legs. The motor homunculus located in the precentral gyrus of the frontal lobe is shown below. (Furthermore, because this patient has a left-sided hemiplegia, if cortex was involved, it would have to be the right cortex.)
question
A 65-year-old man with a history of uncontrolled hypertension is brought to the emergency department after new onset weakness in his right leg. He denies any recent head trauma but does say that he has had a recent history of episodic palpitations and light headedness. Physical examination reveals 1/5 strength and decreased pinprick and light touch sensation in his right lower extremity. The patient is able to speak normally. Physical examination reveals no evidence of papilledema and pupils are equal, round, and reactive to light and accommodation. Occlusion of which of the following blood vessels is most likely responsible for this patient's symptoms? A. Left anterior cerebral artery B. Left middle cerebral artery C. Left posterior cerebral artery D. Right anterior cerebral artery E. Right middle cerebral artery F. Right posterior cerebral artery
answer
The correct answer is A. This patient likely had atrial fibrillation, leading to an embolic stroke. Hypertension is a common predisposing factor for atrial fibrillation. This patient has decreased motor function and sensation in his right leg. The medial aspect of the cerebral hemispheres is supplied by the anterior cerebral arteries. The small portion of primary motor cortex located here sends fibers to spinal cord segments innervating the lower limbs. Since the corticospinal tracts cross over to the opposite side in the caudal medulla, the left cerebral hemisphere controls the right side of the body. Hence, paralysis of the right leg results from loss of blood flow to the portion of the left hemisphere supplied by the left anterior cerebral artery.
question
A 78-year-old woman with a history of atrial fibrillation is brought to the emergency department after falling down a flight of stairs. Current medications include warfarin. She appears grossly lethargic and is responsive only to painful stimuli. Her blood pressure is 187/99 mm Hg, and pulse is 45/min. A noncontrast CT scan is shown. Which of the following best explains the hemodynamic abnormalities in this patient? A. An autonomic response triggered by increased intracranial pressure B. Autonomic dysfunction secondary to brainstem injury during herniation C. Essential hypertension D. Malignant hypertension E. Stress-induced catecholamine release
answer
The correct answer is A. The patient has hypertension and bradycardia in response to increased intracranial pressure (ICP) and brainstem ischemia. Clinical manifestations of increased ICP may also include respiratory depression, a response known as the Cushing reflex or Cushing response. Cerebral blood flow (CBF) is a function of the pressure drop across the cerebral vasculature divided by vascular resistance (CBF = ΔP / R), as in other vascular beds. Unlike other vascular beds, however, cerebral flow is also susceptible to changes in ICP. Because the tissues of the brain are incompressible and because both brain and vasculature are contained within a rigid cranium, increases in ICP reduce cerebral perfusion pressure (CCP): CCP = CAP - ICP where CAP = carotid arterial pressure. ICP is normally low (≤ 15mm Hg). The cerebral vasculature compensates for small increases in ICP through autoregulation (i.e., reflex dilation of resistance vessels) and by displacing cerebrospinal fluid (CSF). Large changes in ICP (e.g., due to tumor growth, hematoma, edema, increased CSF volume) can decrease CPP to the point where ischemic injury occurs. This process begins with the watershed areas, which are located between the territories supplied by two different large vessels (i.e., anterior cerebral and middle cerebral arteries). The Cushing response reflects brainstem compression and ischemia and is often a sign of impending herniation. The brainstem houses autonomic cardiovascular and respiratory control centers; the symptom triad (Cushing's triad: hypertension, bradycardia, and respiratory depression) is a manifestation of severe ischemia and control-center dysfunction.
question
A 32-year-old woman comes to the physician because of headaches and a seizure. She states that she delivered a full-term girl two weeks ago. Physical examination shows bilateral papilledema. An MRI of the brain in a normal individual is shown. It's at the occipital lobe. The arrow in the normal scan indicates where a thrombus formed in this patient. The most likely cause of this patient's symptoms is occlusion of which of the following structures? A. Anterior cerebral artery B. Cavernous sinus C. Middle cerebral artery D. Middle meningeal artery E. Posterior cerebral artery F. Superior sagittal sinus
answer
The correct answer is F. This patient has a sagittal sinus thrombosis, which is often associated with hypercoagulable states. It occurs with increased frequency during pregnancy, and during the first few weeks postpartum. Obstruction of the venous drainage can lead to increased intracranial pressure, seizures, parasagittal hemorrhages (due to back-pressure in the cortical veins), and infarcts from decreased cerebral perfusion (due to increased venous pressure). It is an uncommon cause of cerebral infarction compared to arterial disease, but has significant morbidity. Treatment usually involves anticoagulation therapy or surgical intervention is also performed.
question
A 68-year-old woman comes to the physician because of a worsening tremor that is now affecting her ability to brush her teeth and perform other activities of daily living. For several years, the patient has had a tremor at rest that originally started on the left side but now occurs on the right side as well. Neurologic examination shows a bilateral resting tremor, mask-like facies, and a slow, shuffling gait. She is currently taking levodopa and carbidopa. Due to the patient's worsening symptoms, trihexyphenidyl is added to her drug regimen. Which of the following side effects will this patient most likely experience? A. Diaphoresis B. Diarrhea C. Dry mouth D. Miosis E. Urinary incontinence
answer
The correct answer is C. Trihexyphenidyl and benztropine are muscarinic antagonists that are used as adjunctive therapy in Parkinson disease (PD). Antimuscarinics are most useful in treating tremor, and are less efficacious in treating bradykinesia and rigidity. It is thought that motor symptoms of PD result from an imbalance of dopamine and acetylcholine in the striatum, and therefore treatment is directed at increasing dopaminergic tone and decreasing cholinergic tone. Trihexyphenidyl also blocks muscarinic receptors in the autonomic nervous system.
question
A 65-year-old man comes to the physician because of a persistent tremor that decreases during voluntary activity. Vital signs are unremarkable. He has a 3-year history of bradykinesia, rigidity, gait difficulty, and tremor of his right arm. Treatment with levodopa plus carbidopa in addition to bromocriptine has provided significant benefit. However, the patient says the tremor in his right dominant arm is worsening and is now occurring in his right leg. The physician decides to add trihexyphenidyl to his treatment regimen and provides information to the patient about side effects that could occur. The adverse effects of trihexyphenidyl are most similar to those of which of the following drugs? A. Amitriptyline B. Fluoxetine C. Lorazepam D. Pilocarpine E. Prazosin F. Verapamil
answer
The correct answer is A. This patient has Parkinson disease and has the classic symptoms of bradykinesia, a resting tremor, gait difficulty, and rigidity. This disease is caused by the degeneration of nigrostriatal dopamine neurons, leading to decreased levels of dopamine in the striatum. This leads to an imbalance of dopamine (DA) and acetylcholine (ACh) in the striatum (DA is too low and ACh is too high). As a result, increasing dopaminergic tone (primarily) and decreasing cholinergic tone is the goal of pharmacotherapy. Increasing dopaminergic tone can be achieved by levodopa/carbidopa (dopamine precursor and peripheral dopa decarboxylase inhibitor), dopamine agonists (e.g., bromocriptine, pramipexole, ropinirole, apomorphine), MAO B inhibitors (e.g., selegiline, rasagiline), COMT inhibitors (tolcapone, entacapone), and amantadine (increases DA release). Decreasing cholinergic tone by using muscarinic antagonists (trihexyphenidyl, benztropine) provides the most benefit for tremor, but is less effective for treating bradykinesia or rigidity. Antimuscarinics can be used as monotherapy for patients who have primarily have a tremor or as part of combination therapy for those whose tremor is not adequately controlled by agents such as levodopa and dopamine agonists. Trihexyphenidyl has side effects common to other drugs that also block muscarinic receptors. Another drug with well-known antimuscarinics side effects is amitriptyline, a tricyclic antidepressant.
question
A 72-year-old man is brought to the physician for a follow-up examination by his wife. For the past year, his wife has noticed that when the patient is reading the newspaper, he continually moves his fingers and hands. At first, the movements were only on the right side, but they have since spread to the left side too. His movements subside when he performs acts such as picking up a glass of water. To view the patient, click on the "Play Media" button. Which of the following drugs acts by inhibiting the metabolism of the neurotransmitter that is deficient in this patient? Media shows tremor at rest, mask-like face. A. Benztropine B. Bromocriptine C. Levodopa D. Ropinirole E. Selegiline
answer
The correct answer is E. The patient presentation is classic for Parkinson disease (PD), and all of the answer choices are drugs used in the treatment of this disorder. MAO-B inhibitors such as selegiline (deprenyl) and rasagiline inhibit monoamine oxidase B (MAO-B). MAO-B preferentially metabolizes dopamine, whereas MAO-A preferentially metabolizes norepinephrine and serotonin. These agents slow the breakdown of dopamine. They are often used as monotherapy for early symptomatic treatment of PD because of better adverse effect profiles and they may improve long-term outcomes.
question
A 50-year-old woman comes to the physician because of rhythmic, involuntary shaking of both hands that has been gradually worsening over the last 4 years. She states that the shaking is worse when she is nervous or upset. Physical examination shows a moderate amplitude, 8-Hz tremor of the upper extremities that particularly involves flexion and extension at the wrists and fingers. The tremor is mildly accentuated when her arms are outstretched and when she performs the finger-nose-finger maneuver. There is also a very low amplitude, similar-frequency tremor of the head and voice. There are no other abnormalities. Which of the following is the most appropriate pharmacotherapy for this patient? A. Agonist at dopaminergic receptors B. Agonist at glutamatergic receptors C. Antagonist of beta-adrenergic receptors D. Antagonist of GABA-A receptors E. Antagonist of muscarinic cholinergic receptors
answer
The correct answer is C. Tremors can be characterized as static or kinetic tremors. Static tremors include the resting tremor (occurs at rest; common in Parkinson disease) and postural tremors (occur with head and limbs while held at a fixed posture). Kinetic tremors are action tremors (remain unchanged during voluntary movement) and intention tremors (increase during a goal directed movement). Essential tremor (ET), which our patient has, is the most common neurologic disorder that causes progressive postural and/or action tremors, affecting up to 5% of the population worldwide. Approximately 50% of cases have an autosomal dominant pattern of inheritance; those cases are referred to as familial tremor. ET usually affects both upper extremities and the head and voice may also be affected. ET is not typically associated with other neurologic symptoms. Essential tremor usually worsens with stress, and improves with relaxation and alcohol consumption. Our patient is exhibiting postural and action tremors. Typical tremors are seen in the upper extremities and are 4- 10 Hz flexion-extension in nature. She also has involvement of her head and voice. She is not exhibiting additional neurologic symptoms. Propranolol , a nonselective beta-adrenergic receptor antagonist, and primidone, a barbiturate anticonvulsant, are effective first-line therapies. Second-line therapies for this condition include gabapentin, topiramate, and alprazolam.
question
A 72-year-old woman is found lying in her nursing home bed unresponsive. Two hours before, she was in good condition, although she has a history of Alzheimer disease. There is no evidence of trauma, and the nursing home staff assure the physician that she did not fall. Her temperature is 37.5°C (99.5°F), pulse is 64/min, respirations are 15/min and irregular, and blood pressure is 154/92 mm Hg. Physical examination is limited owing to patient's altered mental state. No evidence of trauma is visible on external examination. A CT scan of the head is shown. Which of the following is the most likely cause of the hemorrhage? A. Amyloid angiopathy B. Epidural hematoma C. Rupture of berry aneurysm D. Rupture of Charcot-Bouchard aneurysm E. Subdural hematoma
answer
The correct answer is A. Alzheimer disease patients are prone to large "lobar" hemorrhages that are usually centered in the parietal lobe (thus the name "lobar") and may spread to totally destroy an entire cerebral hemisphere, resulting in death. This is due to amyloid deposition into the walls of cerebral blood vessels (amyloid angiopathy), similar to the amyloid plaques seen in the parenchyma of the brain with this disease. Amyloid makes these vessels weak and prone to rupture.
question
A 20-year-old man is brought to the emergency department because of sudden onset severe headache and significant nausea and vomiting. His past medical history is remarkable for recurrent epistaxis during childhood, episodic seizures, melena, and iron deficiency anemia. He denies history of headaches. Physical examination shows multiple red lesions across the patient's face and palate. Since arriving at the emergency department, the patient has also developed significant neck stiffness. Laboratory studies show: Leukocyte count 5,000/mm3 Erythrocyte count3 6 million/mm3 Hematocrit 65% Platelets 475 x103/mm3 Prothrombin time 12 seconds Activated partial thromboplastin time 35 seconds Which of the following is the most likely cause of this patient's most recent symptoms? A. Arteriovenous malformation B. Berry aneurysm C. Bridging vein D. Charcot-Bouchard aneurysm E. Middle meningeal artery
answer
The correct answer is A. The patient presents to the emergency department with the classic symptoms of a subarachnoid hemorrhage. These include sudden onset severe headache ("worst headache of my life"), nausea, vomiting, and meningeal signs such as neck stiffness (due to irritation of meninges from blood in the subarachnoid space). CT scan may or may not show blood in the intraventricular space. There are several potential causes of subarachnoid hemorrhage including saccular (berry) aneurysm rupture, and in the case of this patient, bleeding from arteriovenous malformations (AVMs) in the brain. To distinguish between berry aneurysm and arteriovenous malformation, it is important that you look at other elements of this patient's history. His history points towards a diagnosis of hereditary hemorrhagic telangiectasia (HHT), also known as Osler-Weber-Rendu syndrome. Key findings include a history of recurrent epistaxis (due to telangiectasias in nasal mucosa), melena (due to GI telangiectasias), and multiple cutaneous and mucosal vascular lesions. These patients also have a propensity to develop large AVMs in the brain, lungs, and liver, and cerebral AVMs can produce seizures.
question
A 45-year-old man with a history of untreated hypercholesterolemia and hypertension is brought to the emergency department after collapsing at home. He never regains consciousness, and he dies 8 hours later. An autopsy shows a massive intracerebral hemorrhage filling the ventricles. Which of the following is most likely to be identified as the location of the bleeding? A. Basal ganglia and internal capsule B. Central white matter C. Cerebellum and medulla D. Pons E. Thalamus and hypothalamus
answer
The correct answer is A. Intraparenchymal hemorrhage is typically due to hypertension, and is the most common cause of death from stroke. Such hemorrhages can occur at all of the sites listed in the answer choices; however, roughly half of such cases involve the basal ganglia (putamen is #1 site) and internal capsule.
question
A 44-year-old man with a life-history of intellectual disability dies from complications of congenital heart disease. Prior physical examinations were notable for a flat nasal bridge, single palmar crease, and macroglossia. Also noted was a holosystolic murmur located at the apex. Autopsy shows the presence of a 3-cm ventricular septal defect and numerous senile plaques and neurofibrillary tangles in the cerebral cortex and hippocampus. There is also evidence of prior hemorrhage of small cerebral vessels. These changes are most likely associated with the presence of an extra copy of the gene encoding which of the following? A. Amyloid precursor protein B. Apolipoprotein E C. Presenilin-1 D. Presenilin-2 E. Superoxide dismutase
answer
The correct answer is A. Neurofibrillary tangles and senile plaques are the neuropathologic hallmarks of Alzheimer disease (AD), which develops in the great majority of Down syndrome patients who survive beyond 40 years. The presence of a triplicate copy of the amyloid precursor protein (APP) gene on chromosome 21 (trisomy 21) is thought to be the reason for the association between Down syndrome and AD. Alzheimer-associated amyloid Aβ derives from proteolytic cleavage of APP (a transmembrane protein expressed on the cell surface) and accumulates within the core of senile plaques. Thus the extra copy of APP in Down syndrome patients predisposes them to greater accumulation of beta amyloid. The evidence of prior hemorrhages in this patient is related to amyloid angiopathy, where accumulation of amyloid can weaken vessel walls and produce spontaneous hemorrhages.
question
An 80-year-old woman dies after a long history of progressive memory loss, apraxia, and recurrent episodes of confusion. In the last months of life, she was bedridden and unable to recognize familiar faces and objects. Microscopic examination of brain tissue obtained at autopsy shows numerous flame-shaped intracytoplasmic inclusions in neurons of the neocortex and hippocampus. Electron microscopy shows that the inclusions consist of paired helical filaments. Which of the following is most likely involved in the development of the intractyoplasmic aggregates in this condition? A. Abnormal degradation of amyloid precursor protein B. Abnormal phosphorylation of tau C. Accumulation of advanced glycosylation end products D. Accumulation of PrPsc E. GAA repeats in a gene encoding a mitochondrial protein F. Mutation in zinc-copper superoxide dismutase
answer
The correct answer is B. This patient has all of the characteristic clinical features of dementia (memory loss, apraxia, and confusion). Alzheimer disease (AD) is the most common cause of dementia in the elderly. Gross findings in AD include cortical atrophy, widening of cerebral sulci, ventricular enlargement (hydrocephalus ex vacuo, due to loss of brain tissue), and atrophy of the medial temporal lobe (hippocampus, entorhinal cortex, amygdala). The major microscopic findings include neurofibrillary tangles (NFTs), which are intracellular and neuritic (senile) plaques, which are extracellular plaques of amyloid.
question
A 59-year-old Caucasian man who is a farmer is admitted to the hospital because of gradually worsening right-sided weakness and sensory deficits over a period of months. MRI of the brain shows multiple well-circumscribed masses located the gray-white matter junction. A biopsy specimen of one of these lesions is shown. Immunohistochemical studies show the atypical cells stain positive for S100 and HMB45. Which of the following is the most likely underlying cause of these findings? A. Glioblastoma multiforme B. Intracerebral hemorrhages C. Metastatic colon carcinoma D. Metastatic melanoma E. Metastatic small cell carcinoma
answer
The correct answer is D. This patient presents with metastatic melanoma, a neuroectodemal tumor derived from skin melanocytes, with resulting intracerebral lesions. There are three important clues pointing towards this answer. Multiple masses located at the gray-white matter junction is highly suggestive of metastatic disease. The gray-white junction is not only highly vascular, but many of these vessels are of narrow caliber, thus metastatic cells can easily lodge in these spaces. The patient is Caucasian and works in an occupation involving significant sun exposure. UV light exposure and fair skin are major risk factors for many skin cancers, particularly melanoma. Other risk factors for melanoma include a weakened immune system and family history of melanoma. Finally, the biopsy specimen demonstrates a malignant neoplasm with multifocal deposits of brown pigment (melanin), pointing towards melanoma as the diagnosis.
question
An 82-year-old woman with chronic dementia dies suddenly of an apparent myocardial infarction. Prior to death, a neurologic examination showed no abnormalities except for deficits in cognition and memory. At autopsy, the brain shows diffuse cortical atrophy with relative sparing of primary motor and sensory areas. Which of the following findings is most likely on microscopic examination of tissue from the brain? A. Central chromatolysis B. Gliosis of the caudate nucleus C. Lewy bodies D. Loss of pigmented neurons E. Neurofibrillary tangles
answer
The correct answer is E. The clinical history and gross pathology suggest that this woman suffered from Alzheimer disease. Common clinical features of Alzheimer disease include initial memory loss and problems with executive function. Later Alzheimer's can present with features of other types of dementias such as hallucinations and urinary incontinence. In Alzheimer disease, neurofibrillary tangles (NFTs) and senile plaques (associated with amyloid deposition) are found primarily in higher order association cortex i.e., frontal and temporal lobes.
question
An autopsy is performed on a 60-year-old man who develops visual hallucinations, fluctuating dementia, and eventual parkinsonism. Histopathologic examination shows numerous eosinophilic intracytoplasmic inclusions within neurons of the substantia nigra, limbic cortex, and basal nucleus of Meynert. The inclusions are immunoreactive for ubiquitin. Which of the following is the most likely diagnosis? A. Alzheimer disease B. Amyotrophic lateral sclerosis (ALS) C. Diffuse Lewy body disease D. Parkinson disease E. Pick disease
answer
The correct answer is C. This question focuses on neurodegenerative and dementing illnesses. The correct answer can be deduced by combining clinical and pathologic features. Diffuse Lewy body disease (dementia with Lewy bodies) is characterized by widespread formation of Lewy bodies (intracytoplasmic eosinophilic inclusions), containing alpha-synuclein or ubiquitin within neurons of the substantia nigra (producing parkinsonism), cholinergic nuclei such as the basal nucleus of Meynert (producing dementia), and limbic cortex (which may explain the hallucinations). Once believed to be infrequent, recent clinicopathologic investigations have suggested that this might be one of the most frequent causes of dementia in industrialized countries, perhaps second only to Alzheimer disease. Peculiar to this dementing disorder is the presence of extrapyramidal symptoms and visual hallucinations, together with an undulating clinical course.
question
A 65-year-old man becomes forgetful and depressed. His movements are slow, and he develops a tremor. His condition progresses over the next 6 months, then he falls down the stairs, fractures his neck, and dies. An MRI of the man's brain prior to death is shown. Which of the following is most likely to be found on microscopic examination of tissue from the area indicated by the arrow? A. Granulovacuolar degeneration B. Hirano bodies C. Lewy bodies D. Microglial nodules E. Negri bodies F. Neurofibrillary tangles G. Pick bodies H. Spongiform change
answer
The correct answer is C. This patient has Parkinson disease. The labeled area of the image is the substantia nigra. The substantia nigra is located in the midbrain, separating the crus cerebri and the red nucleus. Although the patient account does not clearly describe the classic triad of Parkinson disease (namely, tremors, rigidity, and akinesia), both mood and cognitive disturbances are commonly associated with this disorder. The histopathologic hallmarks of Parkinson disease are Lewy bodies: round, eosinophilic, intracytoplasmic inclusions seen in dopaminergic nigrostriatal neurons. Lewy bodies can also be found in Lewy body dementia, but this disease features prominent visual hallucinations and early dementia, as well as some parkinsonian symptoms.
question
A 65-year-old woman is brought to the physician by her daughter. The patient has a 5-year history of personality changes, neglect of personal hygiene, impaired judgment, and disinhibited behavior. The patient's daughter says that her mother has had more trouble remembering things during the last year. An MRI scan of the head is shown. Anterior temporal lobe atrophy is apparent in other sections but no evidence of infarcts is seen. Which of the following is the most likely diagnosis? A. Alzheimer disease B. Creutzfeldt-Jakob disease C. Lewy body dementia D. Pick disease E. Vascular dementia
answer
The correct answer is D. This patient most likely has Pick disease, also known as frontotemporal dementia (FTD). The mean age of presentation is between 55 and 60 years. There are two variants of FTD, the most common of which is the behavioral variant, characterized by progressive changes in behavior and personality, followed later in the disease course by cognitive impairments. There is also a primary progressive aphasia variant, which begins with speech and language abnormalities.
question
A 50-year-old woman comes to the physician because of progressive weakness in her legs. She first noticed these symptoms about a year ago. CT scan of the head shows a well-circumscribed 3.5 cm uniformly enhancing mass present in the parasagittal region, compressing the underlying brain parenchyma without infiltration. She undergoes resection of the intracranial mass. The surgical specimen is shown in the photograph. Which of the following is the most likely underlying condition? A. Arteriovenous malformation B. Glioblastoma multiforme C. Medulloblastoma D. Meningioma E. Oligodendroglioma
answer
The correct answer is D. The gross features of our patient's tumor are consistent with meningioma, the most frequent benign intracranial neoplasm, arising in older adults, females greater than males. They arise from arachnoidal cells of the meninges (meningothelial cells). Grossly, the tumor is well-circumscribed and anchored on the dura mater (as shown in this image as the thin pale grey membranous tissue). Histologically, meningiomas consist of whorls of elongated, spindle-shaped cells with scattered psammoma bodies (laminated calcified concretions). These tumors are usually located near venous sinuses (particularly in parasagittal regions) due to the high concentration of arachnoid cells in these areas. The benign behavior of this tumor is apparent from its "pushing" pattern of growth. The tumor tends to grow slowly, expand downward, compressing the brain and producing neurologic symptoms, but without invading it. For this reason, this tumor can be easily removed at surgery.
question
A 57-year-old man is brought to the emergency department after experiencing a seizure. He has no history of prior seizures. An MRI of the head is shown. There is a well circumscribed lesion. He is treated with both chemotherapy and radiation, but his condition continues to progress. Despite therapy, the patient dies 11 months later. Examination of the brain at autopsy shows a large tumor extending from the right to the left cerebral hemisphere, across the corpus callosum. Which of the following features is most strongly associated with this patient's tumor? A. Derivation from primitive neuroectodermal cells B. Hydrocephalus C. Location along parasagittal dura D. Location in the posterior fossa E. Pseudopalisading necrosis
answer
The correct answer is E. Glioblastoma multiforme (GBM) is the most frequent primary malignant tumor of the CNS. It usually affects middle-aged or elderly patients, and most frequently arises in the cerebral white matter. GBM is a malignant astrocytoma; it is referred to as grade IV astrocytoma in the WHO classification. These tumors often spread along white-matter tracts, especially the corpus callosum, resulting in the characteristic "butterfly glioma" described in this question. Pathologic examination often reveals areas of necrosis surrounded by rows of neoplastic cells, a phenomenon known as pseudopalisading necrosis. This tumor is very aggressive; median survival is approximately 12 to 14 months following optimal treatment, i.e., a combination of surgery and radiation therapy.
question
An 8-year-old girl with no significant past medical history is brought to the emergency department because of a 2-day history of intractable vomiting and headache. Her mother says that she was in a normal state of health prior to the onset of these symptoms. The patient denies abdominal pain, diarrhea, sore throat, dysuria, rash, visual changes, or neck pain. Neurologic examination shows papilledema and an unsteady gait. An axial T2-weighted MRI is shown. There is large mass in the cerebellum. Histologic examination of the tumor shows elongated astrocytes with long bipolar processes and numerous eosinophilic intracellular inclusions with a twisted appearance. Which of the following is the most likely diagnosis? A. Ependymoma B. Fibrillary astrocytoma C. Glioblastoma multiforme D. Medulloblastoma E. Pilocytic astrocytoma
answer
The correct answer is E. The posterior fossa is the most common location of primary brain tumors in children. The most common cerebellar tumors in children are pilocytic astrocytoma and medulloblastoma. Juvenile pilocytic astrocytomas are benign, well-circumscribed masses that are classically described as a cyst with a mural nodule (as seen in this patient's MRI). They often cause symptoms of ataxia and clumsiness of the limbs. As the tumors grow, they can cause symptoms of increased intracranial pressure and hydrocephalus. As seen in this patient, symptoms are reflective of mass effect from the increased intracranial pressure and include vomiting, nausea, and altered mental status.
question
A 62-year-old woman is brought to the emergency department by her daughter because of a seizure. Her daughter states that she has complained of headaches over the last couple of weeks and she has episodes of confusion. A MRI of the head with contrast shows a large, left temporal lobe mass with subfalcine herniation. She undergoes surgical resection and a frozen section shows areas of necrosis surrounded by bizarre pleomorphic tumor cells accompanied by necrosis and hemorrhage. Which of the following is the most likely diagnosis? A. Craniopharyngioma B. Ependymoma C. Fibrillary astrocytoma D. Glioblastoma multiforme E. Oligodendroglioma F. Pilocytic astrocytoma
answer
The correct answer is D. Glioblastoma multiforme (GBM) is the most aggressive tumor in the astrocytic line and is the most common CNS primary malignancy in adults (typical age is over 50 years of age). It can also occur in children, often in the brainstem. GBM tends to be a tumor of the cerebral cortex in adults. MRI scan with contrast reveals a ring-enhancing lesion (contrast with central necrosis). Histologically, the tumor tends to show necrosis and hemorrhage. The tumor cells are pleomorphic, and frequently are bizarre in appearance with markedly enlarged, abnormal nuclei. The characteristic histopathologic feature (as shown in the figure below) is an area of necrosis surrounded by rows of neoplastic cells (pseudopalisading necrosis).
question
A 7-year-old child is brought to the physician because of nausea, vomiting, and difficulty walking. His temperature is 37.6°C (99.7°F), pulse is 80/min, respirations are 16/min, and blood pressure is 120/70 mm Hg. Physical examination shows a wide-based, ataxic gait. CT scan of the head shows a mass in the cerebellar vermis. Microscopic examination of a biopsy specimen shows evidence of both neuronal and glial differentiation and occasional rosette formation. Which of the following is the most likely underlying condition? A. Ependymoma B. Glioblastoma multiforme C. Medulloblastoma D. Meningioma E. Oligodendroglioma
answer
The correct answer is C. Medulloblastoma is the only tumor on the list characterized by both neuronal and glial differentiation. It is a primitive neuroectodermal tumor that is highly undifferentiated and originates from a primordial neuroglial precursor. It is predominantly a pediatric tumor; 75% occur in children. Medulloblastomas arise in the cerebellar vermis (infra-tentorial location), which accounts for this patient's truncal ataxia. Adult medulloblastomas are not common and more likely to be located in the lateral cerebellar hemispheres and less commonly in the cerebrum. Histologically, the tumor forms sheets of densely packed, poorly differentiated small cells. Occasional cells show spindle cell morphology and may stain for the glial marker glial fibrillary acidic protein (GFAP). Other occasional cells show neuronal differentiation and form rosettes. The tumor has a 50% 10-year survival rate when treated with combined surgery and radiation.
question
A 55-year-old woman comes to the physician because of worsening headaches. A CT of the head shows a well-circumscribed mass in the right cerebral hemisphere. Biopsy of the mass shows neoplastic cells with round nuclei and clear cytoplasm. Which of the following is the most likely diagnosis? A. Choroid plexus papilloma B. Ependymoma C. Glioblastoma multiforme D. Oligodendroglioma E. Pilocytic astrocytoma
answer
The correct answer is D. Oligodendroglioma has a "fried egg" appearance; tumor cells with round nuclei (the "yolk") and cleared cytoplasm (the "white"). These tumors may contain areas of calcification, hemorrhage, or cysts. They tend to occur in the cerebral hemispheres of middle-aged patients of both sexes, and have a better prognosis (average survival 5 years) than astrocytomas.
question
A 50-year-old woman comes to the physician for a follow-up examination. She has a 5-year history of headaches, generalized tonic-clonic seizures, and bilateral leg weakness. X-rays of the skull show hyperostosis of the calvarium. CT scan shows hyperostosis of the skull and an intracranial mass. Microscopic examination of a biopsy specimen from the mass shows a whorling pattern of the cells. Which of the following is the most likely diagnosis? A. Arachnoid cyst B. Glioblastoma multiforme C. Meningioma D. Metastatic breast cancer E. Oligodendroglioma
answer
The correct answer is C. The most likely diagnosis is an intracranial meningioma. Meningiomas are slow-growing, benign tumors comprising 15% of intracranial tumors; they are most common in the elderly, and because of their indolent course, may be present or even symptomatic for years before definitive diagnosis. Meningiomas are more common in females than males and have estrogen and progesterone receptors, which is why they grow during pregnancy. However, the distinct roles of estrogen and progesterone in meningioma are still unclear. They originate from either dura mater or arachnoid and are sharply demarcated from brain tissue. Meningiomas often incite an osteoblastic reaction in the overlying cranial bones. Clinically, they present as mass lesions; seizures may occur. The superior parasagittal surface of the frontal lobes is a favorite site of origin. This can often produce leg weakness, since the corticospinal fibers that control the legs originate in parasagittal cortical regions.
question
A 52-year-old man comes to the physician because of persistent headaches. A CT scan of the head shows a 2-cm spherical mass in the lateral aspect of the cerebral hemisphere at the junction of the white and gray matter. Which of the following is the most likely diagnosis? A. Astrocytoma B. Ependymoma C. Glioblastoma multiforme D. Meningioma E. Metastatic carcinoma
answer
The correct answer is E. Metastatic cancer from a non-CNS location is the most common CNS tumor. In order of frequency: metastases → astrocytomas (including glioblastoma) → meningioma → pituitary tumor. Additional indications that the tumor described is metastatic disease are the location at the junction of cortical gray and white matter and the round shape. They can be single or multiple. Tumors that tend to metastasize to the brain include lung, breast, kidney, colorectal cancer, and melanoma. Tumor emboli lodge into the gray/white matter junction, which is the most distant of the arterial blood supply. Vessels have a sharp reduction in caliber in this region. 70% of adult primary brain tumors occur above the tentorium (70% of childhood tumors occur below the tentorium).
question
[4] [5] (Garcia-Rill, motor; Masangkay, neurodegerative) A 47-year-old woman returns to the physician for follow-up examination for a progressive neurological condition. An MRI of her brain is shown. Which of the following symptoms is most likely to be found in this patient? The MRI shows abnormally large ventricles. A. Bradykinesia, resting tremor, and difficulty initiating movements B. Choreoathetosis and dementia C. Intention tremor and akinesia D. Moon facies, amenorrhea, and hypertension E. Motor and vocal tics F. Slowly progressive memory impairment and irritability G. Violent flinging of proximal limb muscles
answer
The correct answer is B. This patient most likely has Huntington disease (HD), which is characterized by choreoathetosis and dementia. The caudate degenerates markedly in this disease, although the putamen and nucleus accumbens also degenerate. Atrophy of the caudate and putamen can make the lateral ventricles appear very large when imaged (hydrocephalus ex vacuo). The image shows a horizontal brain section. Realize that as the caudate degenerates, the lateral ventricle will get larger.
question
A 60-year-old male executive with a history of angina and depression had bypass surgery one day ago and is now recovering in the intensive care unit. His depression has responded well to selective serotonin reuptake inhibitors and there is no history of psychosis. Today, he presents with confusion and irritability, and he tries to remove his IV line. Vital signs are stable. The nurses later note that he is somnolent, but during the night, he becomes agitated and seems confused. The patient also describes spiders crawling on the walls of his room. Treatment with risperidone is begun, and his symptoms improve. Which of the following is the most likely explanation for this patient's confusion and irritability? A. Adjustment disorder B. Delirium C. Mild neurocognitive disorder D. Schizophrenia E. Schizophreniform disorder
answer
The correct answer is B. Delirium is a common complication of general anesthesia and surgery. It can also be seen in intoxicated patients as well as those with severe infection, head trauma, seizure, and other severe medical conditions. It manifests by acute changes in mental status with waxing and waning level of consciousness, agitation, irritability, and psychosis. Patients may present with impaired awareness, confusion, and/or have perception disturbances, such as illusions, misinterpretations, and visual hallucinations, such as spiders crawling on the walls (as seen in this patient). When delirium is diagnosed, the underlying cause should be determined and subsequently treated. In many instances, resolution of the underlying condition will cause a resolution of delirium symptoms. However, patients may be administered low-dose neuroleptics (risperidone) and/or benzodiazepines (lorazepam, alprazolam) to achieve sedation. The course is usually self-limited. ICU delirium is a common diagnosis in the hospital due to the severe impact on the body and mind from the patient's condition.
question
A 70-year-old man with a history of bacterial meningitis is brought to the emergency department because of progressively worsening neurologic symptoms. Initially, he developed ataxia, then exhibited incontinence, frequent confusion, and difficulty with memory. Neurologic examination shows a bradykinetic, broad-based, magnetic gait, prominent memory loss, and decreased executive functioning. Which of the following is the most appropriate treatment? A. Aspirin B. Donepezil C. L-DOPA D. Ventricular shunt E. Vitamin B1 (thiamine)
answer
The correct answer is D. This patient has normal pressure hydrocephalus (NPH), which is a clinical syndrome produced by the build-up of cerebrospinal fluid (CSF). Patients have enlarged ventricles often with normal opening pressures on lumbar puncture. CT or MRI is often ordered initially to rule out other causes and can demonstrate ventriculomegaly. NPH is characterized by the classic triad (with the accompanying mnemonic): "Wet": urinary incontinence "Wacky": dementia (memory loss, slowed thought) "Wobbly": gait abnormality (apraxic "magnetic" gait) A - vascular dementia B - Alzheimer's C - Parkinson's/LBD E - Wernicke-Korsakoff
question
70-year-old woman with a history of diabetes and a recent severe urinary tract infection is brought to the physician by her son because of a recent episode of confusion and loss of orientation. The son said that his mother called him last night claiming to have seen snakes in her kitchen. However, when he arrived at her house, there were no snakes and the mother indicated that she also no longer saw them. Her temperature is 37.8°C (100°F), pulse is 86/min, respirations are 18/min, and blood pressure is 140/86 mm Hg. Psychiatric examination shows a deficit of recent memory and a diminished level of awareness. During the examination, the patient's speech becomes nonsensical and she begins to describe seeing bugs crawling on the walls of the room. Which of the following is the most likely diagnosis? A. Amnesia B. Delirium C. Dementia D. Schizoaffective disorder E. Schizophrenia
answer
The answer is B. Delirium is characterized by the acute onset of mental status changes that wax and wane. It may present as impaired awareness, easy distraction, confusion, and/or disturbances of perception such as: illusions, misinterpretations, and visual hallucinations. Recent memory is usually impaired, and speech may be rambling, perseverating, nonsensical, pressured, or incoherent. Patients may also be agitated or obtunded. Keep in mind that one of the most important features of delirium is that it is reversible.
question
A 28-year-old man comes to the physician after experiencing five days of intense, vivid dreams and nightmares. He states that he used to sleep erratically for the past several years and just wrote it off as anxiety from his job. Upon further questioning, he states that he was drinking three shots of whiskey to help him fall asleep for the past two years, but stopped this practice a week ago after frequent urgings from his wife. Which of the following could also precipitate the sleep symptom present in this patient? A. Benzodiazepine cessation B. Cocaine intoxication C. Lithium initiation D. Paroxetine cessation E. Phenelzine initiation
answer
The correct answer is D. The patient in this vignette is experiencing rapid eye movement (REM) rebound from the abrupt discontinuation of alcohol. Alcohol, when used chronically, will depress the amount of REM sleep. When the alcohol use is stopped, there will be a compensatory increase in the amount of REM sleep known as REM rebound. This phenomenon is characterized by an increase in the number and intensity of dreams for several days. REM rebound can also occur with the discontinuation of medications or drugs known to suppress REM including alcohol, barbiturates, stimulants, tricyclic antidepressants, selective serotonin reuptake inhibitors (SSRIs), lithium, and monoamine oxidase inhibitors. Paroxetine is a selective serotonin reuptake inhibitor and when discontinued abruptly may cause REM rebound similar to what this patient is experiencing.
question
A 22-year-old woman who is a college student comes to the physician for a routine examination. She says that she sometimes experiences very vivid and detailed visual images in the room about her as she wakes up in the morning. She has learned to recognize that what she is seeing is not real because some details will be inappropriate, such as summer clothes on the chair when she knows it is December. She reports no difficulties going to sleep, although she says she feels tired during the day. More detailed examination of this patient is likely to reveal which of the following additional symptoms? A. Inability to move after waking up B. Loud snoring while sleeping C. Sensations moving up and down her legs while going to sleep D. Sleep walking E. Teeth grinding while sleeping
answer
The correct answer is A. This patient, who likely has narcolepsy, is reporting hypnopompic hallucinations, which occur upon awakening. Hypnagogic hallucinations, which take place when going to sleep, also occur. These hallucinations are vivid, can be frightening, and can involve visual, tactile, or auditory sensations. These hallucinations are most often brief and simple, such as hearing a bell ring or a name called. However, they can be complex and of long duration, such as the one in the question stem. It is hypothesized that this results from the intrusion of the REM sleep process into the waking state; one cardinal feature of REM sleep is the active inhibition of skeletal muscle leading to paralysis. Medications that disrupt sleep architecture, such as serotonergic and anticholinergic antidepressants, can also produce hypnagogic and hypnopompic hallucinations.
question
A 45-year-old man is brought to the physician by his wife because of morning headaches. He has a history of hypertension and type 2 diabetes. He says that he falls asleep easily, but has difficulty staying asleep. His wife says that he snores loudly. He is 180 cm (5 ft 11 in) tall and weighs 133 kg (294 lb); BMI is 41 kg/m2. His temperature is 37.0ºC (98.6ºF), pulse is 83/min, respirations are 14/min, and blood pressure is 150/90 mm Hg. Physical examination shows no abnormalities. An overnight polysomnogram is performed and shows decreased stage 3 and REM sleep. Which of the following is the most appropriate next step in patient care? A. Cognitive behavioral therapy B. Head elevation C. Increase antihypertensive medications D. Tonsillectomy and adenoidectomy E. Triazolam F. Weight loss and use of continuous positive airway pressure
answer
The correct answer is F. This is a classic description of a person with obstructive sleep apnea (OSA) syndrome. The patient is typically a middle-aged, obese male who snores loudly during sleep. The problem stems from obstruction by tissues of the nasopharynx and hypopharynx. The obstruction results in significant periods of apnea associated with arterial hypoxemia and bradycardia. The patient may develop both systemic and pulmonary hypertension, and is at a high risk of sudden death during sleep due to severe hypoxemia and arrhythmias. Often the patient will have a long apneic period followed by gasping respirations and awakening. This cycle repeats over the course of the night and therefore the patient actually gets little sleep, often resulting in daytime somnolence. A polysomnogram (multichannel recording of sleep) reveals a decrease in stage 3 and REM sleep, which is typical of patients with OSA. T he initial medical treatment for OSA is weight loss, with a trial of nasal-prong or face-mask continuous positive airway pressure (CPAP).
question
A 78-year-old woman comes to the physician because of not feeling fully rested after a night's sleep. Physical examination and routine laboratory studies show no abnormalities. She reports no headaches upon wakening or dysesthetic sensations. Her husband says she does not snore often. Which of the following best explains her symptoms? A. Decrease in slow wave sleep B. Increased K-complexes during sleep C. Increased REM sleep time D. Onset of sleep apnea E. Restless legs syndrome
answer
The correct answer is A. Sleep consists of two distinct states: NREM and REM. NREM or Non-rapid Eye Movement sleep is divided into 4 stages on the basis of EEG criteria. NREM alternates with REM sleep throughout the sleep period and is characterized by a slowing of the EEG rhythms, higher muscle tone, absence of eye movements and absence of "thought like" mental activity. REM or Rapid Eye Movement sleep is characterized by an aroused EEG pattern, sexual arousal, saccadic eye movements, and elaborate visual imagery (dreaming). Multiple factors can contribute to sleep disturbances in the elderly, including primary sleep disorders, sleep disorders secondary to other physical and psychiatric conditions, as well as medication-induced sleep problems. However, many elderly complain of sleep problems when they are simply experiencing normal age-related sleep patterns.
question
A healthy 29-year-old man participates in a study to investigate the effects of a new psychoactive drug on sleep. Prior to administration of the drug during sleep, an electroencephalogram (EEG) and electromyogram (EMG) indicate that he has entered rapid eye movement (REM) sleep. After administration of the drug, the frequency and duration of episodes of REM sleep are reduced by 90%. Which of the following neurotransmitters is most likely inhibited by the drug? A. Acetylcholine B. Dopamine C. Epinephrine D. Norepinephrine E. Serotonin
answer
The answer is A. Acetylcholine is the neurotransmitter of primary importance for the induction of REM sleep. Some of the other neurotransmitters do function in sleep, but REM sleep can occur in their absence. The drug specifically disrupted REM sleep; this strongly implicates acetylcholine as the affected transmitter. A good way to remember the neurotransmitters associated with sleep is with the mnemonic "SANDman." Serotonin: helps to initiate sleep Acetylcholine (ACh): higher during REM sleep and also associated with erections in men Norepinephrine (NE): lower during REM sleep; the ratio of ACh and NE is the biochemical trigger for REM sleep Dopamine: produces arousal and wakefulness. Dopamine levels rise with waking.
question
A 55-year-old woman comes to the physician because of chronic headaches. She describes them as pulsatile, unilateral, and associated with light sensitivity. The patient is diagnosed with migraine headaches; however a CT scan of the brain is obtained to rule out an organic cause. An anomaly is reported on the CT scan requiring a follow-up cerebral arteriogram. The arteriogram shows a large aneurysm of the right superior cerebellar artery immediately distal to its origin from the basilar artery. Which of the following clinical findings is most likely to be seen as a result of compression of a cranial nerve by this aneurysm? A. Loss of abduction of the right eye B. Loss of adduction of the right eye C. Loss of depression of the right eye from the adducted position D. Loss of sensation on the right side of the face E. Loss of vision in the right eye
answer
The correct answer is B. The oculomotor nerve (CN III) emerges from the interpeduncular fossa of the midbrain and then passes between the superior cerebellar artery and the posterior cerebral artery immediately lateral to the basilar artery. Aneurysm of any of these three arteries may compress the nerve. The oculomotor nerve innervates a number of extraocular muscles in the orbit, including the medial rectus muscle, which is responsible for adduction of the eye.
question
A 62-year-old woman is brought to the emergency department because of a loss of consciousness. She has a history of hypertension. She regains consciousness several days later. Physical examination at that time shows quadriplegia; only vertical eye movements and the ability to blink are intact. When the physician asks her to blink two times if she can hear him, she does so. A lesion in which of the following structures is the most likely cause of this condition? A. Cervical spinal cord B. Medulla oblongata C. Midbrain D. Pons E. Thalamus
answer
The correct answer is D. This patient cannot move either her arms or legs, thus she has bilateral symptoms. The basilar artery supplies both sides of the ventral and medial pons. This patient has sustained a stroke of a segment of the basilar artery, leading to a lesion of the ventral pons (see figure of mid-pons cross-section below). Descending corticospinal and corticobulbar fibers are interrupted.
question
A 54-year-old man with a history of chronic otitis media comes to the physician because of gradually progressing hearing loss and discharge from the right ear. An MRI shows evidence of a cholesteatoma in the middle ear. After diagnosis, surgery is performed to remove the cholesteatoma. The surgery resulted in injury to a nerve that passes through the middle ear cavity. Following the procedure, the patient notes significant improvement in his otologic symptoms; however, he begins to notice increasingly severe symptoms of dry mouth. Salivation from which of the following glands is most likely to be impaired as a result of the surgery? A. Parotid and sublingual glands B. Parotid and submandibular glands C. Parotid gland only D. Sublingual gland only E. Submandibular and sublingual glands F. Submandibular gland only
answer
The correct answer is E. The chorda tympani branch of the facial nerve (CN VII) passes through the middle ear cavity and can be injured during this surgery. The chorda tympani contains preganglionic parasympathetic fibers that synapse in the submandibular ganglion. Postganglionic fibers from this ganglion innervate the submandibular and sublingual glands. The chorda tympani also carries special sensory fibers for taste from the anterior 2/3 of the tongue, so this patient might also have a diminished sense of taste.
question
A 38-year-old man comes to the physician because of a painful lesion on his tongue. He has a 5-week history of crampy left-sided abdominal pain, an urgency to move his bowels, and low-volume bloody mucoid stool for which he had been previously seen. Physical examination shows an aphthous ulcer on the tip of the tongue. The cranial nerve that transmits this patient's pain from this ulcer is also involved in which of the following functions? A. General sensation of the posterior 1/3 of tongue and pharynx B. Mastication C. Salivary gland secretion D. Taste from the anterior 2/3 of the tongue E. Tongue protrusion
answer
The correct answer is B. This patient has inflammatory bowel disease. Patients can also experience extra-intestinal symptoms such as oral aphthous ulcers, arthritis, and erythema nodosum. The innervation of the tongue is complex. The mandibular division of the trigeminal nerve (V3) carries general somatic sensation (pain and temperature) from the anterior two-thirds of the tongue. So the question is, what else does CN V3 do? It provides motor innervation to the muscles of mastication (temporalis, masseter, medial and lateral pterygoids). Other motor functions of this branch of the trigeminal nerve include innervation to the mylohyoid, the anterior belly of the digastric muscle, the tensor tympani, and the tensor veli palatini. CN V3 also provides general sensation to the lower third of the face, buccal mucosa, and parts of the outer ear.
question
An 11-year-old girl presents to her primary care physician after her seventh episode of a sore throat during the last year. Her mother reports that she has a history of recurrent upper respiratory tract infections. Because of this, the decision is made to undergo bilateral tonsillectomy. There is significant bleeding during the surgery and while attempting to ligate a blood vessel, the nerve that lies in the tonsillar fossa, deep to the palatine tonsil, is damaged. Which of the following deficits is most likely to result from this injury? A. Loss of sensation on the posterior one-third of the tongue B. Loss of taste on the anterior two-thirds of the tongue C. Paralysis of the constrictor muscles of the pharynx D. Paralysis of the muscles of the soft palate E. Paralysis of the muscles of the tongue
answer
The correct answer is A. The nerve that lies in the tonsillar fossa is the glossopharyngeal nerve. This nerve provides general sensory innervation to the mucosa of the pharynx, and general sensory and taste sensation to the mucosa of the posterior one-third of the tongue. This nerve also innervates the stylopharyngeus muscle, which helps to elevate the pharynx, but this innervation occurs proximal to the point at which the nerve crosses the tonsillar fossa.
question
A 59-year-old man is brought to the emergency department after being found unconscious by his wife. On examination, he is unable to move his extremities and has decerebrate posturing. The patient is unable to speak or move his face. He is able to answer yes/no questions by using eye blinks or vertical eye movements, but has no horizontal eye movements. He is unable to move his limbs in response to commands and has upgoing plantar responses bilaterally. Pathology of the following vessels is most likely responsible for this patient's condition? A. Anterior inferior cerebellar artery (AICA) B. Basilar artery C. Middle cerebral artery D. Posterior inferior cerebellar artery (PICA) E. Vertebral artery
answer
The correct answer is B. One large clinical clue is that patient has bilateral symptoms—all four limbs are affected and he has bilateral Babinski signs. In addition, the patient has awareness and is able to move his eyes vertically and blink. The only vessel listed that can cause bilateral signs and symptoms is the basilar artery, which supplies both sides of the ventral and medial pons.
question
An 84-year-old woman who lives in a nursing home has had multiple strokes. Following the last stroke, she has experienced worsening dysphagia and dysarthria. Neurologic examination shows an absent gag reflex. Which of the following is an additional function of the nerve that is involved in the afferent arc of this reflex? A. Innervates muscles of mastication B. Salivation C. Sensory input from thoracic viscera D. Taste from epiglottis E. Taste from the anterior two-thirds of the tongue
answer
The correct answer is B. The gag reflex is elicited by touching either side of the posterior pharynx with a tongue blade, producing bilateral elevation of the palate and bilateral contraction of the pharyngeal muscles. The afferent of this reflex arc consists of the ipsilateral glossopharyngeal nerve, while the vagus nerve, bilaterally, supplies the efferent limb. After determining that the glossopharyngeal nerve participates in the afferent arc of this reflex, you must determine the other functions of CN IX. One function of CN IX is to innervate the parotid glands, which helps mediate salivation. The other salivary glands (submandibular and sublingual) are innervated by the facial nerve (CN VII).
question
A 32-year-old woman comes to the physician because of double vision that is worse when she looks to the right. She states that when she closes one eye, her vision returns to normal. An MRI of the brain shows a demyelinating lesion in the brain stem. She says that earlier in the year, she experienced tingling in her right arm. Which of the following best represents the most likely appearance of the patient's eyes if she looks to the right?
answer
The correct answer is D. Right eye goes to the right, while the left eye remains centered. The combination of diplopia with a demyelinating central nervous system lesion identified on MRI, coupled with a history of other neurologic symptoms separated by space and time, is strongly suggestive of multiple sclerosis. Binocular diplopia results when there is misalignment of the two eyes. This may result from congenital malposition of extraocular muscles, disorders of the neuromuscular junction, cranial nerve involvement, or central nervous system (CNS) lesions. Given the MRI findings, the problem is most likely caused by a CNS lesion. Choice D demonstrates an inability to adduct the left eye on right lateral gaze, a pattern that is pathognomonic for internuclear ophthalmoplegia (INO). Internuclear ophthalmoplegia is caused by damage to the medial longitudinal fasciculus (MLF), which links the abducens nucleus of CN VI with the oculomotor CN III nucleus during lateral gaze. Note that the left eye cannot move past the midline (adduct) during attempts at conjugate gaze. However, convergence is intact in these patients (i.e., the left eye CAN adduct during non-conjugate gaze; this fact distinguishes an INO from a medial rectus (CN III) palsy). As the patient attempts to look to the right, nystagmus in the right eye will also be observed.
question
A 34-year-old woman with a history of diabetes comes to the physician because of ear pain 2 days after swimming in a local pool. Physical examination shows a greenish discharge from the right ear canal. Erythema is localized to the anterior half of the external ear canal, and manipulation of the tragus elicits extreme tenderness. Which of the following nerves transmits the painful sensations this patient is experiencing? A. Auricular branch of the vagus nerve B. Auriculotemporal branch of the mandibular nerve C. Chorda tympani D. Lesser occipital nerve E. Vestibulocochlear nerve
answer
The correct answer is B. The ear has a complex sensory nerve supply, which includes all of the nerves listed. A consequence of this complexity is that pain actually originating in other sites (teeth and sinuses are notorious) may be misinterpreted as ear pain; or, less commonly, pain originating in the ear may be misinterpreted as arising from other sites. The anterior half of the external ear canal is supplied by the auriculotemporal branch of the mandibular nerve, which also supplies the facial surface of the upper part of the auricle. The complex sensory innervation of this region is a reflection of its embryonic origin. The external ear canal arises from the first pharyngeal cleft. This means that the first arch is in front of it and the second, third, and fourth arches are behind it. Since the mandibular division of the trigeminal nerve innervates the first arch, the facial nerve innervates the second arch, the glossopharyngeal nerve innervates the third arch, and the vagus nerve innervates the fourth arch we can anticipate that all of these nerves would provide sensory innervation in this region. The auricular branch of the vagus (choice A) supplies the posterior half of the external ear canal. It is because of this vagal innervation that a painful examination of the ear canal can lead to fainting in the patient.
question
A 54-year old woman is admitted to the intensive care unit because of sepsis secondary to a urinary tract infection. The decision is made to place a central line catheter in the lumen of the right internal jugular vein to better track hemodynamic information. A resident performs the procedure guided by ultrasound, but during the course of the procedure, inadvertently pierces through the vein and damages structures located posterior to the carotid sheath. Which of the following is most likely to occur? A. Constriction of the right pupil B. Dilation of the right pupil C. Inability to abduct the right eye D. Inability to close the right eye E. Paralysis of the platysma muscle on the right side
answer
The correct answer is A. The right sympathetic trunk lies posterior to the right carotid sheath, and may be injured in this procedure. Preganglionic sympathetic nerve fibers arising from the T1 spinal cord segment and ascending through the cervical sympathetic chain will be damaged. These nerve fibers synapse in the superior cervical ganglion on postganglionic sympathetic neurons that innervate structures in the head. The dilator pupillae muscle (smooth muscle of the iris that dilates the pupil) is sympathetically innervated; paralysis of this muscle due to interruption of its innervation results in constriction of the pupil (miosis).
question
A 73-year-old woman who was recently diagnosed with a brain tumor comes to the physician because of progressively worsening headaches. To view the neurologic examination, click on the "Play Media" button. Disruption of function of which of the following is most likely responsible for this patient's symptoms? Media shows patient's right eye is down and out with dilated pupil. A. Abducens nerve B. Facial nerve C. Oculomotor nerve D. Optic nerve E. Sympathetic chain F. Trigeminal nerve G. Trochlear nerve
answer
The correct answer is C. This patient has a right oculomotor palsy, as evidenced by ptosis of her right eye (impaired levator palpebrae muscle function), lack of medial gaze of her right eye (impaired medial rectus muscle function), lack of upward gaze (impaired superior rectus muscle function), and loss of downward gaze (impaired inferior rectus function). When looking straight ahead, her right eye is abducted as a result of unopposed lateral rectus action. Additionally, she has a dilated right pupil (CN III normally carries parasympathetic fibers to constrict the eye). If evaluated further, she would probably also have paralysis of accommodation (cycloplegia) of her right eye (due to interruption of parasympathetic innervation to her eye).
question
A 67-year-old man comes to the physician because of hearing loss. Physical examination shows a large amount of wax blocking the patient's right ear canal. The patient faints as a water-filled syringe is used to expel wax from the ear canal. This event can be explained by the fact that the posterior half of the external ear canal receives sensory innervation from which of the following? A. Auricular branch of the vagus nerve B. Auriculotemporal branch of the mandibular nerve C. Chorda tympani D. Greater auricular nerve E. Vestibulocochlear nerve
answer
The correct answer is A. The vagus nerve, through its auricular branch, supplies the posterior half of the external auditory meatus. Since the vagus also supplies many chest and abdominal organs, reflex symptoms may occur, including fainting (vasovagal syncope), coughing, and gagging. Vasovagal syncope is the most common cause of syncope and accounts for approximately 20 -25% of all cases, particularly in patients without neurologic or cardiac disease. Reflex syncope is also the most common cause of syncope in heart disease patients and should be considered as a possible cause in these patients.
question
A 22-year-old woman comes to her physician because of anxiety and irritability. She reports that she recently has had a significant weight loss despite normal eating habits. Her menstrual cycle has become irregular, she has been having frequent bowel movements, and she has been having difficulty sleeping. Physical examination shows a fine tremor of her hands and a rapid and irregular heartbeat. There is a noticeable bulge in her anterior neck just below her larynx. She initially undergoes medical management, but it is not successful. Surgical treatment of her underlying disease is scheduled. Three days after surgery, the patient develops hoarseness and can barely speak above a whisper. Damage to a branch of which of the following nerves is the most likely cause of the hoarseness in this patient? A. Facial B. Glossopharyngeal C. Hypoglossal D. Trigeminal E. Vagus
answer
The correct answer is E. This patient had hyperthyroidism, likely a result of Graves' disease. The surgical treatment for this is subtotal or total thyroidectomy. The recurrent laryngeal nerves are at risk during this surgery because of their close relationship to the inferior thyroid arteries. The recurrent laryngeal nerves are branches of the vagus (CN X), and supply all intrinsic muscles of the larynx except the cricothyroid (which is innervated by the external laryngeal nerve, a branch of the superior laryngeal branch of CN X).
question
A 26-year-old man comes to the physician because of a 3-week history of headaches and difficulty looking up. Neurologic examination shows pupils are 6 mm in diameter bilaterally with minimal reaction to light. Pupils do react to accommodation. He is unable to look upward past the horizontal plane, but otherwise has normal eye movements. MRI of the head shows a brain tumor. Compression of which of the following structures is most likely responsible for the findings in this patient? A. Mammillary body B. Optic chiasm C. Pons D. Superior cerebellar peduncle E. Superior colliculus
answer
The correct answer is E. The clinical syndrome is Parinaud syndrome, caused in this case by a pineal tumor that is compressing the superior colliculi and pretectal area of the dorsal midbrain, the region that contains the vertical gaze center. This produces a vertical palsy. Pineal tumors can also cause compression of the cerebral aqueduct, resulting in a noncommunicating hydrocephalus.
question
A 12-year-old girl is brought to the physician because of a severe sore throat. She describes significant pain with swallowing and general feelings of malaise. Her parents report that the patient has had a high fever for the past several days. Physical examination shows an erythematous patch on the upper posterior pharynx and tenderness of the cervical lymph nodes. Her temperature is 39.5°C (103.1°F). The nerve responsible for this patient's throat pain is also involved in which of the following functions? A. Head rotation to opposite side B. Movement of intrinsic tongue muscles C. Sensation of the lower jaw D. Sensory input from carotid sinus baroreceptors E. Taste in the anterior 2/3 of the tongue
answer
The correct answer is D. Recall that afferent nerve traffic (sensory) travels toward the central nervous system (CNS) and efferent nerve traffic (motor) travels away from the CNS. The glossopharyngeal nerve (IX): • carries general somatic sensation from the posterior part of the upper pharynx, eustachian tube, middle ear, and posterior one-third of the tongue • conveys afferent fibers from the carotid sinus baroreceptors and carotid body chemoreceptors • carries taste sensation from the posterior one-third of the tongue • sends efferent fibers to the stylopharyngeus muscle • carries parasympathetic fibers that synapse in the otic ganglion in the pathway for innervation of the parotid gland.
question
A 70-year-old man with a history of diabetes and hypertension comes to the physician because of sudden-onset ataxia, hoarseness, and difficulty swallowing for the past 2 days. He says that he felt the room was spinning, then he fell to the ground and was unable to ambulate because of weakness and lack of coordination. One day later, he developed trouble swallowing food and a hoarse voice. Neurologic examination shows anisocoria with the left pupil constricted relative to the right, left-sided ptosis, and decreased elevation of the palate. Temperature and pinprick sensation are reduced on the left face and on the right side of the body below the neck. Which of the following diagrams best represents the pattern of this man's lesion? Where is the lesion? Midbrain, medulla, pons? What area of this section?
answer
The correct answer is B. Lateral Medulla. This patient has a lateral medullary syndrome, also known as Wallenberg syndrome, which is caused by occlusion of the posterior inferior cerebellar artery (PICA).
question
A 62-year-old woman comes to her physician for a follow-up examination. To view her neurologic examination, click on the "Play Media" button. Which of the following sites contains the cell bodies that give rise to the nerve fibers that are damaged in this patient? Media shows left Bell Palsy. A. Left medulla B. Left midbrain C. Left motor cortex D. Left pons E. Right medulla F. Right midbrain G. Right motor cortex H. Right pons
answer
The correct answer is D. This patient has a lesion of the left facial nerve (CN VII), a condition known as Bell palsy. This patient is showing weakness of muscles of facial expression, including a left-sided weakness in her ability to wrinkle her forehead and to move her mouth, indicating a LMN (CN VII) lesion. The facial nucleus is located in the pons and sends motor fibers to the ipsilateral face via CN VII.
question
A 42-year-old woman develops a painful blister over the vermilion of her upper lip. She has had several episodes of similar symptoms since age 12. The causal agent has most likely established latency on a nerve that exits the skull through which of the following? A. Foramen ovale B. Foramen rotundum C. Jugular foramen D. Stylomastoid foramen E. Superior orbital fissure
answer
The correct answer is B. Herpes simplex is an enveloped, linear DNA virus that causes painful skin vesicles, often called "cold sores" or "fever blisters" to denote the precipitating event that preceded the appearance of the lesions. Herpes simplex virus-1 (HSV-1) belongs to the herpesvirus family; herpesviruses multiply in the nucleus and surround themselves with nuclear membrane envelopes. HSV-1 can remain latent in the trigeminal root ganglion. The treatment in immunocompromised patients or patients with severe infections is acyclovir. Because the patient's lesions are on the upper lip, the nerve that transported HSV-1 to the site of recurrence had to be a branch of the maxillary division (V2) of the trigeminal nerve. The maxillary nerve exits the skull via the foramen rotundum.
question
A 58-year-old man comes to the physician because of recurrent headaches and ear pain. The pain worsens with chewing and yawning. Occasionally, the patient also notices a "clicking sensation" near his ear while chewing. On physical examination, firm palpation below the ear reproduces the pain. The structures indicated by the arrows on this CT scan are tender to palpation. Which of the following is the most likely location of the cranial nerve nuclei that provide motor innervation to the indicated structures (it's the temporalis muscle)? A. Lower pons B. Medulla C. Midbrain D. Mid-pons E. Spinal cord
answer
The correct answer is D. This patient has temporomandibular joint syndrome (TMJ). Common symptoms include chronic pain in the muscles of mastication, pain that can radiate to the ear and is worse when chewing, ear clicking or popping, headache, and locking of the jaw when attempting to open the mouth. The muscle indicated on the CT scan is the temporalis muscle, one of the muscles of mastication (the others are the masseter, the lateral pterygoid and the medial pterygoid muscles). It is innervated by cranial nerve V, the trigeminal nerve.
question
A 47-year-old man with a history of diabetes comes to the physician because of recent onset of double vision. He says his symptoms are particularly bad when looking to the right. He also describes polyuria, polyphagia, and tingling in his extremities. His last HgBA1c was 8.6%. To view the examination, click on the "Play Media" button. This patient most likely has a lesion of which of the following nerves? Media shows difficult to abduct the right eye. A. CN III B. CN IV C. CN V D. CN VI E. CN VII
answer
The correct answer is D. This patient has dysfunction of the right lateral rectus muscle, which is innervated by CN VI (abducens nerve). The patient is unable to abduct his right eye when looking to the right, but can adduct it normally.
question
A 47-year-old man with a history of diabetes comes to the physician because of recent onset of double vision. He says his symptoms are particularly bad when looking to the right. He also describes polyuria, polyphagia, and tingling in his extremities. His last HgBA1c was 8.6%. To view the examination, click on the "Play Media" button. This patient most likely has a lesion of which of the following nerves? Media shows difficult to abduct the right eye. A. CN III B. CN IV C. CN V D. CN VI E. CN VII
answer
The correct answer is D. This patient has dysfunction of the right lateral rectus muscle, which is innervated by CN VI (abducens nerve). The patient is unable to abduct his right eye when looking to the right, but can adduct it normally.
question
A 45-year-old man comes to the emergency department because of neck pain, dizziness, hoarseness, and facial numbness following chiropractic neck manipulation. Neurologic examination produces a gag reflex with deviation of the uvula to the right when the right side of the pharyngeal mucosa is stimulated. When the left side of the pharyngeal mucosa is stimulated, the patient does not gag. These findings are most likely caused by a lesion in which of the following structures? A. Left glossopharyngeal nerve and left vagus nerve B. Left glossopharyngeal nerve and right vagus nerve C. Left glossopharyngeal nerve only D. Left vagus nerve only E. Right glossopharyngeal nerve and left vagus nerve F. Right glossopharyngeal nerve and right vagus nerve G. Right glossopharyngeal nerve only H. Right vagus nerve only
answer
The correct answer is A. The number of options may at first appear overwhelming, but with a little basic knowledge and using the process of elimination, the correct answer can be easily identified. In order to reason through this question, we have to analyze the sensory limb of the gag reflex (CN IX) and the motor limb of the reflex (CN X) separately. In order for there to be any gag, the patient must feel the stimulus using CN IX. So in this case, when the right side of the pharynx is stimulated there is a gag (even though not a normal gag) and when the left side is stimulated there is no gag. So we know that the right glossopharyngeal nerve is functioning, so we can eliminate all of the choices that include a lesion of right CN IX (choices E, F, and G) Normally, when there is a gag, both the right side of the palate is elevated (right CN X) and the left side of the palate is elevated (left CN X) and the uvula therefore goes straight up (no deviation). But in this case, when there is a gag, the uvula deviates to the right. This means that only the right side of the palate is elevating. This means that the right vagus nerve is functional but the left is not. We can now eliminate any choice that includes a lesion of the right vagus nerve (choices B, F, and H). We can eliminate left CN IX only (choice C) because we know there is a lesion of left CN X because of the deviation of the uvula when the patient gags. We can eliminate left CN X only (choice D) because we know there is a lesion of left CN IX because when the left side of the pharynx is stimulated, there is no response at all because the patient cannot feel it. We have now eliminated all choices except choice A. With a lesion of the left CN IX and left CN X, the patient will not gag when the left side is stimulated and will gag with deviation of the uvula to the right, when the right side is stimulated. Remember, the uvula deviates away from the side of the lesion of CN X.
question
A 39-year-old woman comes to the physician because of a 2-month history of headaches. She has no history of major medical illness. Vital signs are within normal limits. A head CT shows a mass above the left petrous bone. A biopsy specimen shows a malignant schwannoma on the left trigeminal nerve. The patient undergoes surgery for resection of the mass. Which of the following is most likely to be found on examination if there is damage to the nerve during the surgery? A. Decreased accommodation of the left eye B. Decreased bilateral corneal reflex when the left cornea is touched C. Decreased gag reflex D. Decreased taste from the anterior two-thirds of the tongue E. Inability to adduct the left eye
answer
The correct answer is B. The trigeminal nerve (CN V) is a mixed (sensory and motor) nerve. It provides sensory innervation to the face, cornea, mucous membranes of the nasal and oral cavities, frontal sinus, anterior two-thirds of the tongue, teeth, and hard palate. It also provides motor innervation for the muscles of mastication (i.e., temporalis, masseter, lateral and medial pterygoids), anterior belly of the digastric, tensor tympani, tensor palati, and the mylohyoid muscles. If this patient's left CN V was lesioned during surgery, she could lose the corneal (blink) reflex on the left.
question
A 50-year-old woman comes to the physician because of headache, nausea, and vomiting. She does not drink alcohol or smoke cigarettes. She does not take any medications. Neurologic examination shows that her pupils have a minimal reaction to light, but do constrict during accommodation. She is unable to look upward past the horizontal plane, but has full conjugate eye movements in other directions; she is unable to converge. Visual field testing shows no abnormalities. Which of the following is the most likely diagnosis? A. Acoustic neuroma B. Astrocytoma in the cerebellum C. Craniopharyngioma D. Parasagittal meningioma E. Pinealoma F. Tabes dorsalis
answer
The correct answer is E. Based on the neurological examination, this patient has a lesion of the vertical gaze center in the dorsal midbrain. Of the answer choices, a tumor of the pineal gland (e.g., germinoma) is most likely, because the close proximity of the pineal gland to the midbrain tectum.
question
A 75-year-old woman comes to the physician because of fatigue and exhaustion. A review of her records shows that her spouse died 4 months ago in a motor vehicle collision and that she was a passenger in that vehicle. She states that she has bouts of crying for no reason, and that she sometimes thinks she hears her husband calling. For the most part, when she is outside or around friends she does not experience any of those symptoms. It is only when she is alone at home or looking at past pictures that she experiences sadness. She is still able to drive around and perform her activities of daily living and says that she does not have nightmares about the accident or flashbacks in which she re-experiences the event. Her vital signs are within normal limits, however, her weight has decreased by about 10 pounds since the death of her husband. Which of the following is the most appropriate response? A. "Do you think about your husband often?" B. "I'm going to admit you to the hospital for overnight observation so we can run a few tests." C. "I'm going to give you something to help you feel better and make it easier for you to sleep." D. "I'm going to schedule you for an evaluation with a colleague of mine who can help us understand why you are feeling this way." E. "It sounds like the loss of your husband was difficult for you, but it is time for you to think about your own life now." F. "You have all of the symptoms of depression. Do you understand what that means?"
answer
The correct answer is A. This case presents a woman who is having a normal grief and mourning reaction to the loss of her husband. This is also known as bereavement. The grief response is likely exacerbated by the fact that she survived the accident and he did not, which is called "survivor's guilt." The key to this question is that the physician must recognize that although some of the symptoms of depression are present, this woman is experiencing normal grief. Grief usually comes and goes and is normally triggered by memories that are associated with the deceased such as a birthday or special location or object. She does not need diagnosis or drugs, but does need someone to talk to and listen to her.
question
A 47-year-old woman comes to the physician because of dizziness and abdominal pain. She says that the pain is so severe that she cannot stand up or eat anything. Physical examination shows no abnormalities; multiple, colorful tattoos are noted. A review of her records shows previous treatments for alcoholism and indicates that she has seen 4 different physicians in the past year. During the examination, she looks the physician in the eye, smiles warmly, and says that she hopes to have a close, intimate relationship with him. With tears in her eyes, she asks if she will be all right, and then laughs when the physician attempts to reassure her. As she dresses to leave, she puts on a bright yellow dress and an elaborate necklace. Which of the following is the most likely diagnosis? A. Avoidant personality disorder B. Borderline personality disorder C. Dependent personality disorder D. Histrionic personality disorder E. Narcissistic personality disorder F. Schizoid personality disorder G. Schizotypal personality disorder
answer
The correct answer is D. The flamboyant manner, flirtatious behavior, shifting emotions, assumed intimacy, and general theatrical behavior are consistent with a diagnosis of histrionic personality disorder. Other characteristics of this histrionic personality disorder include an inability to maintain long-lasting relationships and are viewed as being attention-seeking and constantly wanting the "spotlight."
question
A 46-year-old man with HIV infection and schizoaffective disorder is admitted to the hospital. A review of his records shows that he has been hospitalized several times and was prescribed combination therapy, but he has been noncompliant with his medications over the past 5 weeks. Laboratory studies show thrombocytopenia, with a platelet count of 11,000/mm3. Based on this information, which of the following drugs is contraindicated in this patient? A. Benztropine B. Haloperidol C. Lithium D. Lorazepam E. Valproic acid
answer
The correct answer is E. In this question, one is asked to determine which of the answer choices are contraindicated for use in this patient. Valproic acid is associated with thrombocytopenia. This effect is dose-related and it has been observed in clinical trials in more than 5% of patients who were on a higher dose of the drug. In the case of this HIV-positive patient, the risk of frank hemorrhage is increased. Valproic acid can also cause hepatotoxicity (from a toxic metabolite), pancreatitis, alopecia, and is a teratogenic (spina bifida).
question
76-year-old woman comes to the physician every week because of various minor health problems. Despite the physician's repeated assurances that the woman is basically healthy and capable of making minor medical decisions herself, she remains unconvinced and repeatedly returns to the physician. She explains to the physician that she feels uncomfortable with making decisions for herself and wants the physician to make medical decisions for her. At the end of each interview, she always thanks the physician for his help. Which of the following is a common feature of this patient's likely personality disorder? A. Distrust and suspiciousness B. Excessive emotionality C. Grandiose sense of self-importance D. Low self-confidence E. Preoccupation with order F. Sensitive to rejection
answer
The correct answer is D. This patient is exhibiting signs of dependent personality disorder, a Cluster C personality disorder, which is evidenced by a marked tendency to surrender control of major areas of the person's life to other individuals. Patients with this disorder often lack self-confidence, feel an excessive need to be taken care of, and are submissive and clinging. Other characteristics include needing others to assume responsibility, difficulty initiating projects, feeling uncomfortable when alone, and often an unrealistic preoccupation with fears of being left to take care of oneself.
question
A 39-year-old woman comes to the physician because of a 4-month history of restlessness. She says she has been having trouble sleeping, and is easily startled by loud noises. She also reports a poor appetite and a 2.3 kg (5 lb) weight loss, and she has been unable to focus at work. She is often distracted by images of a motor vehicle accident that she experienced 7 months ago. She also feels more reclusive, and whenever she needs to leave home, she takes the train or bus only, even though she has a car. Which of this patient's symptoms help distinguish her diagnosis from an acute stress disorder? A. Difficulty with sleep B. Fear of driving C. Onset of symptoms >6 months after the event D. Re-experiencing traumatic events E. Symptoms lasting >1 month F. Weight loss
answer
The correct answer is E. This patient has post-traumatic stress disorder (PTSD), which is now classified in DSM-5 as a trauma- and stressor-related disorder. The diagnostic criteria include exposure to a traumatic event, re-experiencing the event, persistent avoidance of stimuli associated with the trauma (this patient does not want to drive; she uses public transportation only), numbing of general responsiveness, symptoms of increased arousal (insomnia, restlessness), duration of disturbance of more than 1 month, and disturbance causing significant distress or impairment in social, occupational, or other important areas of functioning. PTSD often follows acute stress disorder, which lasts less than 4 weeks.
question
A 47-year-old man comes to the physician 1 year after his wife died in a motor vehicle accident. Over the course of the year, he has had increased feelings of worthlessness and no longer is proud of his accomplishments. He continues to feel that his wife's passing was his fault and blames himself for the event. Over this time, he has had continued difficulty sleeping, which he attributes to his decreased energy at work and why he no longer plays golf with his friends. He has maintained his job, but feels his work is not up to the same standard it was before the accident. As his work has been suffering, he has also been spending a lot less time with friends who have noticed he turns down their offers to socialize much more frequently. He has also lost 6.8 kg (15 lb) since his last office visit. He says that his symptoms have been worsening but he assures the physician that he does not want to hurt himself. Which of the following is most likely associated with this patient's condition? A. Episodes of increased energy B. Increased REM sleep C. Suicidal ideation is rare D. Tactile hallucinations E. Worse during winter months
answer
The correct answer is B. This patient most likely has major depressive disorder. An easy way to remember the symptoms of major depressive disorder is with the mnemonic SIG-E-CAPS: S leep changes: increase during day or decreased sleep at night I nterest (loss): of interest in activities that used to interest them G uilt (worthless): depressed persons may devalue themselves E nergy (lack): common presenting symptom (fatigue) C ognition/Concentration: reduced cognition &/or difficulty concentrating A ppetite (weight loss); usually declined, occasionally increased P sychomotor: agitation (anxiety) or lethargic S uicide/preoccupation with death Patients with major depressive disorder have increased REM sleep, although they often complain of insomnia. They usually exhibit decreased REM latency and decreased stage N3 sleep.
question
A 32-year-old woman comes to the physician because of major depressive episodes that have not responded to multiple standard antidepressant medications in addition to various types of psychotherapy. The physician explains that her depression is not responding to standard drugs and that a course of electroconvulsive treatment (ECT) may effectively treat her depression. When getting informed consent for this treatment, the physician should inform the patient that which of the following is a likely adverse effect of ECT? A. Acute psychosis B. Auditory hallucinations during the 2 to 3 day period following the treatment C. Memory loss of events before and after the treatment D. Miscarriage if she is pregnant E. Status epilepticus
answer
The correct answer is C. Electroconvulsive therapy (ECT) has been shown to be a safe and effective treatment measure for many psychiatric disorders, including major depressive disorder. The precise mechanism of action of ECT is not fully understood. However, it is believed to affect multiple CNS components, including hormones, neuropeptides, neurotrophic factors, and neurotransmitter systems, including beta-adrenergic, serotonin, muscarinic, cholinergic, and dopaminergic systems. It is indicated for selected patients with major depressive disorder, bipolar disorder, schizophrenia, and many other disorders. Most patients report some adverse cognitive effects during and after a course of ECT. ECT causes three basic types of cognitive impairment: acute confusion, anterograde amnesia and retrograde amnesia. Memory loss is usually short-term but occurs in 75% of the patient population. The memory loss includes both retrograde (events occurring before treatment) and anterograde (events occurring after treatment).
question
A 36-year-old woman comes to the physician for a routine examination, but arrives 2 hours late and the physician has already left. The patient states in very loud and pressured speech that her time is precious, she cannot return on another day, and demands to be seen by another physician. The receptionist has seen this patient act this way on many occasions. Based on the patient's history and interactions in the office which of the following is the most likely diagnosis? A. Antisocial personality disorder B. Bipolar disorder C. Borderline personality disorder D. Generalized anxiety disorder E. Histrionic personality disorder F. Narcissistic personality disorder G. Obsessive-compulsive personality disorder
answer
The correct answer is F. This patient most likely has narcissistic personality disorder, which is characterized by grandiose self-images, fantasies of glory, exploitative behavior, a sense of entitlement, and a lack of empathy and concern for others. Patients with this disorder demand constant attention and have fragile self-esteem. There is genuine surprise and associated anger when they do not obtain their goals. It is more commonly diagnosed in men.
question
A 14-year-old boy is brought to the emergency department by his parents because of a seizure. His mother says that he developed a fever, a severe prefrontal headache, and an altered sense of smell ten days after swimming in a lake near his camp. Physical examination shows a profuse green discharge from the nose. Examination of cerebrospinal fluid (CSF) shows white blood cell count of 700 cells/mm3 with PMN predominance, 30 mg/dL glucose, and 60 mg/dL protein. He dies two days after admission despite aggressive therapy. Which of the following is most likely to be found on microscopic examination of the central nervous system at autopsy? A. Gram-negative, encapsulated diplococci B. Gram-positive, catalase-negative diplococci C. Motile flagellated trophozoites D. Multinucleated giant cells and intranuclear inclusion bodies E. Ribbon-like hyphae branching at 90-degree angles F. Urease-positive, encapsulated yeasts
answer
Answer:C. This child has acquired primary amebic meningoencephalitis (PAM) caused by Naegleria fowleri. This organism is a free-living ameba that is found in warm, freshwater sources worldwide (lakes, ponds, rivers, hot springs, spas). Transmission to humans occurs primarily through inhalation of infested water. During the act of diving into fresh water lakes, changes in pressure can force the organism through the cribriform plate and result in the production of necrotic lesions spreading from the olfactory lobes. Progression of this disease is often rapid leading to death within a few days
question
A newborn who is born at 30 weeks' gestation is evaluated in the neonatal intensive care unit 10 days after birth because of a seizure. His mother states that she did not receive prenatal care during her pregnancy. His temperature is 38.4°C (101.1°F), pulse is 174/min, respirations are 59/min, and blood pressure is 67/42 mm Hg. Physical examination shows lethargy and a diminished sucking reflex, which was normal at birth. Lumbar puncture shows: Leukocyte count 989 cells/μL Neutrophils (PMNs) 14% Lymphocytes 86% RBC 440 cells/μL Glucose 44 mg/dL Protein 188 mg/dL A PCR test of the CSF is completed, but the results are pending. An MRI of the brain is performed, which shows abnormalities in the temporal lobes bilaterally. Which of the following is the most important factor in the development of this infection? A. Absence of antibodies in the mother B. Exposure of the mother to poorly cooked pork C. Passage through an infected birth canal D. Prematurity E. Transplacental transmission
answer
The correct answer is C. Viral causes of neonatal encephalitis include three members of the herpes family of viruses: herpes simplex-1, herpes simplex-2, and cytomegalovirus. All three types can have devastating effects on the neonate, with extensive CNS damage leading to cognitive impairment, seizures, and focal neurologic problems. Herpes simplex virus (HSV) typically involves the temporal lobes, while the other agents described do not have such a predilection. The most common means of infection of a neonate is by passage through the birth canal of a mother with active infection, therefore, a vaginal delivery would be contraindicated in this situation to prevent disease transmission.
question
A 16-year-old primigravid woman arrives to the emergency department in labor. Her amniotic sac ruptured prior to arrival. She has not received any prenatal care and was not aware that she was pregnant. Upon examination, her cervix is dilated, so she is admitted immediately to the maternity ward. The vaginal delivery was uneventful. Two days later, the neonate develops a fever and refuses to feed. Laboratory studies show leukocytosis. Examination of cerebrospinal fluid (CSF) shows a low glucose level and elevated protein. Gram stain of the CSF shows gram-positive cocci. Which of the following could have most likely prevented this newborn's illness? A. Avoidance of delicatessen foods by the mother B. Enema given to the mother prior to delivery C. Intravenous ampicillin during labor D. Treatment of the neonate's eyes with erythromycin E. Vaccination of the mother
answer
Answer: C. This question requires you to both determine the cause of the neonate's infection and to identify a strategy that would have prevented the infection. This neonate has bacterial meningitis, based on the characteristic CSF findings. The two most common causes of neonatal meningitis are Escherichia coli and Group B streptococcal (GBS; S. agalactiae). The Gram stain information (gram-positive cocci) indicates that GBS is the pathogen and not E. coli, which is a gram-negative rod. Both can be transmitted to the baby during delivery; GBS is acquired by the baby's passage through the birth canal since GBS colonizes the vagina of approximately 20% of women. The question specified that the mother did not receive any prenatal care, which would have included prescreening for GBS prior to delivery. In GBS-positive mothers, intravenous penicillin G should be started during labor.
question
A 42-year-old woman comes to the physician because of a headache and nausea. She underwent a heart transplant 14 months ago. Vital signs are normal. Physical examination does not show any focal neurologic deficits. A CT scan of the head is shown. Examination of tissue obtained on biopsy of the brain shows multiple cystlike structures. Cultures of the lesions are negative. Which of the following is the best explanation for the syndrome in this patient? A. Infected whole-blood transfusion B. Ingestion of cysts in poorly cooked beef C. Ingestion of tachyzoites from cat feces D. Reactivation of latent infection E. Transplant of an infected organ
answer
Answer: D. This patient is most likely infected with Toxoplasma gondii, which is a parasitic infection found throughout the world. In the United States, 15-20% of the population is infected, and once acquired, the parasites can persist for the lifetime of the host. Most primary infections in the U.S. are caused by ingestion of raw or undercooked pork and/or lamb. The persistent cysts provoke no symptoms in a normal individual; immunologic suppression (from cancer chemotherapy, transplant, or AIDS) will reinitiate proliferation and cause symptoms consistent with those described here. Because of the age of this patient, and her immunosuppression due to her prior transplant, this is most likely to be a reactivated infection, dormant since childhood. It should be noted that primary CNS lymphoma and brain abscess (among other conditions) can also produce ring-enhancing lesions, but none of the options was consistent with either of these diagnoses.
question
A 72-year-old man with a history of hypertension and hyperlipidemia is admitted to the hospital after suffering a stroke. He is unable to recognize his family, and he says that he has never seen his wife and son before, however his visual acuity is intact. He appears confused when they talk to him because their voices "sound just like those of his family members." A diffusion weighted-MRI performed later reveals ischemia in the distribution of the right posterior cerebral artery. Which of the following best characterizes this patient's deficit? A. Alexia B. Asomatognosia C. Confabulation D. Gerstmann syndrome E. Ideomotor apraxia F. Prosopagnosia G. Receptive aphasia
answer
The correct answer is F. Agnosia occurs when a patient has normal perception but the perception is devoid of meaning. Prosopagnosia literally means "the inability to recognize faces." Infarcts of the non-dominant (usually right) posterior cerebral artery (PCA) territory are often accompanied by this phenomenon. Patients with prosopagnosia can describe another person's face well and can identify that a face is a face, but they cannot recognize the face even if it belongs to someone that they know well. Patients may be able to recognize people based on other clues such as voice, clothing, or hairstyle. Prosopagnosia can also be the result of bilateral lesions of the fusiform gyri.
question
A 29-year-old woman comes to the physician for a follow-up examination because of increasing pain in her right eye. She had recently been diagnosed with severe optic neuritis in her right eye. Her past neurologic history is significant for weakness in her right leg 2 years ago that resolved spontaneously. She also reports some tingling in her left hand 5 years ago, which also resolved without treatment. The physician tests her pupillary response by shining a light in her right eye and in her left eye. Which of the following findings is most likely in this patient? Shine light in right eye (Right eye response/Left eye response) Shine light in left eye (Right eye response/left eye response) What disease does this patient have?
answer
Right eye - No response, no response Left eye - constrict, constrict This patient has multiple sclerosis, which is the most common autoimmune inflammatory demyelinating disease of the central nervous system. It is described as relapsing neurologic deficits with periods of remission- multiple lesions in time and space. Clinical features include optic neuritis, internuclear ophthalmoplegia (medial longitudinal fasciculus, MLF), motor and sensory deficits, vertigo, and neuropsychiatric symptoms. It is more common in women (onset 20-30 years of age) and there is a higher prevalence in temperate zones (away from the equator). MRI shows plaques (areas of demyelination), increased IgG (oligoclonal bands) in CSF from lumbar puncture, and it is associated with HLA-DR2. Treatment includes high-dose steroids and interferon-beta. This patient was diagnosed with optic neuritis in her right eye, which means she has vision loss because of demyelinating inflammation of the optic nerve (CN II). Patients typically present with acute, often painful vision loss, which is usually monocular. This question is essentially asking what the pupillary light reflex would look like if the patient has a lesion of her right CN II. (Note: CN II is the only cranial nerve myelinated by oligodendrocytes and, thus, susceptible is to MS. CN III is myelinated by Schwann cells and is not affected by MS.)
question
A 35-year-old woman comes to the physician because of "double vision" and the inability to adduct the right eye on attempted left lateral gaze. The left eye abducts appropriately on left lateral gaze. Neurologic examination shows normal direct and consensual light reflexes with intact convergence. Which of the following structures is most likely affected? A. Left oculomotor nerve B. Medial longitudinal fasciculus C. Right abducens nerve D. Right oculomotor nerve E. Right trochlear nerve
answer
The correct answer is B. This patient has an internuclear ophthalmoplegia (INO), which is caused by a lesion of the medial longitudinal fasciculus (MLF). The MLF connects the oculomotor (III), trochlear (IV), and abducens (VI) nuclei and is essential for conjugate gaze. A lesion in the MLF will result in the inability to adduct the ipsilateral eye on attempted lateral gaze. This patient has a right INO resulting from a right MLF lesion. This is frequently seen in MS
question
A 2-year-old boy is returned to the physician because of vomiting and ataxia. Physical examination shows papilledema and disproportionate growth of his head. An MRI of the head shows a large tumor involving the anterior horn of the right lateral ventricle. A diagnosis of choroid plexus carcinoma is made. Which of the following brain structures is most likely to be affected by direct extension of this tumor? A. Caudate nucleus B. Cerebellum C. Hippocampus D. Hypothalamus E. Pons
answer
The correct answer is A. Tumors of the ventricular system of the brain can affect the brain tissue either directly, via pressure on or invasion into a physically close structure, or indirectly, by obstructing CSF flow and causing hydrocephalus. Choroid plexus carcinomas consist of a solid mass with invasion into the periventricular brain parenchyma. The caudate nucleus is a C-shaped structure that comprises part of the wall of the lateral ventricle throughout its extent. The only structure listed that is adjacent to the anterior horn of the lateral ventricle and would therefore be directly affected by the large tumor described in the question is the caudate nucleus.
question
A 78-year-old woman with a history of atrial fibrillation is brought to the emergency department after falling down a flight of stairs. Current medications include warfarin. She appears grossly lethargic and is responsive only to painful stimuli. Her blood pressure is 187/99 mm Hg, and pulse is 45/min. A noncontrast CT scan is shown. A crescent shaped hematoma is seen along the the right side of the skull. Which of the following best accounts for the shape and bright appearance of this patient's lesion on CT? A. Accumulation of fibrin because of disruption of a cerebral artery B. Accumulation of fibrin because of disruption of a cerebral bridging vein C. Accumulation of fibrin because of disruption of a meningeal vessel D. Accumulation of hemoglobin because of disruption of a cerebral artery E. Accumulation of hemoglobin because of disruption of a cerebral bridging vein F. Accumulation of hemoglobin because of disruption of a meningeal vessel
answer
The correct answer is E. This patient most likely has a subdural hematoma, which is caused by tearing of cerebral bridging veins. This type of lesion is clearly demonstrated on CT because it extends into the median fissure along the falx without crossing the midline, is crescent-shaped, and extends across many suture lines. Thus, the lesion shape alone is diagnostic of a subdural hematoma, which eliminates choices A, C, D, and F. Choosing between the remaining options (choices B and E) tests your understanding of what makes structures hyperintense or "bright" on CT and x-ray. This understanding is very useful because you will see many images on the USMLE.
question
A 61-year-old woman comes to the physician for a follow-up examination. To view the neurologic examination, click on the "Play Media" button. She also says she has redness and dryness of her left eye. Which of the following additional findings is most likely present in this patient? (Media shows a woman with Bell's pulsy) A. Diplopia B. Hyperacusis C. Impairment of facial sensation D. Inability to chew E. Inability to shrug the shoulder
answer
The correct answer is B. This patient has unilateral (left) facial paralysis due to a lesion of the facial nerve (CN VII; Bell's palsy). Lower motor neuron lesions of CN VII affect the entire ipsilateral half of the face and upper motor neuron lesions cause contralateral symptoms that spare the forehead, as shown in the image below. Bell's palsy results in a complete ipsilateral paralysis of muscles of facial expression (causing an inability to wrinkle the forehead and shut the eye, and a drooping of the mouth). If the lesion affected the proximal facial nerve, additional findings would be hyperacusis (increased sensitivity to sound because of stapedius muscle paralysis), lack of taste sensation in the anterior two-thirds of the tongue (because of injury to the nervus intermedius fibers carried in the chorda tympani), and disturbed lacrimation and salivation (because of injury to the nervus intermedius fibers carried in the greater petrosal nerve).
question
A 19-year-old man is brought to the emergency department because of a motorcycle accident in which he sustained a closed head injury. After a transient loss of consciousness, witnesses at the scene of accident reported that the patient was conscious and aware of his surroundings. However, after presenting to the emergency department, the patient's condition rapidly declined with loss of consciousness. On physical examination, the patient is obtunded and has a dilated right pupil. An X-ray computed tomography scan is shown. It shows a lentiform shaped hematoma. The damaged artery most likely passes through which of the following? A. Carotid canal B. Foramen magnum C. Foramen spinosum D. Jugular foramen E. Superior orbital fissure
answer
The correct answer is C. The patient has an epidural hematoma. The middle meningeal artery is a branch of the maxillary artery, which enters the cranial cavity through the foramen spinosum.
question
A 54-year old woman is admitted to the intensive care unit because of sepsis secondary to a urinary tract infection. The decision is made to place a central line catheter in the lumen of the right internal jugular vein to better track hemodynamic information. A resident performs the procedure guided by ultrasound, but during the course of the procedure, inadvertently pierces through the vein and damages structures located posterior to the carotid sheath. Which of the following is most likely to occur? A. Constriction of the right pupil B. Dilation of the right pupil C. Inability to abduct the right eye D. Inability to close the right eye E. Paralysis of the platysma muscle on the right side
answer
The correct answer is A. The right sympathetic trunk lies posterior to the right carotid sheath, and may be injured in this procedure. Preganglionic sympathetic nerve fibers arising from the T1 spinal cord segment and ascending through the cervical sympathetic chain will be damaged. These nerve fibers synapse in the superior cervical ganglion on postganglionic sympathetic neurons that innervate structures in the head. The dilator pupillae muscle (smooth muscle of the iris that dilates the pupil) is sympathetically innervated; paralysis of this muscle due to interruption of its innervation results in constriction of the pupil (miosis).
question
A 79-year-old man comes to the physician because he has been "bumping into walls and objects," especially on his right side. He says that his symptoms began suddenly upon awakening last week and that wearing glasses did not help. He does not have vertigo or dizziness, and has had no loss of consciousness, lightheadedness, or tinnitus. Examination shows pupils are approximately 4 mm, equal, round, and react normally to light and accommodation. His extraocular movements are all intact. He has a corrected visual acuity of 20/20 in both eyes. However, visual fields are reduced on the right side in both eyes. Occlusion of which of the following blood vessels is most likely responsible for this patient's symptoms? A. Anterior communicating artery B. Internal carotid artery C. Middle cerebral artery D. Ophthalmic artery E. Posterior cerebral artery F. Posterior communicating artery G. Vertebral artery
answer
The correct answer is E. This patient has most likely undergone a left posterior cerebral artery (PCA) stroke, resulting in a right homonymous hemianopsia. Extensive visual field testing will show macular sparing. Since many individuals associate stroke with motor weakness or language loss, they may often avoid seeking immediate medical attention after experiencing only visual changes. The PCA supplies the occipital cortex, where the primary visual cortex is located. Infarction of this region in one hemisphere will produce a homonymous hemianopsia. However, unlike injury to the lateral geniculate nucleus or the optic radiations, the blood supply in the most posterior region of the occipital cortex, where the macula is represented, is bilaterally redundant. As a result, occipital lobe strokes often spare the macula, and hence visual acuity is preserved, as in this patient.
question
An 83-year-old man develops a slow, painless loss of vision of his left eye. When the patient views an Amsler chart, the grid seems distorted and there are missing areas centrally. The peripheral visual fields are normal. Funduscopy shows pigmentary changes in the maculae of both eyes but more prominently in the left. Which of the following is the most likely diagnosis? A. Age-related macular degeneration B. Central retinal artery occlusion C. Chronic open angle glaucoma D. Proliferative diabetic retinopathy E. Retinal detachment
answer
The correct answer is A. This patient has age-related macular degeneration (AMD). These changes can cause either slow or sudden loss of central visual fields. The peripheral fields are preserved, as is some useful color vision. As a result, even patients who are "legally blind" often have enough residual vision to perform many activities of daily living, even if they cannot read. The Amsler chart is a finely squared grid that is viewed from a distance of one foot, and can be very helpful in identifying areas of visual loss.
question
A 32-year-old woman comes to the physician because of headaches and a seizure. She states that she delivered a full-term girl two weeks ago. Physical examination shows bilateral papilledema. An MRI of the brain in a normal individual is shown. The arrow in the normal scan indicates where a thrombus formed in this patient (it's the central sulcus of the occipital lobe). The most likely cause of this patient's symptoms is occlusion of which of the following structures? A. Anterior cerebral artery B. Cavernous sinus C. Middle cerebral artery D. Middle meningeal artery E. Posterior cerebral artery F. Superior sagittal sinus
answer
The correct answer is F. This patient has a sagittal sinus thrombosis, which is often associated with hypercoagulable states. It occurs with increased frequency during pregnancy, and during the first few weeks postpartum. Obstruction of the venous drainage can lead to increased intracranial pressure, seizures, parasagittal hemorrhages (due to back-pressure in the cortical veins), and infarcts from decreased cerebral perfusion (due to increased venous pressure). It is an uncommon cause of cerebral infarction compared to arterial disease, but has significant morbidity. Treatment usually involves anticoagulation therapy or surgical intervention is also performed.
question
A 54-year-old man who is a musician comes to the physician because of a 2-year history of progressive hearing loss. During his career, he played with a rock band and stood in front of loud amplifiers. The most likely cause of this patient's hearing loss is damage to which of the following structures? A. Ampulla B. Basilar membrane C. Organ of Corti D. Scala tympani E. Scala vestibuli
answer
The correct answer is C. Sensorineural hearing loss can occur by damage to the hair cells of the organ of Corti. These cells can be damaged by trauma from very loud sounds or ototoxic drugs.
question
A 9-year-old boy is brought to the physician because of pain and swelling of the left eyelid. His mother attributes his symptoms to a recent spider bite. Physical examination shows periorbital redness, tenderness, conjunctival injection, and edema that prevents him from voluntarily opening his eye. Visual acuity and extraocular eye movements are normal. Culture of the bite site reveals gram-positive cocci in clusters. Treatment with antibiotics is begun immediately to prevent spread of infection to which of the following areas? A. Cavernous sinus B. Occipital sinus C. Sigmoid sinus D. Straight sinus E. Superior petrosal sinus
answer
The correct answer is A. This patient likely has preseptal cellulitis (in soft tissues anterior to the orbital septum), which can progress to orbital cellulitis if untreated. In preseptal cellulitis, the eye itself is not involved. If a preseptal infection progresses into the orbit, proptosis can develop and extraocular motility can become compromised. Visual acuity changes can occur with involvement of the optic nerve. Potential complications of orbital cellulitis include cavernous sinus thrombosis, blindness, intracranial brain abscess, and death. The anterior continuation of the cavernous sinus, the superior ophthalmic vein, passes through the superior orbital fissure to enter the orbit.
question
A 65-year-old man with a history of hypertension, hyperlipidemia, and myocardial infarction is brought to the emergency department after tripping down the stairs this morning. He reports some disturbances in his vision after waking up this morning that were not present previously. Visual field testing reveals a right lower quadrantanopia. A CT scan of the head is most likely to show a lesion of which of the following? A. Optic nerve B. Optic tract C. Left lateral geniculate nucleus D. Left parietal lobe E. Left temporal lobe
answer
The correct answer is D. A right lower quadrantanopia is produced by a left parietal lobe lesion.
question
A 63-year-old man with a history of hypertension is brought to the emergency department by his wife after briefly slumping to the floor while they were on an errand together. His wife said that her husband said he had tingling in his left hand the week before. She also said that he has been having trouble getting dressed in the morning, and that he only shaves the right side of his face. The patient is irritable and insists that nothing is wrong. Neurologic examination shows no blink to threat reflex on the left and a right gaze preference. Injury to which of the following structures is the most likely cause of this patient's symptoms? A. Left midbrain B. Left parietal cortex C. Left thalamus D. Right midbrain E. Right parietal cortex F. Right thalamus
answer
The correct answer is E. This is an example of a hemineglect syndrome, in this case involving the left side (the patient is still taking care of the right side). This is the most common pattern in right-handed patients (who have the highest percentage of language dominance on the left), and is usually due to damage to the right parietal or frontal cortex. These patients show profound neglect for the contralateral half of their external world and the contralateral half of their own body. Patients with hemineglect syndrome often have a history of bumping into objects on one side, ignoring food on one half of their plate, or grooming only half of their body. They are often unaware of their illness (anosognosia). Patients can even be unaware that the left half of their body belongs to them, and for example, can become distressed that someone left an arm in their bed, unaware that it is their own limb (hemiasomatognosia). Nondominant parietal lobe (usually right) lesions more commonly produce these abnormalities in spatial orientation and awareness than a dominant parietal lobe (usually left, choice B) lesion.
question
A 49-year-old woman is brought to the emergency department 20 minutes after sustaining a closed head injury in a motor vehicle collision. A CT scan of the head shows a small tumor at the cerebellopontine angle of the brain but does not show any intracranial hemorrhage. Which of the following nerves is most likely to be affected by this tumor? A. Facial nerve B. Glossopharyngeal nerve C. Optic nerve D. Trigeminal nerve E. Vagus nerve
answer
The correct answer is A. The cerebellopontine angle is a triangular area formed by the cerebellum, upper medulla, and middle cerebellar peduncle. The facial nerve and the vestibulocochlear nerve emerge from the brain stem at the cerebellopontine angle. These are the two nerves that will be initially affected by a tumor in this region. The most common tumors in this area are acoustic schwannomas (neuromas) (80%); the remaining 20% are primarily meningiomas.
question
A 23-year-old man who is a boxer is brought to the emergency department after sustaining a blow to his right eye during a match. He complains of severe pain and excessive bleeding from his nose. Physical examination reveals restricted movement of his eye, a swollen eye lid, periorbital bruising, and some loss of sensation over his right cheek. Pain medications are started and a CT scan of the head is obtained. Imaging reveals a blow-out fracture of the orbital floor. Damage to which of the following structures is the most likely cause of this patient's symptoms? A. Ethmoid air cells B. Maxillary sinus C. Oral cavity D. Pterygopalatine fossa E. Sphenoid sinus
answer
The correct answer is B. The floor of the orbit is also the roof of the maxillary sinus. The bone separating the orbit from the maxillary sinus is frequently quite thin and may fracture from increased intraorbital pressure caused by a direct blow to the eye. The subsequent bleeding into the maxillary sinus will result in blood draining from the sinus into the nasal cavity.
question
A 62-year-old woman is brought to the emergency department after falling down a flight of stairs. She says that she has had increasingly blurry vision and was planning to make an appointment with the eye doctor. She has a 9-month history of pulmonary Mycobacterium avium intracellulare (MAI). She was initially treated with azithromycin, rifampin, and ethambutol. Rifampin was discontinued during the first month after the patient developed a rash. She has a 12-year history of diabetes mellitus, a 5-year history of Addison disease, and a 3-year history of hypothyroidism. Recent HIV and PPD testing was negative. Visual field testing shows bilateral central scotomas. A lesion involving which of the following structures is the most likely cause of this defect? A. Macula B. Occipital lobe C. Optic chiasm D. Optic radiations in the parietal lobe E. Optic radiations in the temporal lobe F. Optic tract
answer
The correct answer is A. The most likely locations of lesions that produce visual defects are a favorite USMLE topic. Lesions of the retina that include destruction of the macula produce a central scotoma.
question
A 29-year-old woman comes to the physician because of a headache that began several days ago and has become progressively worse. A single-photon emission computed tomography (SPECT) image using technetium (Tc)-labeled red blood cells is shown. Greater blood flow is indicated by orange to red colors and poor blood is indicated by blue or lack of signal. There is an area of great perfusion from the central sulcus, wrapping around the right portion of the occipital lobe towards the temporal lobe. Which of the following structures shows an abnormal blood profusion pattern according to this imaging study? A. Inferior sagittal sinus B. Internal jugular vein C. Middle cerebral artery D. Posterior cerebral artery E. Superior sagittal sinus F. Transverse sinus
answer
The correct answer is F. There is a marked signal asymmetry in this image. The right transverse sinus (shown on the left side of the image) has markedly less signal than the left transverse sinus, revealing lack of blood flow in this region. The transverse sinus receives blood from the superior sagittal sinus, occipital sinus, and the straight sinus at the confluence of sinuses. It transports the blood along the posterior skull to the sigmoid sinus.
question
A 58-year-old man is brought to his general physician by family members because of recent cognitive changes. His family is concerned that he is no longer able to recognize common objects in his surroundings. Neurologic examination shows that when he is presented with a picture of a dog, he is able to copy the picture but is unable to recognize the animal as a dog. This finding is most likely caused by a lesion to which of the following areas? A. Angular gyrus B. Frontal eye field C. Parieto-occipital association cortex D. Precentral gyrus E. Prefrontal cortex F. Temporo-occipital association cortex
answer
The correct answer is F. This patient has agnosia, which is the inability to recognize, despite adequate sensation. This patient has a type of agnosia known as visual agnosia, which is the inability to recognize familiar objects despite the ability to see. The fact that this man can copy a picture of the dog indicates that he has adequate visual acuity, visual fields, and perception. If this patient had only visual agnosia and were to hear a dog bark (auditory) or to pet a dog (tactile), then he would be able to recognize and identify the dog as a dog. Prosopagnosia is a special type of visual agnosia in which a person is unable to recognize faces. There are also other types of agnosias, including auditory and tactile agnosias.
question
A 3-year-old boy is brought to the emergency department because of persistent vomiting, a stiff neck, and decreased responsiveness. An MRI scan of the head shows a posterior fossa tumor filling the 4th ventricle. Which of the following physical findings is most likely present in this patient? A. Bilateral papilledema B. Loss of peripheral vision C. Muscular weakness D. Photophobia E. Resting tremor
answer
The correct answer is A. Any tumor "filling the 4th ventricle" blocks the circulation of cerebrospinal fluid (CSF), and can cause a noncommunicating hydrocephalus. This blockage leads to increased intracranial pressure, which manifests with nausea, vomiting, bilateral papilledema, headache, nuchal rigidity, and mental status changes. Opening pressure would be elevated on lumbar puncture, and in fact, lumbar puncture should be avoided in patients suspected of having increased intracranial pressure (e.g., who have papilledema) because it might cause herniation of the brain stem and death. In children, medulloblastoma and ependymoma are the most frequent neoplasms presenting in this manner. Treatment is surgical to prevent sequelae of increased intracranial pressure.
question
A 50-year-old woman comes to the physician because of headache, nausea, and vomiting. She does not drink alcohol or smoke cigarettes. She does not take any medications. Neurologic examination shows that her pupils have a minimal reaction to light, but do constrict during accommodation. She is unable to look upward past the horizontal plane, but has full conjugate eye movements in other directions; she is unable to converge. Visual field testing shows no abnormalities. Which of the following is the most likely diagnosis? A. Acoustic neuroma B. Astrocytoma in the cerebellum C. Craniopharyngioma D. Parasagittal meningioma E. Pinealoma F. Tabes dorsalis
answer
The correct answer is E. Based on the neurological examination, this patient has a lesion of the vertical gaze center in the dorsal midbrain. Of the answer choices, a tumor of the pineal gland (e.g., germinoma) is most likely, because the close proximity of the pineal gland to the midbrain tectum.
question
A 2-year-old girl is brought to the physician because she bumps into things on her right side. Physical examination shows a whitish hue in the right pupil and strabismus. Palpation of the eye causes pain. She undergoes surgical removal of the right eye. Gross examination of the enucleation specimen reveals multifocal endophytic tumor within the chamber, located adjacent to the optic nerve. Histologically, the tumor contains rosettes composed of cuboidal-to-columnar cells surrounding a central lumen. Which of the following chromosomes is the most likely source of this patient's pathology? A. Chromosome 5 B. Chromosome 7 C. Chromosome 9 D. Chromosome 13 E. Chromosome 21
answer
The correct answer is D. The description of this case is classic for retinoblastoma, the most common malignant eye tumor of childhood. Retinoblastoma can be inherited as an autosomal dominant condition or occur sporadically. The most common finding in retinoblastoma is leukocoria (white pupillary reflex instead of the normal red reflex); however, there are other causes of leukocoria including congenital glaucoma and corneal scarring. Other findings in this disease include strabismus, proptosis, and eye pain. Microscopically, retinoblastoma is composed of primitive-appearing blue (neuroepithelial) cells that form characteristic rosettes. The tumor can "seed" within the ocular chamber, giving rise to multiple lesions. Tumor can also directly invade and extend into the optic nerve, which progresses to a metastatic stage and has a high risk of mortality. Retinoblastoma results from mutations in the tumor suppressor gene, Rb, located on chromosome 13q. In the familial form of the disease, a single Rb mutation in inherited from a parent, which by itself, causes no symptoms. However, a second acquired mutation of Rb in a retinal cell results in loss of the tumor suppressor protein (Knudson's two-hit hypothesis). Rb protein normally regulates progression of the cell-cycle from G1, thus loss results in constitutive cell-cycle activation and tumor formation. In the inherited form of the disease, multifocal and bilateral retinoblastomas may occur. In contrast, in the sporadic form of retinoblastoma, two independent Rb mutational events must occur, which is a much rarer event. As a result, sporadic retinoblastomas are almost always unifocal and unilateral. Our patient has multiple tumors, but a unilateral eye involvement. At this point, it is not determined whether this is a familial or sporadic form. Another important point is that patients with familial retinoblastoma are at risk for other types of malignancies later on, particularly osteosarcoma.
question
A 45-year-old man comes to the physician because of recurring headaches for the past month. He says that the headaches are worse upon wakening. Neurologic examination shows weakness of his right leg. A CT scan shows a large meningioma that is producing a subfalcine herniation. Which of the following structures is most susceptible to damage due to the herniation? A. Cerebellar tonsils B. Cingulate gyrus C. Medial temporal lobe D. Medulla E. Midbrain
answer
The correct answer is B. Brain herniation can occur either with generalized increased intracranial pressure or as a consequence of a mass lesion of the cranium. Subfalcine (cingulate) herniation occurs when part of one cerebral hemisphere herniates under the falx cerebri, which is a portion of dura that separates the two cerebral hemispheres along the midline of the skull. A subfalcine herniation produces a midline shift, which occurs when the herniated tissue causes displacement of the falx cerebri laterally. The gyrus most likely to be affected is the cingulate gyrus, which runs along the medial aspect of the cerebral hemisphere, just above the corpus callosum.
question
A 60-year-old woman dies from a myocardial infarction. Her past medical history is significant for intermittent episodes of transient blurry vision in her 30s, which subsequently resolved without treatment. She also had multiple episodes of tingling in both arms and weakness in her left leg in her 40s; these symptoms resolved over a period of a few months. Which of the following is the most likely mechanism for her neurologic findings? A. Autoimmune B. Hereditary C. Neoplastic D. Prion-related E. Viral-related
answer
The correct answer is A. The disease illustrated in this case is multiple sclerosis (MS). The most important clues include the unmistakable appearance of well-demarcated areas of demyelination of the white matter (acquiring a pinkish gray color similar to the cortex) and the remitting-relapsing clinical course. The photograph shows demyelination (plaques) in the corpus callosum (one of the largest white matter tracts) and brainstem, which is readily distinguishable on this mid-sagittal section.
question
A 72-year-old man with a history of atrial fibrillation and diabetes mellitus is brought to the emergency department by his family. His son says that when the patient awoke that morning, he was unable to use words or sentences that made any sense. Physical examination shows the patient has spontaneous, fluent speech. However, his language is confused and without meaning. He is unable to follow commands, except for closing his eyes. He substitutes words inappropriately when asked to identify objects. He is unable to read and cannot repeat words when asked. The image shows a CT scan of the brain. Damage to which of the following structures is the most likely underlying cause of these findings? A. Arcuate fasciculus B. Inferior frontal gyrus C. Meyer's loop (optic radiation) D. Postcentral gyrus E. Precentral gyrus F. Prefrontal cortex G. Superior temporal gyrus
answer
The correct answer is G. This patient has Wernicke aphasia, which is caused by a lesion of Wernicke's area. This area is located in the posterior part of the superior temporal gyrus of the language-dominant hemisphere. This conclusion can be reached by the patient's description, and also by the location of the lesion on CT. In the CT, the lesion extends nearly from the occipital pole to the Sylvian fissure, thus encompassing the temporal lobe (in which Wernicke's area resides). This area's blood supply comes from the inferior division of the middle cerebral artery. Atrial fibrillation and diabetes are both important risk factors for stroke, which this patient most likely had. The lesion above is in the left brain (which is shown on the right side on CT), which is the typical side for the language centers in a right-handed individual.
question
A 66-year-old man with a history of hypertension is brought to the emergency department by his family after he is noticed to have behavioral changes. To view the examination, click the "Play Media" button. Neurologic examination shows that he is unable to repeat phrases spoken to him. Which of the following findings is most likely in this patient? Media shows a person does not understand speech but keeps talking nonsense. A. Left facial hemiparesis B. Left hemineglect C. Preserved ability to read and write D. Right Babinski sign E. Right upper quadrantanopia
answer
The correct answer is E. The first challenge of this question is to recognize that the patient has Wernicke (receptive) aphasia. The patient produces speech that is fluent but nonsensical, with word sequences haphazardly strung together. The phenomenon of "word salad" arises from an impairment in language comprehension, leading to production of meaningless language, due to damage to Wernicke area. These patients are usually unaware of their disability. Repetition is also impaired. Reading and writing are impaired in a similar fashion as the speech deficits in this condition. The second task is to realize that Wernicke's area is located in the temporal lobe, in the posterior part of the superior temporal gyrus of the dominant (usually left) hemisphere. The third piece to the puzzle is to know what else is in the temporal lobe that could be damaged along with Wernicke's area. Since Wernicke's area often contains a portion of the optic radiation fibers from the left lateral geniculate nucleus projecting to the left primary visual cortex in the occipital lobe (Meyer's loop), Wernicke aphasics commonly present with a right upper visual field cut (right upper quadrantanopia). The figure below shows how a lesion of the optic radiations in the temporal lobe (Meyer's loop; lesion #5) causes a right homonymous superior quadrantanopia ("pie in the sky").
question
A female newborn is delivered at 28 weeks' gestation with a birth weight of 1,200 g. Immediately following delivery, the newborn demonstrates signs of hypoxia and respiratory distress. She is ventilated with an FiO2 of 100% and started on surfactant replacement therapy 60 minutes after delivery. Three weeks later, retinal examination shows a white pupillary reflex. Which of the following is the most likely underlying cause of this finding? A. Accumulation of ganglioside in the macula B. Ganglion cell degeneration C. Inappropriate vascular proliferation D. Optic nerve degeneration E. Pigmented epithelium degeneration with drusen deposits
answer
The correct answer is C. This child has severe retinopathy of prematurity (formerly known as retrolental fibroplasia). The underlying lesion in this disorder is an inappropriate proliferation of vessels in the inner layers of the retina. Major risk factors include prematurity and intensive oxygen therapy (FiO2 of 100%), although other problems such as hypoxia, infection, and acidosis have been implicated. This premature newborn presents with respiratory distress syndrome resulting from inadequate surfactant in the lungs, requiring supportive oxygen therapy. Unfortunately, high levels of oxygen can result in vasoconstriction of retinal vessels. The resulting ischemia stimulates vascular endothelial growth factor (VEGF) production and increases blood vessel proliferation as a compensatory mechanism. The result of this proliferation ranges from mild, usually transient, changes of the peripheral retina to severe progressive vascular proliferation, scarring (producing a white pupillary reflex), and potentially blinding retinal detachment. If the process is disrupted early, the retina may heal with little residual damage. Importantly, the "white pupillary reflex" can reflect a number of conditions including retinoblastoma (classically) and cataracts. However, in this patient, it is related to retinal vascular hyperproliferation and scarring.
question
A 45-year-old woman with progressive myopia develops sudden patchy loss of vision in her right eye. Funduscopic examination shows a large retinal detachment in the right eye. The retina in the left eye is normal. The physician tests the pupillary light reflex by swinging a light source from the left eye to the right eye. Which of the following will most likely occur after the light source reaches the right eye? A. Constriction of the right pupil and constriction of the left B. Constriction of the right pupil and dilation of the left C. Dilation of the right pupil and constriction of the left D. Dilation of the right pupil and dilation of the left E. No reaction of the right pupil and constriction of the left
answer
The correct answer is D. This patient is exhibiting a Marcus-Gunn pupil (relative afferent pupillary defect). This defect is caused by decreased sensitivity of the affected eye to light, which results from lesions of the retina, optic nerve, or the eye. A useful way to detect this defect is a swinging flashlight test, in which a flashlight is moved back and forth between both eyes every 2 to 3 seconds. The afferent pupil becomes obvious when the flashlight is moved from the normal to the affected eye, and the eyes dilate in response to light.
question
A 65-year-old woman comes to the physician because of difficulty reading and blurred vision over the past 9 months. She says that when she rides in a car at night, the oncoming headlights cause more glare than they did previously. Ophthalmologic examination shows bilateral white opacifications. Which of the following is the most likely location of the disease process? A. Aqueous humor B. Cornea C. Lens D. Optic nerve E. Retina
answer
The correct answer is C. This patient has classic signs and symptoms of cataracts (lens opacifications). It is not known whether senile cataracts represent disease or normal opacification with age. Cataracts may occur as a consequence of diabetes mellitus, long-term steroid use, or congenital infections. Currently, they are successfully treated with lens extractions and implantation of prosthetic lenses.
question
A 42-year-old man comes to the physician because of a bulging mass in his left lower abdomen. His temperature is 37.2°C (98.9°F), pulse is 93/min, blood pressure is 152/82 mm Hg, and respirations are 18/min. A diagnosis of inguinal hernia is made and surgical repair is planned. A review of his records shows that he had an allergic reaction to bupivacaine 1 year ago. Which of the following anesthetics should be avoided in this patient? A. Benzocaine B. Cocaine C. Lidocaine D. Procaine E. Tetracaine
answer
The correct answer is C. The local anesthetics act by blocking voltage-gated sodium channels, thus preventing sensory information from being transmitted from the periphery to the brain. There are two primary classes of anesthetics: amides and esters. Bupivacaine is an amide-type local anesthetic, and allergic reactions to local anesthetics tend to be only within groups (ester-type cross-reacts with ester-type and amide-type cross-reacts with amide-type). The only example in the answer choices of an amide-type local anesthetic, which would be metabolized by hepatic amidases, is lidocaine. Other members of this group include mepivacaine, etidocaine, and prilocaine. An easy way to distinguish between the amides and esters is that amides have two "I's" and esters have only one "I" in the spelling of their names.
question
A 45-year-old man is brought to the emergency department 20 minutes after being involved in a motor vehicle collision. The patient has a pneumothorax, several rib fractures, and an orbital fracture. His temperature is 37.5ºC (99.5ºF), pulse is 106/min, respirations are 14/min (ventilated mechanically), and blood pressure is 105/75 mm Hg. He is administered vecuronium when intubated and admitted to the intensive care unit for several days. The effects of vecuronium are reversed to examine the patient's neurologic status. Which of the following will be most effective in reversing the paralysis caused by vecuronium? A. Atracurium B. Dantrolene C. Neostigmine D. Succinylcholine E. Tubocurarine
answer
The correct answer is C. When patients are intubated, different medications are generally needed to produce the desired pharmacologic effects. These effects include paralysis (with neuromuscular blockers such as vecuronium), anesthesia, sedation, analgesia, amnesia and anticholinergic effects to control secretions. Vecuronium is a nondepolarizing (competitive) neuromuscular blocking agent. Unlike other neuromuscular blocking agents, vecuronium lacks both significant cardiovascular effects and dependence on good kidney function for elimination. It has a short duration of action and easy reversibility, which provides several advantages over other neuromuscular blockers. Vecuronium competitively antagonizes the nicotinic acetylcholine (NM) receptors at the neuromuscular junction, and thus reversal is accomplished by administration of acetylcholinesterase inhibitors (e.g., neostigmine, edrophonium, and pyridostigmine), which increase acetylcholine (ACh) levels.
question
A 48-year-old woman comes to the physician because she wants to have a mole removed. The patient says that she had a reaction to a local anesthetic the last time she had dental work performed. When the anesthetic was injected she started to feel flushed and found it difficult to breathe. She received an epinephrine shot and was taken to the emergency department for monitoring. A review of her dental records shows that she received procaine for a tooth extraction. Which of the following drugs is most appropriate for this procedure? A. Benzocaine B. Chloroprocaine C. Cocaine D. Mepivacaine E. Tetracaine
answer
The correct answer is D. This patient had an allergic reaction to the local anesthetic. There are two classes of local anesthetics: esters and amides. The rule of thumb is that if you are allergic to one drug in a given class (usually the ester class), you also will be allergic to other drugs of the same class. The proper course of action would be to switch over to the other drug class. In this question, the patient received procaine, which is an ester. Therefore, you need to identify the amide in the list of answers. The only amide listed is mepivacaine. Other amide local anesthetics include lidocaine, bupivacaine, etidocaine, prilocaine, and ropivacaine. An easy way to distinguish between the amides and esters are as follows: the amides have two "i's" in their names and the esters have either one or no "i's" in their name.
question
A 27-year-old man with a history of paranoid schizophrenia is being treated with an antipsychotic drug. He comes to the physician because of a painful, involuntary twisting of his neck, a feeling of inner restlessness, tremor, and a difficulties initiating movement. His physician starts him on benztropine. Which of the following is the most likely side effect from this medication? A. Bradycardia B. Bronchoconstriction C. Lacrimation D. Miosis E. Xerostomia
answer
The correct answer is E. Extrapyramidal symptoms (EPS) commonly occur in individuals taking dopamine antagonists and include acute dystonic reactions (sustained muscle contractions), pseudoparkinsonism (Parkinson-like symptoms), and akathisia (restlessness). Muscarinic antagonists such as benztropine and trihexyphenidyl are indicated as an adjunct therapy for the treatment of drug-induced EPS. Although these anticholinergic agents can effectively treat and/or prevent the appearance of EPS, they are also commonly associated with a number of unpleasant side effects. A mnemonic to help remember the action of antimuscarinic agents is that they have anti-DUMBBelLSS properties. DUMBBelLSS: D iarrhea, Urination, Miosis, Bronchospasm, Bradycardia, Excitation of CNS and skeletal muscle (this is mediated by nicotinic receptors), Lacrimation, Sweating, and Salivation.
question
A 62-year-old man is brought to the emergency department because of the acute onset of severe ocular pain and blurred vision that is associated with halos around lights. The painful episode began when he walked into a dark room. Ophthalmologic examination shows erythema of the left eye. The eye is firm to palpation, and the cornea appears hazy, and the pupil is dilated. The patient is subsequently treated with traditional topical medications and symptoms persist. Which of the following drugs is most appropriate for this patient? A. Acetazolamide B. Epinephrine C. Latanoprost D. Scopolamine E. Tropicamide
answer
The correct answer is A. The patient is clearly presenting with signs and symptoms of acute angle-closure glaucoma. Primary acute angle-closure glaucoma occurs because of closure of a preexisting narrow anterior chamber angle, as is commonly found in the elderly, hyperopes, and Asians. Symptoms can be triggered when a patient's pupils dilate, for example when entering a dark room, or if he or she takes a drug with anticholinergic side effects, such as scopolamine (choice D) or tropicamide (choice E). Patients often seek immediate medical attention because of the intense pain and blurred vision. The blurred vision is characteristically associated with halos around lights. The eye is often very red and steamy, and the pupil is dilated and nonreactive to light; tonometry reveals elevated intraocular pressure. Acetazolamide, a carbonic anhydrase inhibitor, is a first-line agent that is commonly used systemically after traditional topical agents have failed to reduce IOP. It reduces bicarbonate production in the ciliary epithelium, thus decreasing aqueous humor production. Mannitol is another agent used systemically to decrease IOP. It is a hyperosmotic agent that causes a fluid shift from the eye into the vascular space. Topical timolol (beta-blocker), apraclonidine (alpha-2 agonist), pilocarpine (muscarinic agonist) and prednisolone (corticosteroid) are generally used first to decrease IOP and then acetazolamide is administered when symptoms persist. Patients are maintained on medication until a more definitive laser peripheral iridotomy can be performed.
question
24-year-old man is brought to the emergency department because of paranoid delusions and auditory hallucinations. Treatment with a drug is begun, but he develops involuntary contractions of his neck muscles and focal neck pain and is frequently observed with his head displaced to the right side. Which of the following drugs is most likely being administered? A. Amitriptyline B. Fluphenazine C. Lithium D. Olanzapine E. Risperidone F. Trazodone
answer
The correct answer is B. There are two primary classes of antipsychotic mediations: typical and atypical. In general, the typical class, there are 2 subclasses: high potency and low potency. General rules: Typical antipsychotics have more extrapyramidal side effects (EPS) than atypicals. High potency typical antipsychotics (e.g., haloperidol, fluphenazine) have more EPS and less non-neurologic side effects (e.g., from muscarinic, alpha-adrenergic, or histamine blockade). Low potency typical antipsychotics (e.g., chlorpromazine, thioridazine) have less EPS and more non-neurologic side effects (dry mouth and constipation- muscarinic blockade; orthostatic hypotension- alpha blockade; sedation- histamine receptor blockade). The reason for the difference in side effects with potency is because if the drug is highly potent, a smaller amount of drug will work and will be targeted more to blocking dopamine receptors (responsible for EPS). If a drug is less potent, you need to give more of it and it will start binding other receptors (e.g., muscarinic, α-adrenergic, histamine). This will help with the EPS, but cause the additional autonomic and sedation side effects.
question
A 19-year-old man is brought to the emergency department by police officials because his mother called 911. His mother says that the patient was shouting "They are after me" and "I am going to kill them all" earlier that day. She also says that his behavior has been erratic for the past 1 year. He has been talking to persons who were not there, becoming very agitated, and finally threatening his mother's life. In the emergency department, he began pleading with the hospital staff to let him leave because everyone including the nurses is in imminent danger with him being there. Which of the following is the most likely diagnosis? A. Bipolar I disorder B. Delusional disorder C. Paranoid personality disorder D. Schizoaffective disorder E. Schizophrenia
answer
The correct answer is E. Schizophrenia is a disease of young adults, with the initial onset usually in the mid-to-late teens. It is characterized by bizarre delusions, such as delusion of persecution, auditory hallucinations, blunted affect, loose associations, and disturbances in behavior and thought. This young man has schizophrenia, as defined by the presence of hallucinations, which rules out a personality disorder (choice C).
question
A 22-year-old woman who is a college student is brought to the emergency department by police because she was found talking to herself while walking back and forth across a busy street. She appears to be oriented to person, place, and time. Her sister comes to the hospital and says that the patient dropped out of school 3 months ago and has been living on the streets. A review of her records shows a hospital admission 2 months ago for a similar condition. Psychiatric examination shows difficulty concentrating and talking to unheard voices. Urine toxicology is negative. Which of the following is the most likely diagnosis? A. Brief psychotic disorder B. Schizoaffective disorder C. Schizoid personality disorder D. Schizophrenia E. Schizophreniform disorder F. Schizotypal personality disorder
answer
The correct answer is E. This patient is suffering from schizophreniform disorder. There has been a marked decline in the level of functioning, and she was endangering herself in the middle of the street. Schizophreniform disorder is characterized by schizophrenia-like symptoms, but the duration of symptoms is less than 6 months (but more than 1 month). Fully developed psychotic symptoms are typical.
question
[4] (Chang, hypothalamus) A study is conducted to determine the role of the hypothalamus in controlling pituitary function. As a first step, pituitary stalks in healthy animal models are ligated to interrupt hypophyseal portal blood flow. Two weeks later, serum levels of several hormones are measured. Increased circulating concentrations of which of the following substances is most likely to be seen? A. Cortisol B. Follicle-stimulating hormone C. Growth hormone D. Oxytocin E. Prolactin F. Thyroid hormone
answer
The answer is E. Prolactin production is under negative control by the prolactin-inhibiting factor (i.e., dopamine), produced in the hypothalamus and transported to anterior pituitary lactotroph cells. Thus, when the blood supply from the hypothalamus to the pituitary is interrupted, the production of prolactin is increased.
question
A 32-year-old woman comes to the physician because of a 3-month history of amenorrhea. She also states that there is a "watery secretion" coming from both her nipples. Her temperature is 37.0°C (98.6°F), pulse is 73/min, blood pressure is 125/74 mm Hg, and respirations are 14/min. Breast examination shows a nonpurulent, watery fluid dripping from both nipples. Physical examination is otherwise unremarkable. Examination of the fluid does not show white or red blood cells. Laboratory studies show prolactin levels are 77 ng/mL (normal for nonpregnant women 4-23). Further eliciting of her history reveals she started taking a new medication within the past 6 months. Which of the following drugs is most likely responsible for the findings in this patient? A. Alprazolam B. Captopril C. Clevidipine D. Propranolol E. Risperidone F. Sertraline
answer
The answer is E. Hyperprolactinemia in premenopausal women can cause oligomenorrhea or amenorrhea, infertility, and galactorrhea. A detailed drug history should first be obtained because many commonly used medications can cause hyperprolactinemia. Dopamine inhibits prolactin release via D2 receptors on lactotrophs. Therefore, drugs that block D2 receptors can cause hyperprolactinemia. The best way to approach this question is to look for the drug most likely to block dopamine receptors.
question
An investigator is evaluating the function of the hypothalamic nuclei by ablating different parts of an experimental animal's hypothalamus and then monitoring the animal's behavior. In one experiment, after ablation, the animal begins to eat more food and becomes obese over a period of 2 weeks. Which of the following structures was most likely damaged in this experimental animal? A. Lateral nucleus B. Septal nucleus C. Suprachiasmatic nucleus D. Supraoptic nucleus E. Ventromedial nucleus
answer
The correct answer is E. The ventromedial nucleus (VMN) is thought to be the satiety center. Bilateral destruction leads to hyperphagia, obesity, and savage behavior. Stimulation inhibits the urge to eat.
question
An 87-year-old woman is brought to the emergency department after she is found unconscious inside her home. Paramedics state that the oven was on, but the pilot light had gone out and the apartment was filled with natural gas. The woman is resuscitated, and examined by a neurologist. Examination shows no abnormalities other than a deficient sense of smell. A CT scan of the head shows a mass in the olfactory groove area. The axons likely compressed by this mass project to which of the following structures? A. Insula B. Nucleus ambiguus C. Postcentral gyrus D. Precentral gyrus E. Piriform cortex
answer
The correct answer is E. The olfactory nerves can be damaged by head injury, severe infection, and tumors such as meningiomas (which this patient has) that may compress the olfactory bulbs. Unilateral damage is often not noticed by the patient, but bilateral damage (which surprisingly can also be missed if it develops slowly) may lead to scenarios such as that described in the question stem. Anosmia (loss of sense of smell) results and can also greatly affect a patient's sense of taste. The olfactory nerves are technically the neurons on the mucosal side of the cribriform plate that send processes through the plate to synapse in the olfactory bulb, which in turn sends axons to the piriform cortex (the primary olfactory cortex), that overlies the uncus of the temporal lobe.
question
An 82-year-old man who lives in a nursing home is brought to the physician because of increasing forgetfulness over the past two years. An MRI of the head is shown. The man later dies of a respiratory infection, and an autopsy is performed. Histologic examination of the cerebral structures indicated by the arrows shows accumulation of senile plaques and neurofibrillary tangles. In which of the following cerebral regions are these histologic findings usually most prominent? A. Auditory temporal cortex B. Cingulate cortex C. Hippocampus D. Mammillary bodies E. Thalamus F. Visual occipital cortex
answer
The correct answer is C. The dementing condition referred to here is Alzheimer disease, which results in memory impairment and mostly affects short-term memory functions. The pathogenesis is related to accumulation of beta-amyloid in senile plaques and intraneuronal deposits of abnormal tau protein within neurons (neurofibrillary tangles). The hippocampus is one of the earliest and most severely affected cerebral regions. This phylogenetically old region of the brain is known to play an important role in short-term memory and lies deep in the hippocampal fissure, in the medial temporal lobe.
question
A 50-year-old man is brought to the emergency department after falling through thin ice while ice fishing. His friends were unable to remove him from the water for approximately five minutes. Upon being retrieved, the man was unconscious but later revived through aggressive CPR. Since the episode, the patient has had difficulty holding a conversation. He would interact with the physicians normally initially. However, after the physician leaves the room and returns, he is greeted as if this was their first encounter, and the patient is unable to recall any details of his previous conversation. He has no difficulty recalling information about events prior to his injury. Which of the following structures is most likely damaged in this patient?
answer
The correct answer is the hippocampus. The structure indicated by the arrow that is implicated in memory formation is the hippocampus. It is exquisitely sensitive to hypoxic damage, and is thus damaged frequently in anoxic brain injury. Bilateral damage to the medial temporal lobes including the hippocampus results in a profound loss of ability to acquire new information, known as anterograde amnesia.
question
A 38-year-old woman comes to the physician because of headaches and changes in vision. Physical examination shows acuity of 20/20 in both eyes after correction. The color perception is intact. A Goldmann visual field test from the patient is shown. Goldmann visual field test shows loss of temporal vision. The funduscopic examination is normal except for mild swelling of the optic disks. In addition to visual symptoms, which of the following is also likely to be seen in this patient? A. Breast discharge B. Facial droop C. Palpitations D. Polyuria E. Tinnitus
answer
The correct answer is A. The Goldmann visual field test reveals a bitemporal hemianopsia, which is defined as a loss of peripheral vision in both eyes. The most common cause of this unique pattern of visual field loss is a mass in the sella turcica, such as a pituitary adenoma, craniopharyngioma, or neurosarcoid lesion. Compression of the optic chiasm at this location leads to loss of fibers that originate from the nasal hemiretinae of both eyes and cross at the chiasm; this pattern of damage produces loss of the temporal visual fields on both sides. The most common type of pituitary adenoma is a prolactinoma, which would secrete prolactin and lead to galactorrhea. Patients often complain of a milky breast discharge, which may precede the onset of visual changes
question
A 42-year-old man is brought to the physician by his sister because of a change in his appearance over the past 2 years. He says that he has developed a tingling sensation in the 1st, 2nd, and 3rd digits of the right hand, and loss of coordination and strength of the right thumb. Physical examination shows overgrowth of the frontal bones and enlargement of the hands and feet. Compression of which of the following is the most likely cause of the symptoms affecting the patient's right hand? A. Anterior interosseous nerve B. Median nerve C. Musculocutaneous nerve D. Radial nerve E. Ulnar nerve
answer
The correct answer is B. This patient has acromegaly, which is characterized by overgrowth of the face, jaws, hands, and feet, enlargement of internal organs, hyperglycemia, hypertension, and osteoporosis. It is caused by hypersecretion of growth hormone, often attributed to an adenohypophyseal tumor. Complications include degenerative joint disease, muscular weakness, neuropathies, and diabetes mellitus. In the present case, although the patient's sensory symptoms may be caused by a neuropathy, it is very likely that overgrowth in the wrist area has compressed the carpal tunnel, thereby impinging on the median nerve. Note that the median nerve (root C6-T1) provides motor innervation to the anterior forearm muscles, thenar muscles, and two radial lumbricals. It provides sensory innervation to the radial two-thirds of the palm, the volar surfaces of the thumb, 2nd, and 3rd digits, and the radial half of the 4th digit.
question
A 52-year-old man is brought to the emergency department by police because of confusion. A bystander had stated to the police that he was in a fight earlier and had sustained several blows. He appears malnourished and smells like wine. Physical examination shows orientation only to self and multiple bruises over his torso and extremities. Ocular examination shows horizontal nystagmus on lateral gaze and bilateral lateral rectus palsies. Deficiency of which of the following vitamins best explains this man's neurologic findings? A. Folate B. Thiamine C. Vitamin B12 D. Vitamin C E. Vitamin K
answer
The correct answer is B. This patient has Wernicke-Korsakoff syndrome, which is a neurologic complication of thiamine (vitamin B1) deficiency and is most likely to occur in patients with malnutrition in the setting of alcohol abuse. (It can occur in patients who do not abuse alcohol, e.g., anorexia and other starvation states, hyperemesis of pregnancy, malignancy, AIDS). This is best thought of as two distinct syndromes, each representing a different stage of the disease. Wernicke encephalopathy (WE) occurs acutely and is characterized by the clinical triad of encephalopathy, oculomotor dysfunction (e.g., nystagmus, lateral rectus palsy, conjugate gaze palsy), and gait ataxia. It requires emergent treatment; if left untreated, most patients progress to coma and death. Treatment is with thiamine. Thiamine must be administered prior to glucose. Administration of glucose to malnourished patients can exhaust their thiamine supplies and precipitate or worsen Wernicke-Korsakoff syndrome (metabolism of glucose uses enzymes that require thiamine as a cofactor). If treated quickly and effectively, this stage can be reversible.
question
A 45-year-old woman comes to the physician because of recent onset discharge of white milky fluid from her breast. She reports that her most recent mammogram and pregnancy tests were negative. She has also recently noticed significant morning headaches and indicates that her vision has gradually become more blurry in the last few months. She is afraid to drive because occasionally she is not able to see the cars in the lanes next to her. Physical examination shows bilateral papilledema, bilateral hemianopia, and no focal sensory or motor deficits. An MRI of the brain reveals a mass in the sella turcica and a hypophysectomy is planned. The tumor will most likely be resected transnasally through which of the following structures? A. Cavernous sinus B. Ethmoid sinus C. Frontal sinus D. Mastoid sinus E. Sphenoid sinus
answer
The correct answer is E. In this case, the patient has a pituitary adenoma that is most likely secreting excessive prolactin. The hypophysis lies in the sella turcica which is on the body of the sphenoid bone and can be approached transnasally through the sphenoid sinus.
question
A 61-year-old man is brought to the physician because of confusion and difficulty walking. His daughter says that he has been a heavy drinker for 35 years but does not believe that he has had alcohol recently. Physical examination shows ataxia and a sluggish pupillary light reflex. One week later, he returns to the physician for a follow-up examination. Physical examination shows a normal gait and pupillary light reflex. Neuropsychologic testing shows severe short-term memory impairment and frank confabulation. The vitamin deficiency most likely associated with his condition may also lead to which of the following conditions? A. Dermatitis B. Hypertrophic cardiomyopathy C. Megaloblastic anemia D. Night blindness E. Peripheral neuropathy
answer
The correct answer is E. This clinical case is an example of Wernicke-Korsakoff syndrome, which is caused by thiamine deficiency, usually secondary to chronic alcohol abuse. The acute symptoms are referred to as Wernicke encephalopathy and include encephalopathy (disorientation, indifference, inattentiveness), gait ataxia, and oculomotor dysfunction (nystagmus, lateral rectus palsies, conjugate gaze palsies, pupillary abnormalities). The chronic symptoms are referred to as Korsakoff syndrome and include anterograde amnesia and confabulation in an alert and responsive patient. Thiamine deficiency is also associated with either "dry" beriberi (peripheral neuropathy- symmetric sensory and motor impairments, especially of distal extremities) and/or or "wet" beriberi (neuropathy and cardiac involvement with cardiomegaly, cardiac myopathy, congestive heart failure, and edema). Pyruvate dehydrogenase is a mitochondrial enzyme that catalyzes the conversion of pyruvate to acetyl-CoA. Cofactors and coenzymes used by pyruvate dehydrogenase include thiamine pyrophosphate (TPP; from the vitamin thiamine), lipoic acid, coenzyme A (from the vitamin pantothenate), FAD[H2] (from the vitamin riboflavin), and NAD[H] (from the vitamin niacin). TPP is also a coenzyme for α-ketoglutarate dehydrogenase, branched-chain α-keto acid dehydrogenase, and transketolase.
question
A 35-year-old man comes to the physician because of difficulty controlling jerky, involuntary movements. His family medical history is unknown. His symptoms progress over the following 5 years and he eventually develops difficulty with ambulating, dressing, eating, and cognition. Ten years after the initial presentation, the patient dies. Which of the following findings is most likely on examination of tissue obtained from the brain on autopsy? A. Degeneration of the caudate nucleus B. Depigmentation of the substantia nigra C. Diffuse cortical atrophy with relative sparing of primary motor and sensory areas D. Selective frontal and temporal lobe atrophy E. Widespread neuronal loss and gliosis in subcortical sites
answer
The correct answer is A. The patient has Huntington disease (HD). Patients often present in their 20s to 40s with personality changes and a movement disorder. The movement disorder is characterized by chorea and athetosis. Chorea is characterized by many rapid movements that are random in nature and can resemble dancing; athetosis is a series of slow, writhing movements. Cognitive decline is characteristic of HD. Symptoms of dementia include irritability, loss of interest, impairment of intellectual and executive functions, and memory disturbances. Psychiatric symptoms such as depression are also frequent. Family history is generally positive for Huntington disease, since it is inherited as an autosomal dominant. Pathologic findings include severe atrophy of the caudate nucleus (with loss of GABAergic medium-sized spiny neurons), less severe involvement of the putamen and cerebral cortex, and dilation of the lateral ventricles apparent on CT and MRI studies. The disorder is known to be caused by expansion of a CAG trinucleotide repeat in a gene on the short arm of chromosome 4 coding for a protein called huntingtin. This disease shows anticipation; subsequent family members who have the disease show manifestations earlier than the earlier generations did. The mean age of onset ranges from 35-44 years. There is no effective therapy.
question
A 72-year-old man comes to the physician because of shaking in his right hand and trouble initiating movements. Physical examination shows a tremor of the right hand that decreases with active movement. When asked to walk, the patient has difficulty starting and then takes small, shuffling steps. The patient's has decreased facial expression and his voice is soft. A diagram of the brainstem is shown. Which of the following labeled structures in the diagram is the most likely site of this patient's lesion? A. A B. B C. C D. D E. E
answer
The correct answer is D. This is a classic presentation of Parkinson disease. Parkinson disease is characterized by a pill-rolling resting tremor, masked facies, rigidity, trouble initiating movements, and small, shuffling steps (festinating gait). The onset of motor signs in this disease is often asymmetric and typically presents with a resting tremor in one upper extremity. The disease results from the degeneration of the dopaminergic neurons in the substantia nigra pars compacta of the midbrain.
question
A 48-year-old man comes to the physician with his wife because of progressive memory loss over the past several years. His wife says that she has also noticed changes in his personality. Physical examination shows abnormal writhing movements of the limbs and hyperreactive reflexes. An MRI of the brain shows a loss of volume in the caudate nucleus and putamen. Which of the following pedigrees shows the most likely type of inheritance for this patient's disorder? What's the disease? What's the form of inheritance?
answer
The correct answer is A. This patient has Huntington disease, a progressive neurodegerative disorder characterized by choreiform movements, psychiatric problems, and dementia. This disease has an autosomal dominant inheritance pattern and is caused by a trinucleotide (CAG) repeat expansion in the huntingtin gene on chromosome 4p. It is characterized by severe degeneration of the caudate nucleus along with degenerative changes in the putamen and cortex. Age of onset is typically between 30 and 50. Onset is correlated to the number of CAG repeats, and expansion of the repeats causes the most recent generations to develop the disease at an earlier age or with greater severity, a phenomenon known as anticipation. Because affected individuals must receive a disease-causing gene from an affected parent, the disease is typically observed in multiple generations of a pedigree. Skipped generations are not typically seen because two unaffected parents cannot transmit a disease-causing allele to their offspring. Because these genes are located on autosomes, males and females are affected in roughly equal frequencies.
question
A 68-year-old man comes to the physician because of progressive difficulty walking and headaches over the past 1 month. He has smoked 2 packs of cigarettes daily for 30 years but denies alcohol use. His wife says that he walks as if he were drunk, staggering and losing his balance. Neurologic examination shows a wide-based, unsteady gait and a tendency to fall to the right. An MRI of a normal brain is shown. A lesion of which of the following labeled structures in the normal MRI is most likely responsible for this patient's symptoms? A. A B. B C. C D. D E. E F. F G. G H. H
answer
The correct answer is G. The cerebellum is concerned primarily with the planning and fine-tuning of movements and balance. The vermis is located in the midline of the cerebellum and primarily receives input from the spinal cord. This man is showing signs of truncal ataxia, but not appendicular ataxia, manifested by the wide-based, "drunken" gait. A likely lesion location for this presentation is the cerebellar vermis. The presence of headache suggests the lesion is intracranial. Because of his history of smoking, metastatic lung cancer to the cerebellar vermis should be high in the differential. Letter G labels the vermis of the cerebellum.
question
A 71-year-old man who is a working attorney is brought to the emergency department 20 minutes after a motor vehicle collision. Physical examination shows several lacerations to the face and extremities and contusions to the thorax. X-rays of the spine and chest are normal. Noncontrast head CT shows some evidence of minor age-appropriate brain atrophy, but no evidence of bleeding is seen. After admission for overnight observation, he is discharged from the hospital. Three weeks later, he is brought to the emergency department by his wife because of confusion. He is not oriented to time or place, and can recall only one out of six objects after 3 minutes. Which of the following is the most likely underlying condition? A. Alzheimer disease B. Brain metastases C. Epidural hematoma D. Normal pressure hydrocephalus E. Subdural hematoma
answer
The correct answer is E. This is a classic history and presentation of a chronic or subacute subdural hematoma, in which symptoms can present several days or up to 3 weeks following a traumatic head injury. The patient will often have had an initial hospital admission with a negative workup and no evidence of abnormalities on serial CT scans of the head. However, either gradually or abruptly, the patient can experience decreasing mental status, sleepiness, focal neurologic deficits, seizures, etc. from subdural accumulation of blood and subsequent compression of the brain. The bleeding originates from the bridging veins of the meninges, and since the bleed is under venous pressure (unlike the arterial pressure of an epidural hematoma), there is more time for blood to accumulate before symptoms arise. Atrophy of the brain allows for more space and movement in this area. Thus, trauma with coup contre coup affect can cause trauma and shearing of the bridging vessels. Abnormal hemostasis (e.g., thrombocytopenia, platelet dysfunction, anticoagulant overdose) can also cause a subdural hemorrhage.
question
A 6-year-old boy is brought to a physician because of multiple episodes of sudden-onset fainting episodes that occur without warning and, according to his mother, result in loss of muscle tone, particularly of his neck. He has no abnormal movements or loss of bowel or bladder functions during these episodes. Physical examination is completely normal. ECG is normal. An EEG shows an abnormal spiking pattern. Which of the following types of seizures is most likely present in this patient? A. Absence B. Atonic C. Myoclonic D. Tonic E. Tonic-clonic
answer
The correct answer is B. All of the forms of seizures listed in the answer choices are generalized seizures (seizures that initially involve both hemispheres). The type of seizure that would most closely resemble a fainting spell is the atonic, or "drop," seizure. In this form of seizure, the patient suddenly loses muscle tone and falls to the floor with loss of consciousness. They typically present in young children and may occur as part of childhood epilepsy syndromes.
question
A 22-year-old man is brought to the emergency department because of a sudden-onset headache and mental status changes. He sustained a blow to the right lateral side of his head 8 hours ago on a construction site and was "knocked out." He recovered consciousness after several minutes and remained asymptomatic until 2 hours ago. In the emergency department, he experiences nausea and vomiting. His pulse is 111/min, respirations are 17/min, and blood pressure is 145/83 mm Hg. Physical examination shows altered mental status and a hematoma overlying his right parietal region. Emergent CT head without contrast is ordered, which is diagnostic and which prompts a neurosurgical consultation. Which of the following is the most likely diagnosis? A. Basilar skull fracture B. Epidural hematoma C. Intracerebral hemorrhage D. Subarachnoid hemorrhage E. Subdural hematoma
answer
The correct answer is B. All of answer choices can occur in head trauma, however the scenario described is classic for epidural hematoma. In this scenario, a severe blow to the lateral skull causes both skull fracture and laceration of the middle meningeal artery, leading to a momentary loss of consciousness that is followed by a lucid (asymptomatic) period of 1-48 hours (usually a few hours) before the patient's neurologic condition deteriorates. This scenario is frequently tested on examinations, but you should be aware that in the real clinical world there may be no initial loss of consciousness. A CT scan of the head will typically demonstrate a lens-shaped (biconvex) hematoma adjacent to a fracture (note that a fracture need not always be present, but it is very commonly associated).
question
A 47-year-old woman returns to the physician for follow-up examination for a progressive neurological condition. An MRI of her brain shows enlarged ventricles. Which of the following symptoms is most likely to be found in this patient? A. Bradykinesia, resting tremor, and difficulty initiating movements B. Choreoathetosis and dementia C. Intention tremor and akinesia D. Moon facies, amenorrhea, and hypertension E. Motor and vocal tics F. Slowly progressive memory impairment and irritability G. Violent flinging of proximal limb muscles
answer
The correct answer is B. This patient most likely has Huntington disease (HD), which is characterized by choreoathetosis and dementia. The caudate degenerates markedly in this disease, although the putamen and nucleus accumbens also degenerate. Atrophy of the caudate and putamen can make the lateral ventricles appear very large when imaged (hydrocephalus ex vacuo). The image shows a horizontal brain section. Realize that as the caudate degenerates, the lateral ventricle will get larger.
question
A 58-year-old man comes to the emergency department because of numbness in his right face, arm, and leg for the last 2 days.He has hypertension and has smoked 2 packs of cigarettes per day for 30 years. His temperature is 36.7ºC (98.4ºF), pulse rate is 79/min, respirations are 16/min, and blood pressure is 202/115 mm Hg. Physical examination shows decreased pinprick, temperature, light touch, and vibration sensation over his right face and body. Two-point discrimination is 16 mm on the right index finger and 5 mm on the left. A normal MRI of the brain is shown. These findings are most likely to be caused by a lesion in which of the following labeled structures on the normal MRI?
answer
The correct answer is A. This patient has decreased sensation over his right face and body. This includes all sensory modalities, including those derived from the dorsal column system and the anterolateral system. The structure that both of these systems have in common is the left ventral posterior thalamus. Remember, the right side of an MRI is the left side of the patient and the left side of an MRI shows the right side of the patient. General sensory information relays to the thalamus: From the contralateral body: VPL (ventral posterolateral nucleus) From the contralateral face: VPM (ventral posteromedial nucleus) The thalamus is labeled A in the MRI. A lesion in the thalamus can disrupt all sensory information originating from both the body and the face.
question
A 20-year-old man is brought to the emergency department after a witnessed ground level fall. He reports walking on the sidewalk when he suddenly fell to the ground and hit his head. He has had similar episodes since childhood, but they never caused injury. His friends, who witnessed the event, report that he was only unconscious briefly and recovered within a few seconds. The patient does not report any bladder or bowel incontinence during the episode. His past medical history is unremarkable, and he does not currently take any medications. He has never smoked, only occasionally drinks alcohol, and does not use any recreational drugs. Physical examination in the emergency department is remarkable for minor abrasions on the left side of the face and head. An EKG and CT scan of the head reveal no abnormalities. Which of the following is most likely responsible for this patient's fall? A. Absence seizure B. Atonic seizure C. Myoclonic seizure D. Tonic seizure E. Tonic-clonic seizure
answer
The correct answer is B. Atonic seizures (also known as drop seizures or akinetic seizures) are characterized by a sudden loss of postural muscle tone that lasts only a few seconds. A very brief seizure may cause only a drop of the head, but a longer seizure may cause the patient to slump to the ground. This type of seizure may be quite dangerous because of the risk of head injury with a sudden fall. Consciousness may be briefly impaired; however, unlike grand-mal seizures, there is rarely post-ictal confusion. Atonic seizures are relatively rare and usually occur as part of childhood epilepsy syndromes such as Lennox-Gastaut syndrome (LGS). LGS patients present in childhood with multiple seizure types, mental retardation, and spike and wave EEG pattern. This syndrome typically persists in the adult years. Loss of bowel or bladder control is rare for these types of seizures. Common treatments include valproic acid and clonazepam.
question
A 20-year-old man is brought to the emergency department (ED) by his mother after having seizures. The patient's mother says that he suddenly lost consciousness, had several jerky movements of his muscles, and had total loss of postural control. Over the next half hour while still in the ED, the patient has a series of rhythmic contractions of all four limbs, without a return to full consciousness. Which of the following is the most appropriate medication that should be administered first to this patient? A. Carbamazepine B. Ethosuximide C. Lorazepam D. Phenytoin E. Valproic acid
answer
The correct answer is C. The patient is most likely presenting with status epilepticus. The definition of status epilepticus, a life-threatening neurologic condition, can vary. It can be the occurrence of a single unremitting seizure that lasts longer than 5-10 minutes or frequent seizures without an interictal return to a baseline state. In the case of generalized convulsive status epilepticus, there is often a sudden loss of consciousness associated with a tonic contraction of muscles and total loss of postural control. The patient usually falls to the floor in an opisthotonic posture, often sustaining injury. Next, there is a series of rhythmic contractions of all four limbs. Unlike the classic "tonic-clonic" seizure, in which there is a gradual return to consciousness with cessation of the seizure activity, patients with status epilepticus will present without recovery of consciousness or cessation of seizure activity. Since status epilepticus is a medical emergency associated with a high rate of mortality if untreated, therapeutic intervention is needed to immediately cease seizure activity. Benzodiazepines (lorazepam, diazepam, or midazolam) are considered the initial drugs of choice in patients with status epilepticus because of their rapid action. In addition to benzodiazepines, phenytoin (or fosphenytoin) can be administered for long-term seizure control.
question
A 22-year-old woman comes to the emergency department because of a persistent and severe headache. She says that she has been getting headaches since she was a teenager but they have become increasingly debilitating. The episodes occur once or twice a month and can last up to 2 days. She describes pain beginning in her right temple and spreading over her scalp over a period of hours. The pain is throbbing and she often has severe nausea. She has only taken over the counter medications for pain and tries to lay down in a quiet, darkened room. Physical examination shows normal vital signs. She is administered a drug to stop her headache. This agent most likely acts at which of the following locations? A. Beta-adrenergic receptors B. Calcium channels C. Cholinergic presynaptic terminals D. Dopamine receptors E. Serotonin receptors
answer
The correct answer is E. This patient has a migraine, which is a complex and recurrent headache that occurs most often in woman (75%). A migraine typically presents as unilateral head pain, which throbs and is aggravated by activity. It may be associated with visual or other sensory symptoms prior to the headache (aura). Nausea and vomiting can occur and it is often associated with photophobia and phonophobia. Treatment can involve acute (abortive) and preventative (prophylactic) therapy. The triptans (e.g., sumatriptan, almotriptan, rizatriptan, and zolmitriptan) are 5-HT1B/1D agonists and are used in abortive therapy of migraine. 5-HT1B/1D receptors are found postsynaptically on blood vessels and presynaptically on sensory nerves. The pathophysiology of migraines and the exact mechanism of action of triptans are not fully understood. Triptans likely act on cerebral blood vessels, causing vasoconstriction, and on nerve terminals to inhibit the release of calcitonin gene-related peptide (CGRP) and substance P from trigeminal afferents.
question
A 48-year-old man is brought to the emergency department by his wife because of a severe headache, sensitivity to light, and nausea. His temperature is 37.1°C (98.8°F), pulse is 83/min, and blood pressure is 145/93 mm Hg. The patient is administered a medication to address his symptoms. Approximately 90 minutes later, and the patient says that his headache is better but still does not feel well. Physical examination reveals a blood pressure of 246/144 mm Hg. Which of the following drugs was the patient most likely administered? A. Ergotamine B. Methysergide C. Metoclopramide D. Propranolol E. Sumatriptan
answer
The correct answer is E. The patient comes to the emergency department because of a likely migraine headache and is administered is a medication that causes a hypertensive emergency. A hypertensive emergency presents as evidence of acute end-organ damage in a setting of markedly elevated blood pressure. Hypertensive emergencies can occur in a variety of conditions, including acute head injury or trauma, and both prescription and illicit drugs. Examples of medications that can lead to this condition include cocaine, amphetamine(s), phencyclidine, monoamine oxidase inhibitors, triptans (such as sumatriptan) or recent discontinuation of clonidine or other sympatholytic agents. Medications commonly used in the management of hypertensive emergencies include nitroprusside, clevidipine, nicardipine, fenoldopam, esmolol, and labetalol.
question
A 24-year-old woman comes to the emergency department because of a persistent and severe headache. She says that she has gotten headaches since she was a teenager but they have become increasingly debilitating. The episodes occur once or twice a month and can last up to 2 days. She describes pain beginning in her right temple and spreading over her scalp over a period of hours. The pain is throbbing and she often has severe nausea. She has only taken over the counter medications for pain and tries to lay down in a quiet, darkened room. Physical examination shows normal vital signs. She is administered a drug to stop her headache. Which of the following is the most likely mechanism of action of this drug? A. Serotonin 5-HT1A partial agonist B. Serotonin 5-HT1B/1D agonist C. Serotonin 5-HT2 antagonist D. Serotonin 5-HT3 antagonist E. Serotonin reuptake blocker
answer
The correct answer is B. This patient has a migraine, which is a complex and recurrent headache that occurs most often in woman (75%). It is most often characterized as unilateral head pain, which throbs and is aggravated by activity. It may be associated with visual or other sensory symptoms prior to the headache (aura). Nausea and vomiting can occur and it is often associated with photophobia and phonophobia. Treatment can involve acute (abortive) and preventative (prophylactic) therapy. The triptans (e.g., sumatriptan, almotriptan, rizatriptan, and zolmitriptan) are 5-HT1B/1D agonists and are used in abortive therapy of migraine. 5-HT1B/1D receptors are found postsynaptically on blood vessels and presynaptically on sensory nerves. The pathophysiology of migraines and the exact mechanism of action of triptans are not fully understood. Triptans likely act on cerebral blood vessels, causing vasoconstriction, and on nerve terminals to inhibit the release of calcitonin gene-related peptide (CGRP) and substance P from trigeminal afferents.
question
A 16-year-old girl is brought to the emergency department by her parents because of a 1-day history of right foot pain. She denies any recent trauma, has no history of any major medical illness, and takes no medications. She is active, and runs at least 1 hour daily in the nearby woods. She attends high school and her parents state she has been doing very well, but have noticed that she does not visit with her friends as much. She gets along well with her parents, except that they insist she should eat more; her weight has dropped from 59 kg (130 lb) to 47.6 kg (105 lb) over the past year. BMI is 16 kg/m2. Which of the following is the most likely underlying condition? A. Conversion disorder (functional neurological symptom disorder) B. Depression with somatic manifestation C. Gonococcal arthritis D. Injured medial ankle tendon E. Metatarsal stress fracture F. Tick bite
answer
The correct answer is E. This patient is not eating well, as mentioned by her parents, and her weight loss (over 15% of baseline) as well as her school performance and activity level are consistent with the feeding and eating disorder, anorexia nervosa. Symptoms commonly observed in these patients include amenorrhea, hair loss, lanugo body hair, constipation, lethargy and other mental disorders, including depression. Depression symptoms can include increasing social isolation, as seen with this patient. Patients may also experience exercise-induced injuries secondary to prolonged, excessive exercise sessions. Stress fractures are caused by repetitive actions or impacts to the bones. Metatarsal stress fracture is a complication of rigorous, prolonged walking or running in this population. Stress fractures, in general, have a higher incidence in anorexia. This may be related to the excessive exercise, inadequate nutrition intake and/or hormonal changes (estrogen, progesterone).
question
A 23-year-old man undergoes general anesthesia for an inguinal hernia repair. The anesthesiologist administers fentanyl, propofol, and midazolam to achieve anesthesia. The surgeon finishes the repair and wants to take the patient off the ventilator as quickly as possible. Which of the following drugs or combination of drugs will most effectively reverse the respiratory depression caused by the anesthetic drugs given to this patient? A. Buprenorphine B. Buprenorphine and flumazenil C. Flumazenil D. Naloxone E. Naloxone and buprenorphine F. Naloxone and flumazenil
answer
The correct answer is F. Fentanyl, an opioid analgesic, is frequently used during general anesthesia to produce analgesia and respiratory depression. While allowing the anesthesiologist to easily ventilate the patient intraoperatively, the respiratory depression produced by opioids may linger postoperatively. Patients will be difficult to extubate, as their central respiratory drive is suppressed. Administering naloxone (choices D, E, and F), an opioid antagonist, will expedite this process. Naloxone is an antagonist at all three opioid receptors (mu, kappa and delta) but has the highest affinity for the mu receptor. Care should be taken when using naloxone: administering too much will not only counteract the respiratory depression, it will also counteract the analgesic properties. In addition to fentanyl, midazolam (a benzodiazepine) also causes respiratory depression. Flumazenil is a competitive antagonist at the benzodiazepine site on GABA receptors. Therefore, because both fentanyl and midazolam can cause respiratory depression, both naloxone and flumazenil would be indicated.
question
A 65-year-old man with a history of severe alcohol use disorder is brought to the emergency department because of altered mental status, diaphoresis, and irritability. He appears very agitated. He states that his last drink was 2 days ago and has been unable to purchase any more alcohol. His pulse is 124/min and his blood pressure is 170/82 mm Hg. Treatment with thiamine, magnesium, and a short-acting CNS medication is begun. Which of the following best describes the mechanism of action of this CNS drug? A. Binds chloride ion channels B. Blocks sodium channels C. Decreases effects of γ-aminobutyric acid (GABA) D. Decreases the influx of cations by binding N-methyl-d-aspartate (NMDA) receptors E. Inhibits 5-hydroxytryptamine (5-HT) reuptake
answer
The correct answer is A. In DSM-5, alcohol abuse and alcohol dependence were combined into a single alcohol use disorder. All of the alcohol related conditions, including intoxication and withdrawal, are also included in the alcohol use disorder diagnosis. This patient's is clearly experiencing alcohol withdrawal. Patients with alcohol withdrawal have typically abused alcohol on a daily basis for at least 3 months or have consumed large quantities for at least a week. Withdrawal symptoms typically appear 6-12 hours after individuals stop or dramatically decrease alcohol intake. There is a continuum of signs and symptoms ranging from tremulousness to delirium tremens (DTs). Benzodiazepines are indicated for the short-term management of alcohol withdrawal, panic disorder, and generalized anxiety disorder. These receptors form part of the GABAA receptor-chloride ion channel macromolecular complex. Benzodiazepines bind to these receptors and potentiate the inhibitory actions of GABA (compare to choice C) by increasing chloride ion conductance by increasing the frequency of channel opening.
question
A 45-year-old woman with elevated body mass index comes to the physician because of severe right upper quadrant abdominal pain after ingesting fatty meals. A laparoscopic cholecystectomy is performed. An intraoperative cholangiogram shows no stones within the biliary duct system. Two days later, however, she again develops severe abdominal pain. Laboratory studies show a bilirubin of 2.2 mg/dL, aspartate aminotransferase (AST) of 398 U/L, and alanine aminotransferase (ALT) of 388 U/L. An ultrasound of the abdomen shows hepatomegaly but no abnormalities of the biliary tree. Which of the following best explains the findings in this patient? A. A staple has dislodged from the cystic duct B. A stone has been retained in the common bile duct C. Halothane was used during general anesthesia D. Postoperative aminoglycoside antibiotics were used E. Postoperative ketorolac was used for analgesia
answer
The correct answer is C. Halothane was the first halogenated volatile anesthetic produced and is still indicated for the induction and maintenance of general anesthesia. However, this agent is rarely used in the United States since less toxic and more effective general anesthetics are available. Shortly after its introduction, the first cases of "halothane hepatitis" were reported. After exposure to this inhaled anesthetic agent, patients may experience hepatitis with focal to massive hepatic necrosis. (Note that most patients develop hepatotoxicity rather than hepatitis and this hepatotoxicity results from halothane's P450-mediated biotransformation to reactive intermediates. Most patients are asymptomatic and recover without treatment. Hepatitis from halothane is rare with an incidence of 1/6000 to 1/15000 reported in the literature. Symptoms include fever, rash, nausea, myalgias, right upper quadrant abdominal pain, and anorexia and develop two days to three weeks after exposure. Females older than age 40 are at greatest risk. Labs will be consistent with hepatitis and/or acute liver failure depending on the extent of hepatic injury.) Other serious adverse effects include malignant hyperthermia, cardiac arrhythmias, and blood pressure changes.
question
A 28-year-old woman comes to the physician because of methamphetamine abuse. She states that she began taking methamphetamine because she was working two jobs and needed to stay awake. She liked the euphoric effects of the drug and kept taking it to reexperience the "high." Which of the following mechanisms of action is most likely responsible for the reinforcing effects of methamphetamine? A. Blocks the metabolism of both dopamine and norepinephrine B. Directly stimulates adrenergic receptors C. Directly stimulates dopamine receptors D. Induces dopamine release E. Induces norepinephrine release
answer
The correct answer is D. Methamphetamine acts by gaining entrance to dopamine, norepinephrine, and serotonin nerve terminals, causing the release of these neurotransmitters via the uptake carriers. Dopamine is believed to play an important role in the reward system of the brain, and is thought to be a significant factor in the reinforcing effects of stimulants. One area of the brain that is thought to be involved in this reward system is the dopaminergic projection from the ventral tegmental area of the midbrain to the nucleus accumbens of the forebrain. By comparison, cocaine blocks the reuptake of dopamine and norepinephrine, and methylenedioxymethamphetamine (MDMA) causes the release of serotonin (5-HT).
question
A 15-year-old girl is brought to the emergency department by friends because she has been acting erratically and periodically violent over the past couple of hours. Her friends say that she is likely drinking or taking drugs. The patient appears agitated, ataxic, and disoriented. Her temperature is 37.0ºC (98.6ºF), pulse is 98/min, respirations are 16/min, and blood pressure is 150/92 mm Hg. She needs to be restrained by two hospital staff members to complete a physical examination that shows normoactive bowel sounds, dilated pupils, and a rapid involuntary movement of the eyes. Which of the following drugs is the most likely cause of this patient's symptoms? A. Amphetamine B. Ethanol C. Heroin D. Lysergic acid diethylamide E. Phencyclidine
answer
The answer is E. Phencyclidine (PCP), also known as "angel dust," is a dissociative anesthetic that can be taken orally, by smoking, or by intravenous injection. PCP causes disorientation, detachment, reckless behavior, impaired judgment, and distortions of body image. Somatic signs include horizontal or vertical (mostly) nystagmus (rapid involuntary movement of the eyes), hypertension, tachycardia, diaphoresis, lack of motor coordination, and numbness. High doses can produce vomiting, seizures, stupor, coma, or death. Rapid control of psychomotor agitation is the cornerstone of PCP management and is generally achieved with intravenous benzodiazepines, such as lorazepam. When benzodiazepines are unsuccessful, haloperidol may be used.
question
A 57-year-old man is brought to the emergency department by his son because of a large laceration on the head. The patient says that he was attacked by 5 thugs in the back alley of the supermarket after he picked up milk, fruit, and vegetables. The patient's son says that his father often "makes things up for no reason" and all that had really happened was that his father tripped on a rake. A review of this patient's medical history is most likely to show which of the following? A. Alzheimer disease B. Bipolar disorder C. Chronic alcohol use disorder D. Malingering E. Paranoid personality disorder F. Schizophrenia
answer
The answer is C. The patient is displaying confabulation, in which a patient with severe recent memory problems, such as with chronic, severe alcohol use disorder associated with Wernicke-Korsakoff syndrome, often unconsciously makes up explanations for events that would otherwise be inexplicable to the patient. Confabulation is distinguished from "lying" by its occurrence in the setting of memory loss, and the patient often does not realize that what he is saying is not true.
question
A 29-year-old woman is admitted to the hospital for emergency surgery following an accident. General anesthesia is administered and 2 hours into the surgery, there is excessive bleeding. The patient is transfused, and moved to an open ward after stabilization. Two days later, she becomes agitated and develops tremors. She says that she feels as if insects are crawling on her. A history of which of the following is most likely responsible for the development of these symptoms in this patient? A. Alcohol use disorder B. General anesthesia C. Hypoglycemia D. LSD use E. Schizophrenia
answer
The answer is A. This patient likely has alcohol use disorder and is experiencing alcohol withdrawal. Withdrawal symptoms typically appear 6-12 hours after patients stop or substantially decrease alcohol intake. There is a continuum of signs and symptoms ranging from tremulousness to delirium tremens (DTs).
question
A 28-year-old man comes to the physician after experiencing five days of intense, vivid dreams and nightmares. He states that he used to sleep erratically for the past several years and just wrote it off as anxiety from his job. Upon further questioning, he states that he was drinking three shots of whiskey to help him fall asleep for the past two years, but stopped this practice a week ago after frequent urgings from his wife. Which of the following could also precipitate the sleep symptom present in this patient? A. Benzodiazepine cessation B. Cocaine intoxication C. Lithium initiation D. Paroxetine cessation E. Phenelzine initiation
answer
The correct answer is D. The patient in this vignette is experiencing rapid eye movement (REM) rebound from the abrupt discontinuation of alcohol. Alcohol, when used chronically, will depress the amount of REM sleep. When the alcohol use is stopped, there will be a compensatory increase in the amount of REM sleep known as REM rebound. This phenomenon is characterized by an increase in the number and intensity of dreams for several days. REM rebound can also occur with the discontinuation of medications or drugs known to suppress REM including alcohol, barbiturates, stimulants, tricyclic antidepressants, selective serotonin reuptake inhibitors (SSRIs), lithium, and monoamine oxidase inhibitors. Paroxetine is a selective serotonin reuptake inhibitor and when discontinued abruptly may cause REM rebound similar to what this patient is experiencing.
question
A 56-year-old man with a history of alcohol use disorder comes to the physician because of depression. He says that he has been sober for a month and is enrolled in a 12-step program, but he is unable to go back to work. The physician strongly suspects that the man relapsed and was fired from his work. Which of the following laboratory tests will most likely confirm the suspected relapse? A. Aminotransferases B. Blood urea nitrogen C. Mean corpuscular volume D. Serum amylase E. Serum gamma glutamyl transferase F. Serum triglycerides G. Urine toxicology screen
answer
The correct answer is E. Serum gamma glutamyl transferase (GGT) (also called gamma glutamyl transpeptidase) is a very sensitive indicator of recent alcohol use. It is elevated in approximately 80% of alcohol-related disorders and is usually the first index to react; therefore, it can be used for the above purposes. Other drugs (e.g., barbiturates, phenytoin) can also increase serum GGT levels. It should also be noted that in DSM-5, patients are diagnosed with alcohol use disorder, alcohol intoxication, or alcohol withdrawal.
question
A 44-year-old man with a history of polysubstance abuse is brought to the emergency department because of vomiting, sweating, shakiness, and agitation. His temperature is 38.6°C (101.5°F), pulse is 110/min and blood pressure is 149/93 mm Hg. Ophthalmic examination shows mydriasis. The patient exhibits overall confusion and is not responsive to questions. He appears undernourished, in distress, and is tremulous. He says that he is seeing monsters on the wall. Withdrawal from which of the following best explains this patient's behavior? A. Alcohol B. Caffeine C. Cannabis D. Cocaine E. Hallucinogens F. Heroin
answer
A
Get an explanation on any task
Get unstuck with the help of our AI assistant in seconds
New